TS Inter 1st Year Hindi Model Paper Set 1 with Solutions

Exploring a variety of TS Inter 1st Year Hindi Model Papers Set 1 is key to a well-rounded exam preparation strategy.

TS Inter 1st Year Hindi Model Paper Set 1 with Solutions

Time : 3 Hours
Maximum Marks: 100
सूचनाएँ :

  1. सभी प्रश्न अनिवार्य हैं ।
  2. जिस क्रम में प्रश्न दिये गये हैं, उसी क्रम से उत्तर लिखना अनिवार्य है ।

खंड- ‘क’
(60 अंक)

1. निम्न लिखित किसी एक दोहे का भावार्थ लिखिए । 1 × 6 = 6

सोना सज्जन साधुजन, दूटि जुटै सौ बार ।
दूर्जन कुंभ कुम्हार का, एकै धका दरार ॥
उत्तर:
भावार्थ: कबीरदास इस दोहे में “सज्जन के गुण” कैसे होते है इसके बारे में हमें बता रहे है। सोने को अगर सौ बार भी तोड़ा जाए, तो भी उसे फिर जोड़ा जा सकता है। इसी तरह भले मनुष्य हर अवस्था में भले ही रहते हैं । इसके विपरीत बुरे या दुष्ट लोग कुम्हार के घड़े की तरह होते हैं जो एक बार टूटने पर दुबारा नही जुड़ता । बुरे आदमी के स्वभाव हमेशा दूसरों की प्रती बुरा ही होता है ।

(अथवा)

तुलसी साथी विपत्ति के, विद्या, विनय, विवेक ।
साहस, सुकृति, सुसत्य क्रत, राम भरोसो एक ॥
उत्तर:
भावार्थ : तुलसीदास इस दोहे में “विधा, विनय, विवेक जैसे गुणों के महत्व” के बारे में बता रहे हैं । तुलसीदास जी कहते हैं कि विपत्ति में अर्थात मुश्किल वक्त में ये चीजे मनुष्य का साथ देती है, वे है ज्ञान, विनम्रता पूर्वक व्यवहार, विवेक, साहस, अच्छे कर्म, आपका सत्य और राम (भगवान) का नाम । विपत्ति के समय शिक्षा, विनय, विवेक, साहस, अच्छे कार्य और सच्चाई ही साथ देते हैं । ये सभी मुश्किलों से हमें बचाते हैं ।

TS Inter 1st Year Hindi Model Paper Set 1 with Solutions

2. किसी एक कविता का सारांश लिखिए । 1 × 6 = 6

(1) समता का संवाद
उत्तर:
हमारा देश भारत माता का मंदिर है । यहाँ सबलोग समान है और सबकी वाणी एक ही है । यहाँ पर सबका शुभ हो जाएगा और सभी को भारतमाता की कृपा मिलेगी ।

इसदेश में जाति, धर्म, संप्रदाय का कोई भेदभाव नही है । सभी को समान रूप में स्वागत किया जाता है और सब का समान आदर मिलजाता है। राम रहीम, बुद्ध, ईसा सबकी पूजा की जाती है । भिन्न-भिन्न संस्कृतियां होने पर भी सभी का समान गौरव और सभी से समान ज्ञान प्राप्त होता है । सभी लोग प्रेम को चाहते है, पर शत्रुता को नहीं । इसदेश में सभी का शुभ मंगल होगा और सबकी इच्छाएँ पूरी हो जाएँगी ।

इसदेश में अनेक तीर्थस्थल है । पर हम अपने हृदय को ही पवित्र बनाकर तीर्थस्थल बनाएँगे । यहाँ पर हम अजातशत्रु बनकर सब को मित्र बनाएँगे । हम अपने मन की रेखाओं से एक मित्र बनाते है । अनेक आदर्शों से हम अपने चरित्र का निर्माण करते है ।

भारत माता के समक्ष रहने वाले हम सब भाई – बहन है । हम सब उसी माँ के गोद से पले सन्तान है । हम सब लोग मिलकर सुख – दुख को बाँट देंगे। सभी का कल्याण होगा और सभी की इच्छाएँ पूरी हो जाएँगी ।

भारत माँ की सेवा में हम पूजारी है । उन्ही के कहने पर सब काम करते है । इस जीवन से लाभ उठाकर मुक्ति पाना हमारा कर्तव्य है । हम सब उसके अनुचर है । इस देश के करोडों लोग मिलकर भारतमाता का जयगान करेंगे । इस देश में हम सब का कल्याण होगा और हम सब पर उनकी कृपा रहेगी ।

इसप्रकार भारतदेश की महानता और भारतमाता को एक देवी के रूप में चित्रण करके उसके प्रति हमारा कर्तव्य निभाने का सन्देश कवि दे रहे है उनकी भाषा सरल खडीबोली है।

(2) दान – बल
उत्तर:
कवि का कहना है कि दान के देने से ही मानव जीवन निरंतर पूर्णरूप से आगे चलता है । दान बल से स्नेह ज्योति उज्वलित होती है। रोते हुए या हँसते हुए जो दान देते है, जो अहंकार में पड़कर दान देते है और जो अपने स्वत्व को त्याग मानते हैं, उसका कोई फल नहीं मिलता ।

वास्तव में त्याग देना स्वत्व का त्याग नही है, यह जीवन की सहज क्रिया मात्र है। जो अपनी संपत्ति को रोक लेता है वह जीवित रहते हुए भी मृतक के समान है । अर्थात् जो दूसरों को दान या मदद नहीं करते वह मरे हुए व्यक्ति के समान है । कवि वृक्ष का उदाहरण देते हुए कहते है कि वृक्ष किसी पर कृपा दिखाने के लिए फल नहीं देता है। यदि वृक्ष फल को गिरने से रोक देता है और ऋतु चले जाने के बाद भी फल डाल पर हो रखता है तो ये फल सड जाते है और उससे कीटाणु निकल कर डालों को ही नही, सारे वृक्ष को नाश कर देता है । इसलिए वृक्ष फल को त्याग देता है तो उसके बीजों से नसे पौधे पैदा होते है ।

नदी का उदाहरण देते हुए कवि कहते हैं कि नदि भी अपने पानी को नहीं रोकती है । नदी का पानी भाप बनकर बादलो का रुप लेता है और बरसकर पानी उसी नदी में मिल जाता है । इसलिए जो भी हम देते है उसका संपूर्ण फल हमें प्राप्त होता है ।

इस प्रकार कवि का मानना है कि दान एक प्राकृतिक धर्म है । दान देने मे मनुष्य व्यर्थ ही डरता है। हर एक को किसी न – किसी दिन सब छोडकर जाना ही है । इसलिए समय का ज्ञान समज़ेकर हमे सब कुछ समय पर दान देना चाहिए। नहीं तो जब मृत्यु आती है तो अपना सर्वस्व छोडकर भी लाभ नहीं मिलता ।

इस प्रकार दान देना मनुष्य का सहज स्वभाव होना चाहिए । यह कोई उपकार नहीं है । इस कर्तव्य को निभाना हमारा कर्तव्य है । उनकी भाषा सरल खडीबेली है ।

3. किसी एक पाठ का सारांश लिखिए । 1 × 6 = 6

(1) शिष्टाचार
उत्तर:
लेखक परिचय : ‘शिष्टाचार’ निबंध का लेखक रामाज्ञा द्विवेदी ‘समीर’ है । आप ‘समीर’ उपनाम से साहित्यक रचनाएँ करते थे । इनका सबसे बड़ा योगदान अवधी भाषा में निर्मित ‘अवधी शब्द ‘कोश’ था । आज भी हिंदी की उपभाषा अवधी से संबंधित किसी भी रचना को अच्छी तरह से समझने केलिए इस शब्दकोश का उपयोग किया जाता है। इसके अतिरिक्त उन्होंने कई फुटकल रचनाएँ भी की हैं ।

सारांश : ‘शिष्टाचार’ दो शब्दों ‘शिष्ट’ एवं ‘आचार’ के मेल से बना है । शिष्ट का अर्थ होता है – ‘सभ्य, उचित अथवा सही’ तथा आचार का अर्थ होता है – ‘व्यवहार’ । व्यक्ति को हर समय इस बात का ध्यान रखना चाहिए कि वह किससे बात कर रहा है, कहाँ बात कर रहा है एवं क्यों बात कर रहा है । सामान्य व्यवहार ही नही बातचीत करने के ढंग से भी व्यक्ति के व्यवहार का पता चल जाता है | इसलिए किसी भी बातचीत में शिष्टाचार के सामान्य नियमों का पालन करना चाहिए । यह भी ध्यान रखना चाहिए कि अलग अलग समाजों में शिष्टाचार के नियम भिन्न – भिन्न है यद्यपि उनके आधार प्रायः समान ही हैं ।

शिष्टाचार का सबसे पहला गुण है विनम्रता । हमारी वाणी में, हमारे व्यवहार में विनम्रता घुली होनी चाहिए | इसलिए किसी बड़े के बुलाने पर ‘हाँ’, ‘अच्छा’, ‘क्या’ न कहकर ‘जी हाँ’ या ‘जी नहीं’ कहना चाहिए | किसी की बात का उत्तर ऐसे नहीं देना चाहिए कि सुननेवालों के लगे कि लट्ठ मारा जा रहा है। विनम्र केवल बड़ों के प्रति नही होती। बराबर वालों और अपने से छोटों के प्रति भी नम्रता और स्नेह का भाव होना चाहिए। सभी से बोलते हुए हमारी वाणी में मिठास रहनी चाहिए, कटुता या कर्कशता नहीं । विनम्र केवल भाषा की वस्तु नहीं । हमारे कर्म में भी विनम्रता होनी चाहिए । अपने से बड़े व्यक्तियों के बैठ जाने के बाद ही हमें बैठना चाहिए। महिलाओं के प्रति हमारे व्यवहार में और भी विनम्रता का होना आवश्यक है । बस या रेल में किसी महिला को खड़ी देखकर अपनी सीट उन्हें बैठने केलिए दे देना शिष्ट आचरण है। बड़े लोगों के साथ चलते समय उनके आगे चलना भी अशिष्टता है। हाँ, उनकेलिए झट से आगे होकर दरवाज़ा खोल देना या राह दिखाना शिष्ट व्यवहार है ।

शिष्टाचार का दूसरा विशेष गुण है दूसरों की निजी बातों में दखल न देना । हर व्यक्ति का अपना एक निजी जीवन होता है । इसलिए हमें अकारण किसी से उसका वेतन, उम्र, जाति, धर्म आदि पूछने से बचना चाहिए । यदि कोई कुछ लिख रहा है, तो झाँक-झाँककर उसे पढ़ने की चेष्टा करना उजड्डपन कहा जाएगा। किसी के घर या दफतर जाने पर उसकी वस्तुओं को बिना पूछे उलटने – पुलटमे लगना अशिष्टता है । किसी का नाम लेने या लिखने के पहले श्री, श्रीमती या कुमारी लगाना अच्छी आदत है । कुछ लोग इनके स्थान पर पंडित, डॉक्टर, बाबू, लाला, मियाँ, मिर्ज़ा – जब जैसी आवश्यकता होती है । लगाते हैं । इसी तरह कुछ लोग नाम के बाद ‘जी’ लगाते हैं | यदि कोई कुछ कष्ट या असुविधा उठाकर हमारे लिए कोई काम करता है तो हमें उसके प्रति अपनी कृतज्ञता अवश्य प्रकट करनी चाहिए । इसका सबसे सरल तरीका है उसे धन्यवाद देना । यदि बस या रेल में कोई अपनी जगह हमें बैठने केलिए देता है तो उसे धन्यवाद अवश्य देना चाहिए |

शिष्टता का तीसरा आधार अनुशासन का पालन है । अनुशासन समाज के नैतिक नियमों का भी हो सकता है और कानून की धाराओं का भी । किसी मंदिर, गुरुद्वार या मस्जिद में जाने के पहले जूते उतार देना धार्मिक अनुशासन का पालन है। सड़क पर बाई ओर चलना या जहाँ जाना मना हो, वहाँ न जाना, कानून के अनुशासन का पालन है । हमें हर तरह के अनुशासनों का सामान्य ज्ञान होना ही चाहिए। जैसे किसी सभा में शोर मचाना अनुचित है । किसी वक्ता को अपनी बात कहने का मौका न देना अशिष्टता है । राष्ट्रगान के अवसर पर बैठे रहना या चलना या झूमना अशिष्ट व्यवहार है । जहाँ सब लोग बैठे हो वहाँ लेट जाना या पैर फैलाकर बैठना बहुत अनुचित है । अपने मन को संयम में रखना शिष्ट व्यवहार केलिए अत्यंत आवश्यक है |

संक्षेप में हम कह सकते हैं कि शिष्टाचार वह व्यवहार है जिसके करने पर दूसरों के तथा अपने मन को प्रसन्नता होती है । इसके विपरीत अशिष्ट व्यवहार से दूसरों का दिल दुखता है और उससे अंत में हानि भी होती है। एक बेहतरीन इंसान अपनी जुबान से ही पहचाना जाता है ।

विशेषताए :

  1. बड़ों का आदर सम्मान करना, अपने मित्रों एवं सहयोगियों के प्रति सहयोगात्मक रवैया, छोटों के प्रति स्नेह की भावना, सकारात्मक विचारधारा, स्थान-विशेष के अनुकूल व्यवहार इत्यादि शिष्टाचार के उदाहरण है । शालीनता एवं विनम्रता शिष्टाचार के ही घटक हैं ।
  2. सार्वजनिक स्थल पर धूम्रपान, किसी व्यक्ति से गाली-गलौज, अपने साथियों से अकड़कर बात करना, राह चलते किसी से बिना वजह झगड़ना इत्यादि अशिष्ट आचरण के उदाहरण है । Good manners and soft words have brought many a difficult thing to pass.

(2) बतुकम्मा
उत्तर:
1. प्रस्तावना (बतुकम्मा का अर्थ क्या है) : भारत त्यौहारों का देश है । यहाँ लगभग हर राज्य के अपने – अपने राज्य पर्व हैं । उसी तरह ‘बतुकम्मा’ तेलंगाणा राज्य का राज्य पर्व है । तेलंगाणा राज्य सरकार ने 24 जुलाई, 2014 के दिन सरकारी आदेश संख्या 2 के अनुसार इसे राज्य पर्व के रूप में गौरवान्वित किया है। हर वर्ष धूम-धाम, श्रद्धा और भक्ति के साथ मनाया जाने वाला त्योहार ही बतुकम्मा |

बतुकम्मा तेलुगु भाषा के दो शब्दों से बना है- ‘बतुकु’ और ‘अम्मा’ । यहाँ ‘बतुकु’ का अर्थ ‘जीवन’ और ‘अम्मा’ का अर्थ ‘माँ है। इस तरह बतुकम्मा का अर्थ है – ‘जीवन प्रदायिनी माता’ । बतुकम्मा त्यौहार तेलंगाणा राज्य की वैभवशाली संस्कृति का प्रतीक है । बतुकम्मा त्यौहार दशहरे की नवरात्रियों में मनाया जाता है । यह कुल नौ दिन का त्यौहार है । इसका आरंभ भाद्रपद अमावस्या यानी महालया अमावस्या या पितृ अमावस्या से होता है ।

2. पौराणिक गाथाएँ : वेमुलवाडा चालुक्य राजा, राष्ट्रकूट राजा के उप-सामंत थे, चोला राजा और राष्ट्रकूट के बीच हुए युद्ध में चालुक्य राजा ने राष्ट्रकूट का साथ दिया था । 973 AD में राष्ट्रकूट राजा के उपसामंत थैलापुदु द्वितीय ने आखिरी राजा कर्कुदु द्वितीय को हरादिया और अपना एक आजाद कल्याणी चालुक्य साम्राज्य खड़ा किया, अभी जो तेलंगाणा राज्य है, वो यही राज्य है । वेमुलवाड़ा के साम्राज्य के समय राजा राजेश्वर का मंदिर बहुत प्रसिद्ध था । तेलंगाणा के लोग इनकी बहुत पूजा आराधना करते थे । चोला के राजा परान्तका सुंदरा, राष्ट्रकूट राजा से युद्ध के समय घबरा गए थे। तब उन्हें किसी ने बोला कि राजाराजेश्वर उनकी मदद कर सकते थे, तो राजा चोला उनके भक्त बन गए । उन्होंने अपने बेटे का नाम भी राजराजा रखा। राजराजा चोला ने 985 – 1014 AD तक शासन किया। उनके बेटे राजेन्द चोला जो सेनापति थे, सत्यास्त्राया में हमला कर जीत हासिल की। अपनी जीत की निशानी के तौर पर उसने राजा राजेश्वरी मंदिर तुड़वा दिया और एकबड़ी शिवलिंग अपने पिता को उपहार के तौर पर दी । 1006 AD में राजराजा चोला इस शिवलिंग के लिए एक बड़े मंदिर का निर्माण शुरु करते है, 1010 में बृहदीश्वर नाम से मंदिर की स्थापना होती है, वेमुलावाडा से शिवलिंग को तन्जावूरु में स्थापित कर दिया गया, जिससे तेलंगाणा के लोग बहुत दुखी हुए।

तेलंगाणा छोड़ने के बाद बृहदम्मा (पार्वती) के दुःख को कम करने के लिए बतुकम्मा की शुरुवात हुई, जिस में फूलों से एक बड़े पर्वत की आकृति बनाई जाती है। इसके सबसे ऊपर हल्दी से गौरम्मा बनाकर उसे रखा जाता है। इस दौरान नाच, गाने होते है । बतुकम्मा का नाम बृहदम्मा से आया है। शिव व पार्वती को खुश करने के लिए ये त्यौहार 1000 साल से तेलंगाणा में बडी धूम धाम से मनाया जा रहा है । इसके अतिरिक्त एक पौराणिक कहानी भी है कहा जाता है कि चोल नरेश धर्मागंद और उनकी पत्नी सत्यवती के सौ पुत्र थे । वे सभी पुत्र एक महायुद्ध में मारे गए । इस दुख से उबरने के लिए राजा धर्मांगद और रानी सत्यवती ने कई पूजा-पाठ, यज्ञ आदि पूजन कार्य किए । फलस्वरुप उनके घर में साक्षात लक्ष्मीदेवी का जन्म हुआ । बचपन में घटी कई दुर्घटनाओं के बावजूद वह सुरक्षित बची रही । इसीलिए माता पिता ने उसका नाम ‘बतुकम्मा’ रख दिया । सभी लोग उसकी पूजा करना आरंभ किया ।

3. बतुकम्मा पर्व का महत्व : बतुकम्मा पर्व के पीछे एक खास उद्देश्य है, वर्षा ऋतु सभी जगह पानी आ जाता है, जैसे नदी, तालाब एंव कुँए भर जाते हैं, धरती भी गीली महिम सी हो जाती है और इसके बाद फूलों के रूप में पर्यावरण में बहार आती है । इसी कारण प्रकृति का धन्यवाद देने के लिए तरह- तरह के फूलों के साथ इस त्यौहार को मनाया जाता है, इसमें फोक रीजनल साँग अर्थात क्षेत्री गीत गाये जाते है । इन दिनों पूरे देश में ही उत्साह के पर्व मनाये जाते हैं, इन सबका उद्देश्य प्रकृति का अभिवादन करना ही होता है ।

4. बतुकम्मा पर्व कैसे मनाता और कैसे आचरण करता है : इस त्यौहार को मनाने के लिए नव विवाहित अपने मायके आती है। ऐसा कहा जाता हैं उनके जीवन में परिवर्तन के लिए यह प्रथा शुरु की गई.
a) पर्व के शुरुवाती पाँच दिनों में महिलाएँ अपने घर का आँगन स्वच्छ करती है, गोबर से आंगन को लिपा जाता है ।
b) सुबह जल्दी उठकर उस आँगन में सुंदर-सुंदर रंगौली डालती है ।
c) कई जगह पर एपन से चौक बनाया जाता है जिसमें सुंदर कलाकृति बनाई जाती है, चावल के आटे से रंगोली का बहुत महत्व है ।
d) इस उत्सव में घर के पुरुष बाहर से नाना प्रकार के फूल एकत्र करते हैं जिसमें तंगेडु, गुम्मडि पुव्वु, बंती, मंदारम, गोरिंटा, पोकाबंती, कट्लपाडु, गुंट्लागरगरा, चामंती, तामरा, गन्नेरु, गुलाबी, वज्रदंती, गड्डी पुव्वु आदि फूलों को एकत्र किया जाता हैं ।
e) फूलों के आने के बाद उनको सजाया जाता हैं । तरह-तरह की लेयर बनाई जाती हैं, जिसमें फूलों की पत्तियों को सजाया जाता है । इसे तांबालम (Thambalam) के नाम से जाना जाता है ।
f) बतुकम्मा बनना एक लोक कला है। महिलाएँ बतुकम्मा बनाने की शुरुवात दो पहर से करती है ।

आचरण (Celebration) :
a) नौ दिन इस त्यौहार में शाम के समय महिलाएँ, लड़कियाँ एकत्र होकर इस त्यौहार को मनाती हैं । इस समय ढोल बजाये जाते हैं ।
b) सब अपने- अपने बतुकम्मा को लेकर आती है ।
c) महिलाएँ पारंपरिक साड़ी और गहने पहनती है । लडकियाँ लहंगा चोली पहनती है ।
d) सभी महिलाएँ बतुकम्मा के चारों ओर गोला बनाकर क्षेत्रीय बोली में गाने गाती हैं, यह गीत एक सुर में गाये जाते हैं, इस प्रकार यह त्यौहार नौ दिनों तक मनाया जाता हैं। महिलाएँ अपने परिवार की सुख, समृद्धि, खुशहाली के लिए प्रार्थना करती है ।
e) हर एक दिन का अपना एक नाम है, जो नैवैद्यम (प्रसाद) के अनुसार रखा गया है ।
f) बहुत से नैवैद्यम बनाना बहुत आसान होता है, शुरु के आठ दिन छोटी बड़ी लडकियाँ इसे बनाने में मदद करती है ।
g) आखिरी दिन को सहुला बतुकम्मा कहते है, सभी महिलाएँ मिलकर नैवैद्यम बनाती है । इस अंतिम दिन बतुकम्मा को पानी में विसर्जित कर दिया जाता है ।

5. आनंद का पर्व : बतुकम्मा महीने भर समाज आनन्द – मग्न रहता है । नौ दिनों तक चलने वाले इस त्यौहार में स्त्रियों के करताल से सारा वातावरण गूंज उठता है। नीरस भी सरस बन जाता है। चारों तरफ हरियाली, पानी की भरपूर मात्रा, घर आँगन में खुशियों का वातावरण बनाये रखे । जीवन में महीने भर मानों आनन्द ही आनन्द बना रहता है ।

6. उपसंहार : भारत के सारे पर्वदिन आनन्द के ही त्योहार है । बतुकम्मा अधिक आनंदप्रद त्योहार है। रंक से लेकर रईस तक इसे मनाते हैं । बतुकम्मा त्योहार भारतीय समाज में स्त्रियों के गौरवशाली वैभव का गुणगान करता है । इस त्योहार से हमें यह पता चलता है, कि स्त्रियाँ न केवल ममता, प्रेम, समर्पण, त्याग की प्रतीक है, बल्कि समय आने पर समाज के हितों के लिए अपना सर्वस्व न्यौछावर करने के लिए तत्पर रहती है । यह त्योहार उस रूढिवादिता का विरोध करता है । जहाँ पुरुष को प्रधान माना जाता है । यह त्योहार स्त्री शक्ति को पहचानने, उनका आदर करने और समाज में उचिन स्थान देने पर बल देना है ।

“तेलंगाणा यदि शरीर है तो बतुकम्मा उसकी आत्मा । बतुकम्मा के बिना तेलंगाणा राज्य की कल्पना करना असंभव है” ।

TS Inter 1st Year Hindi Model Paper Set 1 with Solutions

4. किन्हीं दो प्रश्नों के उत्तर तीन या चार वाक्यों में लिखिए | 2 × 4 = 6

1) खड्गसिंह का चरित्र चित्रण कीजिए ।
उत्तर:
खड्गसिंह उस इलाके का प्रसिद्ध डाकू था । लोग उसका नाम सुनकर काँपते थे । होते-होते सुल्तान की कीर्ति उसके कानों तक भी पहुँची । वह एक दिन बाबा भारती के पास आया । उसने घोड़ा देखा, तो उसपर उसे बड़ा मोह हो गया। किसी न किसी सुल्तान को हड़पने की ठान ली । जाते-जाते उसने कहा- बाबाजी इस घोड़े को आपके पास रहने नही दूँगा ।

खड्गसिंह अपाहिज वेष धारण करके बाबा को धोखा दिया । घोड़े को अपना साथ ले गया । बाबा की करुण वचनों से अपना मन परिवर्तित होता है । अंत में उसने सुल्तान (घोड़े ) को बाबा तक पहुँचाता है । डाकू को भी हृदय होता है । डाकू भी सामान्य मानव जैसा सोचता है । इस प्रकार की आलोचना हमें खड्गसिंह चरित्र द्वारा मालूम होता है ।

2) रवींद्र का लक्ष्य क्या था ।
उत्तर:
रवींद्र एक छोटो से गाँव का लड़का था । गरीब परिवार का रवींद्र खुद अपने बलबूते पर पढ़ता
चला गया । हर परिक्षा में प्रथम रहता था । दसवी कक्षा में प्रथम आने पर रवींद्र का उत्साह दुगना हो गया और अधिक लगन से पढ़ने लगा । उसका एक मात्र लक्ष्य खूब पढकर आई.ए.एस बनना है । उसने अपना ध्यान आई. ए. एस की परीक्षा पास कर एक अधिकारी बनने पर केन्दित किया । अंत में कलक्टर बन जाता है ।

3) ध्यान चंद को ‘हाँकी का जादूगर’ क्यों कहा जाता है ?
उत्तर:
26 मई 1928 को ध्यानचंद समेत कई खिलाड़ियों की तबीयत खराब थी। लेकिन उनके हौंसलों
में किसी तरह की कमी नही थी। वो टीम वर्ल्ड चैम्पियन बन चुकी थी, जो उधार लेकर ओलंपिक खेलने आई थी । बर्लिन ओलंपिक में लोग मेरे हाँकी खेलने के ढंग से इतने प्रभावित हुए कि उन्होंने मुझे हाँकी का जादूगर कहना शुरु कर दिया । इसी ओलंपिक के बाद पहली बार ध्यानचंद के नाम के साथ ‘जादुगर’ शब्द जोडा गया ।

4) वनों को नष्ट करने से होनेवाले दुष्परिणामों के बारे में लिखिए ।
उत्तर:
वनों की कटाई से मिट्टी, पानी और वायु क्षरण होता है जिसके परिणामस्वरुप हर साल 16,400
करोड़ से अधिक वृक्षों की कमी देखी जाती है। वनों की कटाई भूमि की उत्पादकता पर विपरीत प्रभाव डालती है क्यों कि वृक्ष पहाडियों की सतह को बनाए रखने में एक महत्वपूर्ण भूमिका निभाते है तथा तेजी से बढती बारिश के पानी में प्राकृतिक बाधाएँ पैदा करते हैं। नतीजतन नदियों का जल स्तर अचानक बढ़ जाता है जिससे बाढ़ आती है। मिट्टी की उपजाऊ शक्ति की हानि होती है। वायु प्रदूषण होता है । प्रजातियां विलुप्त हो जाती है । ग्लोबल वार्मिगं हो जाता है । औषधीय वनस्पति प्राप्त करना दुर्लभ हो जाता है। ओजोन परत को नुकसान हो रहा है । जल संसाधन की कमी होती है ।

5. निम्नलिखित दो पद्यांशों की संदर्भ सहित व्याख्या कीजिए । 2 × 3 = 6

1) सब तीर्थो का एक तीर्थ यह …………… एक चरित्र बना ले हम |
उत्तर:
यह पद्य ‘समता का संवाद’ नामक कविता से लिया गया है । इसके कवि मैथिलीशरण गुप्त जी है । सब को आदर्शमय जीवन बिताने का सन्देश कवि देते हैं ।

कवि का कहना है कि हमारे देश मे अनेक तीर्थ स्थल है । उनके समान हमारे हृदय को भी पवित्र बनाएंगे । हम अजातशत्रु बनकर सबसे मित्रता करेंगे । हमारे मनोभावों को एक निश्चित रूप देंगे और उनसे हमारे चरित्र आदर्श बनाएंगे । कवि की भाषा सरल खडीबोली है ।

2) यह मेरी गोदी की शोभा, सुख सुहाग की है लाली । शाही शान भिखारन की है, मनोकामना मतवाली । दीप – शिखा है अंधकार की, बनी घटा की उजियाली । उषा है यह कमल भृंग की, है पतझड़ की हरियाली ।
उत्तर:
यह पद्य ‘बालिका का परिचय’ नामक कविता से लिया गया है । इसकी कवइत्री सुभद्रा कुमारी चौहान जी है । इसमे नारी चेतना का स्वर स्पष्ट होती है ।

कवइत्री कहती है कि बालिका मेरी गोद की शोभा है और सौभाग्य प्रदान करनेवाली है । वह मेरी मनोकामना का प्रतिफल है । माँ जितनी सम्पन्न होने पर भी बालिका के सामने भिखारिन ही है । वह अन्धकार में दीपशिखा की तरह, कालीघटा में प्रकाश की तरह है । वह पतझड की हरियाली में, कमल भौरों में उषा की पहली किरण जैसी है । अपनी बालिका हो जीवन का सूर्योदय है। उनकी भाषा सरल खडीबोली है ।

3) प्रथम रश्मि का आना रंगिणि, तूने कैसे पहचाना ?
कहाँ, कहाँ हे बाल विहंगिनि
पाया तूने वह गाना |
उत्तर:
यह पद्य ‘प्रथम रश्मि’ नामक कविता से लिया गया है । सुमित्रानंदन पंत इसके कवि है । सूर्योदय को सुन्दर वर्णन कवि इसमे कर रहे है । कवि इसमे बाल विहंगिनि से पूछ रहा है। अभी तुमने नींद से जाग लिया । तुम्हे प्रातः काल के किरणों की पहचान कैसे हुई ? यह जानकर तुम इतना सुन्दर केसे गा रही हो । प्रकृति की सहज सुन्दरता इसमे वर्णित है । भाषा सरल खडीबोली है ।

4) ऋतु के बाद फलों का रुकना डालों का सड़ना है, मोह दिखाना देय वस्तु पर आत्मघात करना है । देते तरु इसलिए कि रेशों में मत कीट समायें, रहें डालियाँ स्वस्थ और फिर नये नये फल आयें ।
उत्तर:
यह पद्य ‘दान – बल, नामक कविता से लिया गया हैं । यह कविता रश्मिरथी नामक काव्य से लिया गया है । इसके कवि श्री रामधारी सिंह दिनकर जी है ।

इसमे दान की महानता को व्यक्त करते हुए कवि वृक्ष का उदाहरण दे रहा है । वृक्ष ऋतु जाने के बाद स्वयं अपने फलों को छोड़ देती है। यदि नही छोडती तो वे फल डालों पर ही सड जाते है । उससे कीडे निकलकर सारा वृक्ष नाश हो जाता है । यदि फल को छोडता है तो उसके बीजों से नये पौधे और नये फल उत्पन्न होते है उसकी भाषा सरल खडीबोली है ।

TS Inter 1st Year Hindi Model Paper Set 1 with Solutions

6. निम्नलिखित किन्हीं दो गद्यांशों की संदर्भ सहित व्याख्या कीजिए । 2 × 3 = 6

(1) यदि कोई कुछ कष्ट या असुविधा उठाकर हमारे लिए कोई काम करता है तो हमें उसके प्रति अपनी कृतज्ञता अवश्य प्रकट करनी चाहिए ।
उत्तर:
संदर्भ : यह वाक्य ‘शिष्टाचार’ नामक पाठ से लिया गया है । यह पाठ एक सामाजिक निबंध है | इसके लेखक रामाज्ञा द्विवेदी ‘समीर’ जी हैं । वे ‘समीर’ उपनाम से साहित्यिक रचनाएँ करते थे | हिंदी के शब्द भंडार को समृद्ध करने की दृष्टि से उन्होंने ‘अवधी’ शब्दकोश का निर्माण किया था | इसके अतिरिक्त उन्होंने कई कुटकल रचनाएँ भी की हैं ।

व्याख्या : कुछ लोग खुद मुसीबतों में रहने पर भी, हम को सहायता करने आते हैं । उन लोगों के प्रति हम अपनी कृतज्ञता अवश्य प्रकट करनी चाहिए । कृतज्ञता कैसा प्रकट करना है यह बात हमारी मन में आता है । इसका सबसे सरल तरीका है उसे धन्यवाद देना । ‘धन्यवाद’ शब्द बोलते समय ऐसा लगना चाहिए कि हम उसे हृदय से धन्यवाद दे रहे हैं, केवल ऊपर-ऊपर से नहीं । जिस आदमी कष्ट में हमारी सहायता किया है उसको याद रखकर कभी उस आदमी को काम आने पर हमें भी उसकी सहायता करनी चाहिए ।

विशेषताएँ: प्रस्तुत निबंध ‘शिष्टाचार’ एक उपदेशात्मक निबंध है । धन्यवाद तभी बताना चाहिए कि जब किसी ने आपके लिए कुछ किया हो । जो कोई बात आपको मालुम नहीं, उसी बात को आप को समझाएँ तो धन्यवाद बताकर कृतज्ञता प्रकट करनी चाहिए। शिष्टाचार का सबसे महान गुण जो आज हम बच्चों को जरूर समझाना चाहिए कि राष्ट्रगान का आदर करना चाहिए |

(2) आखिर हम उठते हैं। लड़के से कहते हैं, “अच्छा अब हम जाते हैं । कह देना कि हम आए थे ।”
उत्तर:
संदर्भ : ये वाक्य ‘समय पर मिलने वाले’ नामक पाठ से दिये गये हैं । इस पाठ के लेखक ‘हरिशंकर परसाई’ जी हैं। आप हिंदी गद्य – साहित्य के व्यंग्यकारों में अग्रगण्य हैं । सामाजिक और राजनैतिक क्षेत्र में फैली विसंगतियों पर अपना लेख लिखता हैं । परसाई जी के व्यंग्यपरक निबंध पाठकों को सचेत करते हैं । प्रस्तुत पाठ एक व्यंग्य रचना है जिसमें एक व्यक्ति दूसरे व्यक्तियों के समय कैसा बरबाद करते है, इसका व्यंग्यपूर्ण चित्रण मिलता है ।

व्याख्या : एक दिन लेखक के मित्र लेखक से सुबह आठ बजे अपना घर पर मिलने का वादा किया था; पर मित्र घर पर नही हैं । लेखक मित्र केलिए उस का घर में इंतजार कर रहे हैं। मित्र के पुत्र लेखक के पास बैठकर पुस्तक पढ रहा है । बहुत देर तक रहने पर भी मित्र नही आता है । घर के अंदर से स्त्रियाँ लेखक के बारे में भला-बुरा कहता है । लेखक सभी बातें सुनकर लज्जित हो जाता है । अखिर उठ कर मित्र के पुत्र से कहते है, अच्छा मै अब जा रहा हूँ। तुम्हारे पिताजी आने के बाद कहदेना कि आपसे मिलने आपका दोस्त आया था |

विशेषताएँ :

  1. सामान्य आदमी समय को काटने के बारे में सोचता है, जबकि महान व्यक्ति सोचते हैं इसके उपयोग के बारे में ।
  2. दुनिया में जितनी भी चीजे हैं, उन सभी में समय समाया हुआ है ।

(3) मेधावी पुत्री की विलक्षण बुद्धि ने फिर मुझे चमत्कृत कर दिया है। सरस्वती जैसे आकर जिह्वाग्र पर बैठ गयी थी ।
उत्तर:
संदर्भ : यह वाक्य ‘अपराजिता’ नामक कहानी से दिया गया है। इसकी लेखिका ‘गौरा पंत शिवानी’ जी है । भारत सरकार ने सन् 1982 में उन्हे हिंदी साहित्य में उत्कृष्ट सेवा के लिए पद्ममश्री पुरस्कार से सम्मानित किया । शिवानी जी की अधिकरतर कहानियाँ और उपन्यास नारी प्रधान रहे । प्रस्तुत कहानी अपराजिता में लेखिका ‘डाँ. चंद्रा’ नामक एक अपंग युवती की जीवन संबंधी विषयों के बारे में हमें बतायी ।

व्याख्या : डॉ. चंद्रा की माँ श्रीमती टी. सुब्रह्मण्यम चंद्रा के बारे में इस तरह कह रही है कि “चंद्रा को अठारह वे महीने में ज्वर आया । बाद में पक्षाघात हुआ । गरदन के नीचे सर्वांग अचल हो गया था । बड़े-से-बड़े डाक्टर को दिखाया, कोई लाभ नही हुआ। अंत में एक सुप्रसिद्ध आर्थोपेडिक के पास लेगया । एक वर्ष तक कष्टसाध्य उपाचार चला । अचानक एक दिन चंद्रा की ऊपरी धड़ में गति आगयी” । पाँच वर्ष तक (टी श्रीमती सुब्रह्मण्यम ) माँ ही बेटी को शिक्षिका बनकर पढायी । चंद्रा बहुत मेधावी थी। चंद्रा की विलक्षण बुद्धि को देखकर अपनी माँ आश्चर्य हो जाती थी। स्वयं सरस्वती ही चंद्रा की जिह्वा पर बैठी थी ऐसी सोचती थी। चंद्रा एक बार सुनती तो कभी नही भूलती थी । चंद्रा ‘एकाग्रचित ‘ थी ।

विशेषताएँ :

  1. अपंगों को शिक्षा से जोड़ना बहुत जरूरी है ।
  2. शिवानी की कृतियों में चरित्र – चित्रण में एक तरह का आवेग दिखाई देता है ।
  3. डाँ. चंद्रा जैसे अपंग युवती सबकी मार्गदर्शिका है ।

(4) रूई की पतली पत्ती दूध से भिगोकर जैसे- जैसे उसके नन्हें से मुँह में लगाई पर मुँह खुल न सका और दूध की बूँदे दोनों ओर दुलक गई ।
उत्तर:
संदर्भ : यह वाक्य ‘गिल्लू’ नामक पाठ से दिया गया है । इस पाठ की लेखिका ‘श्रीमती महादेवी’ वर्मा है । आप हिन्दी साहित्य के आधुनिक काल में छायावाद युग की प्रसिद्ध कवइत्री एवं ख्यातिप्राप्त गद्य – लेखिका है । आपको ‘आधुनिक मीराबाई’ कहा जाता है । ‘नीहार’, ‘नीरजा’, ‘रश्मि’, ‘सन्ध्यागीत’, आदि आपके काव्य है। ‘स्मृति की रखाएँ’, ‘अतीत के चलचित्र’, ‘रटंखला की कडियाँ’ आदि आपकी प्रख्यात गद्य रचनाएँ हैं । आपको ‘यामा’ काव्य पर भारतीय ज्ञानपीठ का पुरस्कार प्राप्त हुआ । प्रस्तुत पाठ में एक छोटी जीव की जीवन का चित्रण करती हैं |

व्याख्या : महादेवी वर्मा के घर में एक दिन बरामदे से तेज आवाज आने लगी । तब लेखिका बाहर आकर देखती है । दो कौए गिलहरी को खाने का प्रयत्न करते हैं । लेखिका उस छोटे जीव को कौओं से बचाकर घर के अंदर ले आती है । गिलहरी के शरीर पर हुए घावों पर पेंसिलिन मरहम लगाती है | उसे खिलाने या पिलाने की प्रयत्न करती है। दूध पिलाने केलिए रूई को दूध से भिगोकर, उसके नन्हे से मुँह में लगाती है, पर मुँह खुला सका । तब दूध की बूँदे ढुलक जाती है । अंत मैं कई घंटे के बाद उसके मुँह में टपकाया जाता है । गिलहरी को बचाने के लिए बहुत प्रयास करती है ।

विशेषताएँ :

  1. किसी भी प्राणी आपत्ति में रहने पर उसकी सहायता करना हमारा कर्तव्य है ।
  2. छोटे जीवों के प्रती भी लेखिका के मन में करुण भावना है ।
  3. कई घंटे उस छोटी जीव की उपचार करती रही, अंत में सफल हुई |

7. किन्हीं दो प्रश्नों के उत्तर लिखिए | 2 × 3 = 6

1) तुलसी के अनुसार मीठे वचन बोलने से क्या लाभ हैं ?
उत्तर:
तुलसी के अनुसार मीठे वचन बोलने से चारों ओर खुशियाँ फैल जाती हैं सब कुछ खुशहाल रहता है। मीठी वाणी से कोई भी इंसान किसी को भी अपने वश में कर सकता है। शत्रु को भी अपना मित्र बनाते है |

मधुर वाणी सभी ओर सुख प्रकाशित करती है और यह हर किसी को अपनी ओर सम्मोहित करने का कारगर मंत्र है | इसलिए हर मनुष्य को कटु वाणी त्याग कर मीठे बोल बोलना चाहिए ।

2) माँ के लिए बेटी किसके समान है ?
उत्तर:
माँ के लिए बेटी गोदी की शोभा है और सौभाग्य प्रदान करती है। वह अपने अंधकारमय जीवन के लिए दीपशिखा की तरह है । माँ जीवन मन उषा की पहली किरण है । नीरस मन में अमृत की धारा और रस भरने वाली है – वह बालिका नष्ट नयनों की ज्योति है और तपस्वी को मन की सच्चीलगन है । एक माँ के लिए उसकी अपनी संतान है सबकुछ होती है । माँ और बेटी में भेद न करने की भावना समाज को उन्नति के शिखर पर पहुँचा सकती है ।

3) कवि ने प्रातः काल का वर्णन किस प्रकार किया है ?
उत्तर:
पंत जी ने प्रातः काल का सुन्दर वर्णन किया है । उषा काल मे सूरज की प्रथम किरण धरती को छूने से कितने सुन्दर परिवर्तन होते है, उनका सुन्दर वर्णन किया है । सूर्योदय के स्वागत में नन्ही सी पक्षी की मधुर आवाज मे गाना, नन्ही सी कलियों का चन्द्रके किरण तितलियों के रूप मे स्पर्श करने से मुस्कुराना, रात के चमकीले तारे मन्द पड़ जाना, सूर्योदय के स्वागत करते हुए कोयल का गाना सभी का सुन्दर वर्णन करके कवि यह प्रश्न कर रहा है कि सुर्योदय के आगमन के बारे मे इन सबको कैसा पता चल रहा है ।

4) फलों का दान करने से पेड़ को क्या लाभ होता हैं ?
उत्तर:
दान देना एक स्वाभाविक प्रक्रिया है । इसका समर्थन करते हुए कवि कहते है । कि वृक्ष फल देता है । यह कोई दान नहीं है । यदि ऋतु जाने के बाद वृक्ष फल को नहीं छोडता है तो फल उसी डाल पर सड़ जाते है । इससे कीडे निकलकर सारा वृक्ष नाश हो जाता है । यदि वृक्ष फल को गिरा देता है तो फिर नए फल आते है और उस फल के बीजों से नये पौधों उत्पन्न होते है ।

TS Inter 1st Year Hindi Model Paper Set 1 with Solutions

8. किन्हीं दो प्रश्नों के उत्तर लिखिए | 2 × 3 = 6

1) शिष्टाचार के कितने गुण हैं, वे कौन कौन से हैं ?
उत्तर:
शिष्टाचार के मुख्यतः तीन गुण हैं। वे हैं विनम्रता, दूसरों की निजी बातों में दखले न देना, अनुशासन ।

शिष्टाचार का सबसे पहला गुण है, विनम्रता । हमारी वाणी में, हमारे व्यवहार में विनम्रता धुली होनी चाहिए । इसलिए किसी बड़े के बुलाने पर ‘हाँ’, ‘अच्छा’, ‘क्या’ न कहकर ‘जीहाँ’ (या) जीनही कहना चाहिए । विनम्रता केवल बडों के प्रति नहीं होनी चाहिए । बराबरवालों और अपने से छोटों के प्रति भी नम्रता और स्नेहकाभाव होना चाहिए ।

शिष्टाचार का दूसरा विशेष गुण है दूसरों की निजी बातों में दखल न देना । हर व्यक्ति का अपना एक निजी जीवन होता है । इसीलिए हमें अकारण किसी से उसका वेतन, उम्र, जाति, धर्म आदि पूछने से बचाना चाहिए । यदि कोई कुछ लिख रहा है तो झाँक झाँक कर उसे पढ़ने की चेष्टा करना अच्छा नहीं है ।

शिष्टाचार का तीसरा आधार अनुशासन का पालन है । अनुशासन समाज के नैतिक नियमों का भी हो सकता है और कानून की धाराओं का भी । जहाँ जाना मना हो, वहाँ न जाना, कानून के अनुशासन का पालन है । ठीक समय पर कहीं पहुँचना अनुशासन भी सिखाता है ।

2) हरिशंकर परसाई की रचनाओं के नाम लिखिए ।
उत्तर:
हरिशंकर परसाई की प्रमुख रचनाएँ इस प्रकार हैं ।

निबंध संग्रह : तब की बात और थी, भूत के पाँव पीछे, बेईमानी की परत, पगडंडियों का जमाना, सदाचार का ताबीज, वैष्णव की फिसलन, माटी कहे कुम्हार से शिकायत मुझे भी है और अंत में, अपनी अपनी बिमारी, आवारा भीड़ के खतरे – ऐसा भी सोचा जाता है आदि उनके उल्लेखनीय निबंध संग्रह हैं ।

कहानी संग्रह : जैसे उसके दिन फिरे, दो नाकवाले लोग, हसते हैं रोते हैं, भोलाराम का जीव ।
उपन्यास : तट की खोज, रानी नागफनी की कहानी, ज्वाला और जल उनके प्रसिद्ध उपन्यास हैं ।

3) चंद्रा की माँ को ‘वीर जननी’ का पुरस्कार क्यों दिया गया ?
उत्तर:
चंद्रा की माँ श्रीमती टी. सुब्रह्मण्यम थी । वह एक साहसी जननी है । चंद्रा की माध्यमिक और काँलेज शिक्षा में बेटी के साथ रहकर पूरी कक्षाओं में अपंग पुत्री की कुर्सी की परिक्रमा स्वयं कराती । बचपन में चंद्रा को देखकर अपनी आत्मशक्ति खो नही बेठी । अपने आप को संभाल कर चंद्रा को भी संभाली। हर एक पल बेटी की कामना पूरी करने की कोशिश किया । चंद्रा की माँ अपने सारे सुख त्यागकर, नित्य छायाबनी । आज चंद्रा जो कुछ नाम प्राप्त किया सबकी वजह उसकी माँ ही है । इसलिए जे. सी. बेंगलूर उसकी माँ को ‘वीर जननी’ का पुरस्कार दिया । सचमुच चंद्रा की माँ एक वीर जननी है।

4) गिल्लू किसका नाम है ? उसके बारे में आप क्या जानते हैं ?
उत्तर:
गिल्लू एक ‘गिलहरी का नाम है । सोन जूही में लगी पीली कली को देखकर लेखिका के मन में छोटे से जीव गिलहरी की याद आ गई, जिसे वह गिल्लू कहती थी। बचपन में लेखिका गिल्लू को कौओं से बचाती हैं | अपना घर में गिल्लू को रखती थी । गिल्लू केलिए एक हलकी डलिया लेकर, उसके अंदर रूई बिछाकर उसे तार से खिडकी पर लटका दिया । यह रही गिल्लू का घर । गिल्लू डलिया को स्वयं हिलाकर झूलता रहता था । गिल्लू की झब्बेदार पूँछ और चंचल चमकीली आँखे सबको विस्मित करते थे । जब लेखिका लिखने बैठती तब पैर तक आकर शोर मचाती थी । लेखिका गिल्लू को लंबे लिफाफे में रख देती थी। दो पंजों और सिर के अतिरिक्त सारा लघुगात लिफाफे के भीतर बंद रहता । बाहर सी गिलहरियों से खेलने जाती थी । लेखिका बाहर जाने के लिए गिल्लू के लिए मार्ग बनाती थी । गिल्लू दो वर्ष तक लेखिका के साथ रहती थी । परंतु एक दिन प्रभात की प्रथम किरण के स्पर्श के साथ ही वह किसी और जीवन में जागने केलिए सो गया। सुबह होते – होते गिल्लू की मृत्यु हो गई ।

9. एक शब्द में उत्तर लिखिए । 5 × 1 = 5

1) कबीरदास किस शाखा के प्रमुख कवि थे ।
उत्तर:
निर्गुण भक्ति शाखा ।

2) तुलसीदास के माता पिता का नाम लिखिए ।
उत्तर:
हुलसी और आत्मराम दुबे |

3) प्रथम रश्मि कविता मे पक्षियों के घोसलों के पास कौन पहरा दे रहे थे ।
उत्तर:
प्रथम रश्मि कविता में पक्षियों के घोसलों के पास जुगुनू पहरा दे रहे थे ।

4) समता शब्द का अर्थ क्या है ।
उत्तर:
समता शब्द का अर्थ समानता या बराबरी है ।

5) गोदी की शोभा कौन है ।
उत्तर:
माँ की गोदी की शोभा अपनी बेटी ही है ।

10. एक शब्द में उत्तर लिखिए । 5 × 1 = 5

1) परसाई जी के मित्र ने उन्हें कितने बजे खाने पर बुलाया ?
उत्तर:
परसाई जी के मित्र ने उन्हें 11 बजे खाने पर बुलाया ।

2) सेठ जी के बेटे का नाम क्या है ?
उत्तर:
बलराम ।

3) ‘अपराजिता’ पाठ की लेखिका का नाम क्या है ?
उत्तर:
गौरा पंत “शिवानी” जी है।

4) राजा धर्मागद की पत्नी का नाम क्या था ?
उत्तर:
सत्यवती ।

5) समाज में सभ्य बनकर रहने के लिए कौनसे नियम जनना आवश्यक है ?
उत्तर:
शिष्टाचार |

TS Inter 1st Year Hindi Model Paper Set 1 with Solutions

खंड – ‘ख’
(40 अंक)

11. निम्नलिखित गद्यांश पढ़ित । प्रश्नों के उत्तर एक वाक्य में लिखिए । 5 × 1 = 5

यह नितांत एक मूर्खता है कि हम प्रगति के नाम पर अपने वनों को नष्ट कर रहे हैं और पारिस्थितिक संतुलन को बिगाड़ रहे हैं । व्यापक स्तर की दूरगामी योजनाओं के अभाव में शहरीकरण ने अनेक शहरी तथा उपशहरी क्षेत्रों को व्यावसायिक जंगलों में बदल दिया है । जमीन के इस गलत उपयोग का एक गंभीर परिणाम है विभिन्न प्रकार के जीवों की समाप्ति । शहरों की वृद्धि कृषि के प्रसार, बाधों के निर्माण तथा वनों के विनाश से जंगली जीवों के आवास नष्ट हुए है । जीवों की बहुत सी प्रजातियों और उपप्रजातियों के विलुप्त होने का खतरा उत्पन्न हो गया है ।

प्रश्न :
1) वनों को काटना कैसा काम है ?
उत्तर:
वनों को काटना मूर्खता वाला काम है ।

2) यहाँ किस संतुलन के बारे में चर्चा हो रही है ?
उत्तर:
यहाँ पारिस्थितिक संतुलन के बारे में चर्चा हो रही है ।

3) किसके गलत उपयोग से गंभीर परिणाम होते हैं ?
उत्तर:
जमीनों के गलत उपयोग से गंभर परिणाम होते हैं ।

4) किनके आवास नष्ट हुए हैं ?
उत्तर:
जंगली जीवों के आवास नष्ट हुए हैं ।

5) इस अनुच्छेद के लेखक का नाम क्या है ?
उत्तर:
इस अनुच्छेद के लेखक का नाम एन मणिवासकम है ।

12. सूचना के अनुसार लिखिए । 8 × 1 = 8

(12.1 ) किन्हीं चार (4) शब्दों के विलोम शब्द लिखिए ।
(1) इच्छा
(2) राजा
(3) घृणा
(4) स्वर्ग
(5) कठिन
(6) उषा
उत्तर:
(1) इच्छा × अनिच्छा
(2) राजा × रंक
(3) घृणा × प्रेम
(4) स्वर्ग × नरक
(5) कठिन × सरल
(6) उषा × संध्या

(12.2 ) किन्हीं चार (4) शब्दों के समानार्थी शब्द लिखिए ।
(1) घोड़ा
(2) पेड
(3) क्रोध
(4) नदी
(5) राजा
(6) आँख
उत्तर:
(1) घोड़ा = अश्व, हथ, वाजि, घोटक, तुरंग, तुरंगम
(2) पेड = वृक्ष, तरु, झाड़, विटप
(3) क्रोध = रोष, गुस्सा, अमर्ष, नाराजगी
(4) नदी = सरिता, तटिनी, तरंगीणी, निर्झरिणी, सलिला
(5) राजा = नृप, नृपति, सम्राट, नरेश
(6) आँख = लोचन, नयन, नेत्र, चक्षु, दूग, विलोचन

TS Inter 1st Year Hindi Model Paper Set 1 with Solutions

13. किन्हीं आठ (8) शब्दों की शुद्ध वर्तनी लिखिए । 8 × 1 = 8

(1) भारथ
(2) बोजन
(3) हीन्दी
(4) पच्चिस
(5) गूण
(6) पूजारी
(7) आदमि
(8) इमलि
(9) धोका
(10) भूक
(11) सीक
(12) कन
उत्तर:
(1) भारथ – भारत
(2) बोजन – भोजन
(3) हीन्दी – हिंदी
(4) पच्चिस – पच्चीस
(5) गूण – गुण
(6) पूजारी – पुजारी
(7) आदमि – आदमी
(8) इमलि – इमली
(9) धोका – धोखा
(10) भूक – भूख
(11) सीक – सीख
(12) कन – कण

14. कारक चिह्नों की सहायता से रिक्तस्थानों की पूर्ति कीजिए । 8 × 1 = 8

1) वह बाज़ार …………… (में/से) सब्जी लाया ।
2) रामू …………….. (का / की) घर है ।
3) लड़के …………….. (ने / पर) पुस्तक पढ़ी |
4) ………….. ( राय / वाह) । वह बुरी तरह से घायल हो गया ।
5) पेड़ ………… ( से / के द्वारा ) पत्ता गिरा ।
6) लड़के ………….. (की / को) यहाँ बुलाओ ।
7) कक्षा में एक …………… ( पर / से) बढ़कर एक छात्र हैं ।
8) छत …………. (के लिए / पर) पंखा लटका हैं ।
उत्तर:
1) वह बाज़ार से सब्जी लाया ।
2) रामू का घर है ।
3) लड़के ने पुस्तक पढ़ी ।
4) वाह वह बुरी तरह से घायल हो गया |
5) पेंड़ से पत्ता गिरा ।
6) लड़के को यहाँ बुलाओ ।
7) कक्षा में एक से बढ़कर एक छात्र हैं ।
8) छत पर पंखा लटका हैं ।

15. निर्देश के अनुसार छ: (6) वाक्यों को शुद्ध कीजिए । 6 × 1 = 6

1) तुम चाय पीता है । (वाक्य शुद्ध कीजिए 1)
उत्तर:
तुम चाय पीते हो ।

2) पण्डिताइन मंदिर जा रही है। (रेखांकित शब्द का प्रत्यय लिखिए ।)
उत्तर:
आइन

3) नौकर बाज़ार गयी है । (रेखांकित शब्द का लिंग बदलकर लिखिए ।)
उत्तर:
नौकरानी बाजार गयी है ।

4) वातावरण अपकूल हैं । (रेखांकित शब्द का उपसर्ग लिखिए ।)
उत्तर:
अप

5) मै खेलता हूँ । (भविष्यत् काल मैं लिखिए ।)
ज.
मै खेलूँगा ।

6) ये राजु के घरों हैं । (रेखांकित शब्द का वचन बदलकर लिखिए ।)
उत्तर:
ये राजु के घर हैं |

7) तू तेरे भाई के साथ खेल । (वाक्य शुद्ध कीजिए 1)
उत्तर:
तू अपने भाई के साथ खेल ।

8) वह चतुरिक से काम करता है । (शुद्ध कीजिए ।)
उत्तर:
वह चतुरता से काम करता है ।

TS Inter 1st Year Hindi Model Paper Set 1 with Solutions

16. किन्हीं पाँच (5) वाक्यों का हिंदी में अनुवाद कीजिए । 5 × 1 = 5

1) I have read this book.
उत्तर:
यह किताब मै ने पढ़ी है ।

2 ) Bring those books.
उत्तर:
उन किताबो को लाओ ।

3) Does he eat ?
उत्तर:
क्या वह खाता है ?

4) I have got my haircut.
उत्तर:
मैं ने अपने बाल कटवाये हैं ।

5) I read.
उत्तर:
मै पढता हूँ ।

6 ) I prefer cricket to football.
उत्तर:
मै फुटबाल से क्रिकेट को अधिक पसंद करता हूँ |

7) He has finished writing a story.
उत्तर:
वह कहानी लिख चुका ।

8) You sit there.
उत्तर:
तुम वहाँ बैठो ।

TS Inter 1st Year Political Science Question Paper May 2022

Varied difficulty levels in TS Inter 1st Year Political Science Question Papers and TS Inter 1st Year Political Science Question Paper May 2022 cater to students with diverse academic strengths and challenges.

TS Inter 1st Year Political Science Question Paper May 2022

Time: 3 Hours
Max. Marks: 100

Section – A
3 x 10 = 30

Note:

  • Answer ANY THREE of the following questions in 40 lines.
  • Each Question carries TEN marks.

Question 1.
Define Political Science and explain the scope.

Question 2.
Define State and discuss its essential Features.

Question 3.
Write an essay on basic ideas of Gandhism.

Question 4.
Define Democracy and explain its features.

TS Inter 1st Year Political Science Question Paper May 2022

Question 5.
Identify the safeguards of Rights.

Question 6.
What is Sovereignty and explain its characteristics.

Question 7.
What is executive? What are its functions?

Section – B
8 x 5 = 40

Note:

  • Answer ANY EIGHT of the following questions in 20 lines.
  • Each Question carries FIVE marks.

Question 8.
Explain the difference between state and Government.

Question 9.
Write a brief note on evolution of Political Science.

Question 10.
What are the difference between State and Society?

Question 11.
Explain any four kinds of Sovereignty.

Question 12.
What is equality? Explain its characteristic features.

Question 13.
Write about any three Sources of Law.

TS Inter 1st Year Political Science Question Paper May 2022

Question 14.
What are the characteristics of liberty?

Question 15.
Explain any three kinds of equality.

Question 16.
Describe any three types of Justice.

Question 17.
Explain Political Rights.

Question 18.
What is the role of Public Opinion in Democracy?

Question 19.
What are the features of a secular state?

Question 20.
What are the features of Rights?

Question 21.
What is direct democracy and mention its devices?

Question 22.
What are the merits and demerits of a written constitution?

Question 23.
Distinguish between written and unwritten constitutions.

Question 24.
What are the reasons for the decline of the significance of legislature?

Question 25.
Discuss Judicial Review.

Section – C
15 x 2 = 30

Note:

  • Answer ANY FIFTEEN of the following questions in 5 lines.
  • Each Question carries TWO marks.

Question 26.
Post Behaviouralism

Question 27.
Master Science.

Question 28.
Behaviorism.

TS Inter 1st Year Political Science Question Paper May 2022

Question 29.
Internal Sovereignty.

Question 30.
What is Society?

Question 31.
What is Constitutional law?

Question 32.
What about political Liberty?

Question 33.
What is Economic Equality?

Question 34.
Marxism.

Question 35.
Moral Rights?

Question 36.
Classify Rights?

Question 37.
Natural Rights

Question 38.
What is indirect Democracy?

Question 39.
Define Citizenship.

Question 40.
What is recall?

Question 41.
What are Referendum?

Question 42.
Define Secularism.

Question 43.
What in Theocratic State.

Question 44.
Define Secular State.

Question 45.
What do you mean by the Constitution?

Question 46.
What is on unwritten constitution?

Question 47.
What is Riged constitution?

TS Inter 1st Year Political Science Question Paper May 2022

Question 48.
What is meant by the theory of separation of powers?

Question 49.
How many organs of government are there? What are they?

Question 50.
House of Lords.

TS Inter 1st Year Political Science Question Paper March 2023

Varied difficulty levels in TS Inter 1st Year Political Science Question Papers and TS Inter 1st Year Political Science Question Paper March 2023 cater to students with diverse academic strengths and challenges.

TS Inter 1st Year Political Science Question Paper March 2023

Time:3 Hours
Max. Marks:100

Section- A
3 x 10 = 30

Note:

  • Answer ANY THREE of the following questions in 40 lines each.
  • Each question carries TEN marks.

Question 1.
Define the State and discuss its Essential features.

Question 2.
Define Political Science and explain its scope.

Question 3.
Define Law. Explain its various sources.

Question 4.
Explain the structure and the functions of the Legislature.

Question 5.
What is Scientific Socialism? Examine its basic principles.

Section – B
8 x 5 = 40

Note:

  • Answer ANY EIGHT of the following questions in 20 lines each.
  • Each question carries FIVE marks.

Question 6.
Explain the difference between State and Government.

Question 7.
Explain any three kinds of equality.

Question 8.
Explain Political Rights.

TS Inter 1st Year Political Science Question Paper March 2023

Question 9.
Explain any four essential conditions for the success of Democracy.

Question 10.
Explain the main features of Presidential form of Government.

Question 11.
Distinguish between Written and Unwritten Constitutions.

Question 12.
How a citizen looses Citizenship?

Question 13.
What are the differences between Nation and Nationality?
Answer:

Question 14.
Explain the concept of Satyagraha.

Question 15.
Discuss Judicial Review.

Question 16.
What are the features of Secular State?

Question 17.
How Political Science is different from economics?

Section – C
15 x 2 = 30

Note:

  • Answer ANY FIFTEEN of the following questions in 5 lines each.
  • Each question carries TWO marks.

Question 18.
Classify Rights.

Question 19.
What is Recall?

Question 20.
How many organs of government are there? What are they?

Question 21.
Father of Political Science

Question 22.
What is a State?

Question 23.
Define Nation.

Question 24.
Define the concept of “Rule of Law.

TS Inter 1st Year Political Science Question Paper March 2023

Question 25.
What is a Written Constitution?

Question 26.
Mention any two qualities of a good citizen.

Question 27.
What is a Theocratic State?

Question 28.
What is power?

Question 29.
What is civil Liberty?

Question 30.
Senate

Question 31.
Behaviouralism

Question 32.
Civil-disobedience

Question 33.
Laissez-faire

Question 34.
Capitalism

Question 35.
What is indirect Democracy?

Question 36.
Natural Rights.

Question 37.
Collective Responsibility.

TS Inter 1st Year Political Science Model Paper Set 8 with Solutions

Thoroughly analyzing TS Inter 1st Year Political Science Model Papers Set 8 with Solutions helps students identify their strengths and weaknesses.

TS Inter 1st Year Political Science Model Paper Set 8 with Solutions

Time: 3 Hours
Max. Marks: 100

Section – A
3 × 10 = 30 Marks

Note: Answer any THREE of the following questions In not exceeding 40 lines each. Each question carries 10 Marks.

Question 1.
Define Democracy and explain its features.
Answer:
Democracy is an important and most significant form of government. The term “Democracy is derived from two Greek words namely Demos’ and “Kratio”. In greek language Demos means the people and Kratio means power. Hence Democracy means power of the people.

Definitions of Democracy:
Abraham Lincoln:“ Democracy is a government of the people, by the people and for the people”.
J.R. Seeley: ‘Democracy is a government in which everyone has a share.”
Lord Bryce: “Democracy is that form of government in which the ruling power of the state is vested not in a particular class but in the members of the community as a whole”.

Features of Democracy:
1. Liberty: Democratic government aims at securing maximum liberty for its citizens. it is built on the foundations of Liberty and Equality. Democracy is the only government which assures liberties to the people by incorporating them in the Constitution.

2. Equality: In the words of Prof. Seeley, democracy is “a government in which everyone has a share”. A democracy 5. ernment gives opportunities to all for making them to take part in political matters. Every citizen has the right to vote and to contest in the election.

3. Independent Judiciary: Independent Judiciary is important principle of Democracy. The higher courts in democratic states act as watch dog of the people’s freedoms. They issue several writs for preserving and upholding fundamental freedoms. They serve as the Custodian of people’s rights.

4. Government of the people: Democracy is government by the representatives of the people and these are elected by the adults, who are free to vote as they please without being coerced or pressurized by anyone. Democracy is the government by ballot not by bullet.

5. Majority Rule: Democracy stands for a rule of the majority with adequate safeguards to the minorities. Every state has political parties. One of the political parties comes to power by capturing the majority of seats in the legislature. This means democracy is a system of government based on the principle of majority rule.

6. Follows constitution principles: A democratic government functions strictly according to the principles of a constitution. Whether written or unwritten, this has been accepted by the people.

7. Against to violence and evolutions: Democracy provides for a change in government according to constitutional principles and it is against any change by violent or revolutionary means.

8. Against to coercive methods: Democracy opposed to coercive’ methods, even if they are for the social good. A government cannot be called democratic, if it uses illegitimate coercion in the name of social welfare.

9. Importance to Human Rights: Democracy upholds the dignity of the human personality and gives various kinds of rights to the individual. Actually, to providing constîtutional rights to the people is the fundamental principle of the democracy. The constitutions of India and U.S.A. had provided several rights to their people.

10. Right to speak: Democracy allows all individuals the right to speak, criticize, and disagree with others constructively.

11. Encouragement to Ideas: Democracy allows plurality of ideas and ideologies and stands flrmly on the principles of tolerance. In the legislature there is plenty of worth full discussions occurred among the public representatives related to public issues.

12. Against Imperialism and War: Democracy in the international sphere stands for the principle of self-determination and for the regulation of interstate relation on the basis of equality, justice, and reason. Democracy is against aggressive nationalism, imperialism, and war. Besides the above, Democracy has some more principles like Rule of law, welfare mechanism, Decentralization of powers, Judicial review etc.

TS Inter 1st Year Political Science Model Paper Set 8 with Solutions

Question 2.
Define the Constitution and explain its features.
Answer:
Introduction: The age of Democracy led to political civilization. Nowadays every civilised state possess a constitution. A Constitution is a condition of modern State. The constitution is a living text of a political system. it represents the political character of the state and its constituents.

The term constitution implies a written document embodying the provisions relating to the powers and functions of the government organs, the rights and duties of the citizens.

Meaning: The term constitution is an English word. It was derived from a Latin word “Constitution, which means to Establish”.
Definitions:

  1. Aristotle: “Constitution is the arrangement of offices in a state, especially the highest of all”.
  2. Lord Bryce: “Constitution is a set of established rules embodying and enacting the practice of Government’.
  3. Stephen Leacock: “Constitution is the form of Government”.
  4. K.C. Wheare: “Constitution is that body of rules which regulates the ends for which governmental power is exercised”.

Features of the Constitution:
1. Preamble: Every Constitution will have a preamble. The preamble denotes the aims and aspirations of the Constitution. It is like the l.of the Constitiution. Hence, preamble is considered as an important feature of the Constitution.

2. Clarity: Clarity is another important feature of the constitution. The Constitution clearly explains about the different policies and methods of governance. it is written In a simple and clear language.

3. Incorporation of Fundamental Rights: Every Constitution includes some fundamental rights. These fundamental rights are meant for safeguarding the freedoms of the citizens. They enable the citizens to realise their personality in various spheres. They help the citizens for leading a happy and honorable life in the state.

4. Brevity: Brevity is another feature of a Constitution. Brevity avoids confusion among the Individuals in understanding and interpreting provisions. Unnecessry elements are not included in the Constitution. It should be precise. It must not contain large number of clauses.

5. Flexibility: The Constitution must be flexible for adapting the wishes are aspirations of the people from time to time. There must be a scope of amending the provisions of the Constitution if necessary Frequent changes in the Constitution tend to weaken the spirit of the Constitution. But, at the same time, the Constitution of a modern state should be adaptable to the progressive changes.

6. Permanence : Permanence is one more feature of the Constitution. The Constitution must have everlasting values for the welfare of the Whole nation. It rerešents the actual structure of the state and its political institutions. It obliges the customs of the people.

7. Mode of Amendment: The Constitution Specifies the mode of amendment. It will be relevant to the contemprary conditions of the state. It contains a special chapter on the constitutional amendment procedures. Usually the constitutional amendments are of three types, namely

  • Rigid
  • Flexible and
  • Half rigid and Half flexible.
  • On the whole, the constitution of every state comprises both rigid and flexible elements.

8. Explanatory: The Constitution is explanatory in nature. It denotes and discusses almost all elements relating to the People, Government, and State. It contains separate provisions on the structure, powers, and limitations of state activity.

Question 3.
Define liberty and describe various types of Liberty.
Answer:
Introduction: The concept of Liberty is of great significance in . the study of political science. Liberty is an essential condition without which man cannot develop his personality It became a source of inspiration to the millions of the people living all over the world.

Meaning: The term liberty is derived from the Latin word ‘LIBER’ which means free from restrictions.
Definitions: Liberty is defined in many ways by different political thinkers. Some of them are cited below:

  1. ‘Liberty means the absence of rêstraints”. – JR. Seeley
  2. “Liberty is the freedom of an individual to express without any external hindrance to his personality”. – G.D.H. Cole
  3. “Liberty means the positive power of doing or enjoying something worth doing or enjoying”. – T.H. Green
  4. “liberty is the easer maintenance of that atmosphere in which men have the opportunity to be at their best selves”. -H.J.Laski.

Types of Liberty: Liberty is of different types. Some of them may be described in the following:

  1. Natural liberty
  2. Civil liberty
  3. Economic liberty
  4. Political liberty
  5. National liberty.

1. Natural liberty: Natural’ liberty is understood as uncontrolled freedom or absolute freedom. It is believed that natural liberty existed in the pre-social and pre-state human life. According to this concept there were no rules and regulations except the laws of nature in the past. All persons were free to do anything according to their will and capacity But this type of liberty is not possible in civilised society, since liberty is a social condition. True liberty does not inconvenience others. Liberty is not a licence. Natural liberty is equal to anarchy. The weak cannot enjoy liberty without law. Some writers imagined that this kind of liberty existed before the origin of state.

2. Civil liberty: Civil liberty is also known as personal liberty. It relates to the individuals freedom in his life as a member of the social organisation. It is eñjoyed by the individuals in the society. Civil liberty is the essential pre-requisite to the existence and survival of human beings. It enables them to lead happy, honourable and civilised life in the state. So it is a must to everyone.

The state recognises the various freedoms of individuals. In this regard Gettle said thus “Civil liberty is the group of rights recognised and implemented by the state’. The constitutions of the America and the India have incorporated civil liberty in the fom of fundamental rights. Civil liberty is manifested in several rights.

These include:

  • Right to Life
  • Right to Work
  • Right to Property
  • Right to Religion.
  • Right to Speech, Expression, Assembly, Movement and Residence etc.

3. Economic liberty: Economic liberty means the right of everyone to earn ls livelihood. Laski described economic liberty as the security and opportunity to find reasonable significance in earning one’s daily bread. Economic liberty ensures everyone freedom from want and feax hunger and starvation. unemployment and insufficiency.

Economic liberty denotes freedom from want or insecurity of economic nature. Without economic liberty democracy is meaningless. It will also be secured by adopting the following measures.

  1. Provision of minimum wages.
  2. Guarantee of the right ‘to work.
  3. Protecting the workers from unemployment, sickness and other types of insecurity.
  4. Providing adequate leisure.
  5. Giving representation to the workers in the management of the industries.

4) Political liberty: Political liberty facilitates the citizens to take part in the political affairs of the state. Laski regarded political liberty as the power to be active in the affairs of the state. Gilchrist considered that political liberty is synonymous with democracy. Leacock described political liberty as constitutional liberty or liberty to choose one’s own government. Political liberty is confined to the citizens alone. It is positive in nature. Political liberty prbvides several rights in political matters.

These include:

  • right to vote
  • right to contest as candidates in electoins
  • right to hold public office
  • right to criticism etc.

Laski emphasised that political liberty becomes real when there prevails:

  • education
  • honest and impartial press etc.

Political liberty will be realized only in a democratic system.

5. National liberty: National liberty implies the freedom and independence of the state. It also denotes the sovereignty of the state. Every state enjoys this liberty and remains fake from the political domination of other states. This liberty is essential for the progress of nation in all spheres. It was manifested in several countries since ancient period. Many countries made efforts for securing this type of liberty The Greeks fought a war of independence against the Turks.

Similarly, the Indian masses under the leadership of Mahatma Gandhi waged a non-violent struggle to secure national freedom from the British rule. National liberty is very valuable. Great leaders like Thomas Jefferson emphasised the significance of national liberty.

TS Inter 1st Year Political Science Model Paper Set 8 with Solutions

Question 4.
Discuss the relationship between Nation and Nationalism.
Answer:
Introduction: The concepts of Nation and Nationalis, have become important components in the domain of International Relations and political science respectively. Both inspired the people of several countries with patriotic feelings prior to the two world wars.

Meaning: The word “Nation” and “Nationalism” are used as complementary to each other.
The word Nation is derived from a latin word “NATIO” which means ‘BORN” (BIRTH) or “Common descent”.
Nation: “Nation is a Nationality which has organised itself into a political body, either independent or desiring to be independent”.

Nationalism: “Nationalism is a state of mind in which the supreme loyalty of the individual is felt to be due to the Nation-state.”
Relationship between “Nation” and “Nationalism”:

  1. Nationalism is a psychological feeling prevailing in the minds of the people.
  2. People Through the feeling of Nationalism sacrifice all their interests for the sake of their Nation.
  3. The strong desire of the people of a nationality to emerge as a nation-state is nationalism.
  4. The seeds of nationalism originated in the cultural renaissance of Europe in 16th century.
  5. In French Revolution (1789) pushed the nationalism further in Europe and took to the great heights.
  6. The Vienna congress (1815) further supported the cause of nationalism in Europe.
  7. The Italian and Gerpian unifications boosted the cause of nationalism.
  8. The American War of Independence (1774) was i great leap forward in spreading nationalism among the people.
  9. The much-publicized theory of Nation’s self-determination of Woodrow Wilson in 1917 was further generated hope among the people of the world to form Nation States.
  10. The two world wars completed to redraft the European map with the formation of Nation-states.
  11. The Freedom struggles and national aspiration of the people of the Asia, Africa and Latin America are fulfilled with the formation of nation-states immediately after the end of II- World war.
  12. The Indian national movement from 1885 to 1947 had fulfilled with the formation of India and Pakistan as an Independent States. In this regard, we can understand that the feeling of nationalism when it acquires unity and independence. It becomes a Sovereign Nation. There were science writers who treated the two terms as synonymous.

Question 5.
Define Political Science and explain its scope.
Answer:
Introduction: Political Science is a premier social science. It is mainly concerned with the study of the state in its relation with Sociefy; Citizens, Associations and the world at large. Aristotle is regarded as the Father of Political Science. He wrote famous book ‘THE POLTICS”.

Meaning: The word politics is derived from the ancient Greek word “POLIS” meaning city. State anðlity from Poletieia’ meaning government of constitution. Politics came to mean the study of state and government and the Institutions of the state.

Definitions: Political Scientists gave various definitions on Political Science. They are as follows:

  1. J.W. GARNER: “Political Science begins and Ends with the State”.
  2. R.G. GETTLE: “Political Science is” The historical investigation of the state in the past, an analytical study of the state of present and what the state ought to be in the future”.
  3. ROBERT DAHL: “Political Science is the Scientific study of importance of power, Authority and influence.”

Scope of Political Science: The Scope of Political Science means the subject matter covered by it or the topics which are included in its study.

It may be explained in the following ways:
i) Study of man in relation to the Society State and Government: Political science is concerned with the perennial and central issue of establishing proper relationships, among state, society and government with individual. Aristotle stated that Man is a social and political animal as well.

ii) Study of State: Political science explains the origin, evolution and purpose of the state and its intimate relationship between the state and the citizens. It explains the various theories of the origin of the state and it also studies the nature, functions and various theories of the state.

iii) Study of the Government: Scope of political science includes the study of government. Political science explains the relationship between state and element. The state realises its aims and objectives through the government. Government formulates various policies, programs, and their implementation for wellbeing of the people. Political science also studies various forms and structures of the government and their merits and demerits.

iv) Study of Associations and Institutions: There are several associations and institutions which influence the life of the individual. Political science studies various associations, institutions and their relationship with the state. Political science explains structure, nature and functions of the various associations and institutions. It also studies voluntary organizations and their role in the political processes.

v) Study of Rights and Duties: Scope of Political Science includes the study or rights and duties of citizens. In recent times, issues relating to civil dghts, human rights and civil society got significance in the study of political science.

vi) Studies of National and International Issues: The scope of Political science comprises the study of national and international affairs. The political science deals with the matters relating to nationstate, territorial integrity and it’s sovereignty. it also studies international aspects like armaments and disarmaments, balance of power, defence. and security studies. It also covers international law, international organisations etc.

vii) Study of Comparative Government and Polities: The importance of the comparative study of government and politics has been increased in recent times. Political science covers the study of various world governments their structures and functions. it studies the relationship among the different political systems in the world.

viii) Study of Modern Political Analysis: The 20th century political science is regarded as a study of sharing and shaping of power, and its execution in a day-to-day political process. Political science studies the modern concepts like, political socialisation, political participation, political development, political culture and political communication.

ix) Study of Public Policies: Modern political scientist like David Easton, Gabriel A. Almond, Charles Merriam argued that ‘political science is a policy science’. They considered political science as the study of formulation, execution and evaluation of public policies. They also emphasised the study of political parties, pressure groups. mass media and organs of the governments and their influence in the formulation of the public policies.

It also studies major polices like Agriculture policy, Industrial policy, Environmental policy, Reservation policy and Education policy etc. Thus, the scope of political science has, in recent times, extended to the study of above concepts and theories and has become one of the most relevant social sciences.

Section – B
8 X 5 = 40 Marks

Note: Answer any EIGHT of the following questions In not exceeding 20 lines each. Each question carries 5 Marks.

Question 1.
Write about the relationship of Political Science with Sociology.
Answer:
The knowledge of Sociology is essential to a political scientist. It is essential to study the sociological roots to understand the nature of the State. The Greeks never conceived any difference between society and State. In the opinion of Greek philosophers, State is not only a political system but also a social system of commendable value.

Social customs help to systematize social set up and social life. The political scientists give importance to the trends of socialisation to understand the behaviour of the citizen as an individual and part of a community of late, political sociology developed as a special science.

This very factor shows how much functional is the impact of social institutions on political life. Political parties, groups and public opinion are subjected to the influence of social factors. The political changes in a State cannot be understood without the knowledge of the social conditions and institutions. For instance, an indepth study of political activity in India needs a study of the social factors like caste, religion, area and language and also processes related to them.

As Political Science discusses the organised groups of people, Sociology studies both organised and unorganised groups. Political Science deals with all the political institutions in the past, present and future. Sociology discusses the rise and growth of all institutions in a society in the past and present.

Question 2.
Explain the difference between State and Government.
Answer:
Differences between State and Government.
Question 3.
Briefly, explain the essential elements of Nationality.
Answer:
Introduction: The concepts of Nation and Nationality have become Important components in the domain of International Relations and political science respectively Both inspired the people of several countries with patriotic feeling prior to the two world wars.

The Events that took place in the erstwhile Soviet Union, ethnic Riots between serbians and croatians in the former Yugoslavia, the unification of East and west Germanies, the peace talks between Israel and Palestine Liberation Organization (PLO) on West Asia etc., reflect the serious concern of the people for realising Nationality and Nation States.

Meaning: The Word “Nation” is derived from a latin word “NATIO” which means “BORN” (BIRTH) or “Common Descert”.

Definitions:.

  1. R.G. Gettle: “Nationality is a population having the common bonds of Race, Language, Religion, Traditions and History.
  2. J.H. Rose: “Nationality is a union of Hearts once made and never unmade”.
  3. J.W Garner: “Natiånality is a group or portion of population which is united by Racial and other bonds”.

Essential Elements of Nationality:
1. Purity of Race: Racial purity helps in the formation and strengthening of the idea of Nationality. Race is a physical phenom erion It depends on certain distinctions of skull, stature, hair; complexion etc. These distinctions serve as a cementing bond among the members of a group.

But we should remember that common race is not an indispensable factor in the growth of Nationality. Modem races are so mixed that none of them can claim to be pure. Pure races have disappeared because of wars and migrations. Racial purity is now a myth only. Ex: Canada and United states have transfonned into single nations inspite of their racial diversities in their respective populations. Similaiiy Australia and Britain are two distinct Nations although they belong to one racial stock.

2. Common Language: Language plays a key role in the promotion of nationality. The philosophers and scientists said that common language is essential for the development of nationality. Language is a medium to express all their feelings. It helps to express one’s own selves to have cordial relations and to share the miseries and happiness in a group languages also promotes common feelings and traditions. Common language promotes the feeling of oneness and keeps the entire race on single track.

3. Common Religion: Religion is one important factor to strengthen nationality. There are many instances when people of different nationalities with common religion remain citizens in the same state. For instance, the main reason for the partition of Indian subcontinent into India and Pakistan in 1947 lies in the religion.

4. Geographical Unity: Geographical unity is necessary for the emergence of nationality. Nationality sentiments prevail and develop among the people living in a single geographical area. The people residing in such an area love, worship their country and make sacrifices for the sake of their motherland. People, who belong to one religion, converse the same language, same race living in a geographical area inculcate and improve their nationality sentiments.

The formation of Israel in 1946 was purely due to the feelings of the hitherto wandering Jewish people to live in a single geographical area. Hence their desire of live in a territory made them united. This ultimately transformed them as patriotic persons.

5. Common History: Common History is considered as an important element of Nationalist It invokes an inspiration among the people and binds them together. Some historical incidents may give a chance to the people to develop national sentiments. Ex: Indians have learnt the lessons of Nationalism from the British legacy.

6. Common Culture: Culture in its broad sense means a way of life. It is reflected through certain common elements like dress, customs, conventions, food habits, religious beliefs, ethical values etc. They easily develop into a single Nation. These elements bind the people together and hold together.

7. Common Political aspirations: Nationality sentiments prevail and develop among the people having common political
aspirations. The political ideas, conventions and institutions which were formed due to the single political rule will have a considerable impact and influence over the people. For instance, the Swiss people love very much their direct democratic devices in political matters.

Similarly, the Americans express the feeling of worship towards their constitution. The British people aise feel proud of their political and judicial institutions like rule of law, parliamentary democracy and judicial review etc.

8. Common Economicties: This element of nationality has been stressed by Karl Marx. Since then onwards the importance of this element has been increasing. The Russians have great regard for their economic system, even though there exist diversities. Their unflinching love for socialism inspired nationalism among them. They successfully repulsed the attacks of Germany during the Second World War. Thus the common economic ties made them united and inte grated them into a nation.

TS Inter 1st Year Political Science Model Paper Set 8 with Solutions

Question 4.
Discuss any four kinds of Law.
Answer:
1. Natural Law: Natural law is also known as divine law. It is abstract. It is not created by any human agency It is considered as the gift of nature, based on metaphysical power. Itrçfers to the use of reason to analyse human nature. It is written in the heart of human beings by the finger of God.

2. Positive Law: Positive law is created by the human agency It is also known as political law. It is framed on the basis of the existing social and political conditions. It is sanctioned by the Sovereign Political Authority. Violation of positive law leads to punishment.

3. Constitutional Law: It is a basic law of any state. It defines the political system. All the basic principles of administration are included in this type. All other Jaws in the State are subservient to constitutional law. It is framed by the constituent assembly.

4. Ordinary Law: It determines the relation between the State Administration and people. These laws are framed by a group of officials authorised by law.

Question 5.
What do you mean by Civil Liberty?
Answer:
Civil liberty is also known as personal liberty. It relates to the individual’s freedom in his life as a member of the social organisation. It is enjoyed by the individuals in the society Civil liberty is the essential pre-requisite to the existence and survival of human beings. It enables them to lead happy, honourable and civilised life in the state. So it is a must to every one. The state recognises the various freedoms of individuals. In this regard Gettle said thus “Civil liberty is the group of rights recognised and implemented by the states.

The constitutions of the America and the India have in corporated civil liberty in the form of fundamental rights. Civil liberty is manifested in several rights.

These include:

  • Right to Life
  • Right to Work
  • Right to Property
  • Right to Religion
  • Right to Speech, Expression, Assembly, Movement and Residence etc.

Question 6.
Gandhiji’s views on Religion and Politics.
Answer:
The moral concepts of Gandhi can be founded in his ideas expressed very frequently. Politics without morals character create a degenerated state and Government in a human society indulged in material pleasures. All the political means are used only to ratify power. But Gandhi described the theory of authority. He says that both authority and ethics should become the focal points of politics.

He mixed humanistic, political religious and ethical values in politics. He opposed the segregation of religion from politics. In his opinion both of there are equally essential. Politics without religion is like a dead corpse, not useful to a country except to burn. He says that his search for truth pulled him into politics and his moral strength helped him to remain very firm in politics.

Even a political programme is intended for the social and ethical advancement of the people. We cannot segregate politics from life, most importantly from religion. Segregation means nurturing religious fundamentalism, bigotry and evil in politics. Religion according Gandhi was not Rituels and blind faith, but a co-ordinated moral values of all religious. A religion should not promote sectarianism. But Gandhi never supported a state religion. He wants to use the religion to oppose all types of evils in the society.

Question 7.
Explain Political Rights.
Answer:
Political rights are those rights which enable the individuals to participate in the political affairs of the state.
The following are the important political rights:
1. Right to vote: Right to vote is the most important political right enjoyed by the citizens in modern democratic states. It serves as a powerful weapon for adult citizens in choosing their representatives to various legislative bodies. It makes them as real sovereign. All the citizens are entitled to this right without any discrimination based on creed, colour, language, race, region, religion, sex etc. However, persons such as aliens and minors are deprived of this right.

2. Right to contest in elections: This ‘right empowers the citizens to contest as candidates to various legislative bodies in the state. Especially this right enables those, whó have political sagacity, enthusiasm and dynamic nature, to actively participate in the political dynamics of the state. As a result, it increases political enthusiasm among the citizens. Such an element is considered as a base of democratic polity.

3. Right to hold public offices: This right provides opportunities to the citizens to hold various public offices for a definite period. It gives no scope for exclusion of citizens or conferring special privileges to some at the cost of others. This helps the citizens to exercise authority in a dignified manner.

4. Right to petition: This right enables the citizens to forward petitions denoting their requijçmentS or grievançes. It is considered as a vital political right in the modem state. The citizens could be able to find solutions to their immediate or long pending issues by bringing them to the notice of the government through this right. It also helps the public authorities to know the grievances of the people and attend, to them properly and promptly.

5. Right to criticism: This right gives opportunity to the citizens to criticize the various public policies and programmes. It also enables them to highlight the omissions and commissions of the leaders, and administrative personnel at various Levels, it also gives scope for the citizens to render positive and constructive criticism about the on goings in the government from time to time. Ultimately it keeps the administrative authorities and policymakers tobe vigilant in discharging their obligations.

Question 8.
How citizenship is lost?
Answer:
Citizens loose their dtizenship under the following conditions:
1. Renunciation: A person is deprived of,his citizenship, if he wishes to become the citizen of another state. One will lose the citizenship of orteas parent state and may become the citizen of a foreign state by naturalization. In India, the Constitution prescribes that a person who voluntarily acquires citizenship of any other state will no longer be an Indian citizen.

2. Marriage: Generally a woman lose her citizenship when she marries an alien. However some states allow retention of citizenship. For instance in Britain, there is an option to retain British citizenship who marnes an alien.

3. Accepting Foreign Service: A person may lose his citizenship when he enters into the service of another state. If a person accepts a permanent job in the government of a foreign state, he foregoes the citizenship of his native state.

4. Obliging Foreign Decorations or Titles: When a citizen obliges to receive foreign decoratiotis or titles, it may lead to the forfeiture of his Citizenship.

5. Prolonged Absence: Prolonged absence in the native state beyond a certain period may lead to the loss of citizenship. In some states like France and Germany citizens who are absent themselves from their native country for more than ten years will loose their citizenship.

6. ‘Treason or Crime: Involvement of a citizen in a serious crime and subsequent proof of his action will also lead to the loss of citizenshi Especially those persons who directly or indirectly participate or extend assistance to anti-state, anti-social and anti-governmental activities, will loose their citizenship by a special notification to that effect.

7. Desertion from Army: Desertion from army thereby jeo pardizing the security of a state leads to the forfciture of citizenship.

Question 9.
Discuss the essential conditions for the success of Democracy.
Answer:
The following conditions are essential for the success of democracy.
1. Sound system of Education: The success of democracy requires adequate education for the citizens. ignorance innocence and undereducation prevent them from adopting right attitudes and large-scale reforms. Education sharpens the intellect of individuals. It develops a proper understanding of various things. It makes the citizens vigilant. Besides, this enables them to assess and criticise the policies of government.

2. Enlightened Citizenship: Enlightened citizens are an asset to the democratic state. They can exercise proper vigilance. They can actively participate in public affairs and help their fellow citizens in the exercise of their rights and discharge of their reproducibilities. They extend co-operation to the government in all its good work.

3. Independent Press: An independent press is a prerequisite of democracy. it enables the people to receive accurate and unbiased information regarding the activities of the government. it not only keeps the people in touch with government activities but also ventilates their grievances. It strives to promote harmonious relationship between the people and the government.

4. Strong Opposition: The success of parliamentary democracy depends to a great extent on the strong and effective opposition. Such an opposition will act as a check against the government by pointing out its lapses. In this regard, the role of opposition in some advanced states.

5. Decentialization of powers: Decentralization of powers and establishment of democratic institutions at the grass roots level is indispensable for the healthy organization of democratic institutions. The representative bodies at the grassroots level as known as Panchayat Raj Institutions (PRIs) in India) will ct as the mini legislatures. The residents of local areas will be able to know how to exercise their franchise.

6. Absence of economic disparities: Democracy can not function smoothly when there are economic disparities in a country. When a country comprises a large number of poor people and a few wealthy persons, democracy could not work successfully.

7. Social Equality: Social equality is another prerequisite of democracy. Caste, class and racial differences will impede the healthy working of democracy Such elements encourage of democratic polity To be successful, democracy must open its doors to everybody on equal basis by providing equal social opportunities to all in social sphere. Social equality must not only be proclaimed but also be practiced.

8. Faith in democracy: Certain democratic beliefs and values like individual’s worth, need for tolerance of differences, decisions through discussions etc., should be inculcated among the people.

9. Sagacious Leadership: Sagacious leadership is another essential condition of democracy Sagacious leaders, by didn’t of their administrative acumen, political propriety, social commitment and economic perspective, will be able to lead the democratic state to greater heights of glory.

10. Honesty and transparency: Honest persons belonging to various walks life, when entrusted with major responsilites of the government; will strive for the success of democracy Similarly transparency in administration also acts as a basic ingredient for the success of democracy.

11. Absence of militarism: Democracy functions mostly in countries which are relatively free from militarism. In rules out the use of force and believes in the worth of individuals. It provides adequate opportunities to the people basing on worth, ability and dedication militarism, on the other hand, demands concentration of authority and favours despotism.

TS Inter 1st Year Political Science Model Paper Set 8 with Solutions

Question 10.
Distinguish between Written and Unwritten Constitutions.
Answer:

Written Constitution Unwritten constitution
1. Written constitution implies a document or few documents in which the rules regulating the main institutions of Government are written down. 1. Unwritten constitution denotes a sum of customs, conventions, and usages which have not been systematically documented.
2. All the basic principles of the State are clearly written. 2. All the basic principles of the State exist in the form of customs and traditions.
3. A written constitution is framed by a special assembly convened at a particular point of time. 3. Unwritten constitution contains some written elements also in the form of enactments of fundamental charters made from time to time.
4. It is suitable to the educated and literate people. 4. It is suitable to the uneducated and illiterate people.
5. Courts of law protect the Liberties of the citizens. 5. Courts of law cannot provide much protection.
6. It is formulated at a particular time. 6. It is evolutionary in nature.
7. It provides political stability. 7. It could not ensure political stability.
8. It cannot be easily amended. 8. It can easily be amended.
9. It is useful to federal states. 9. It is advantageous to the unitary states.

Question 11.
Write about on Rule of Law.
Answer:
Rule of law is a unique feature of British constitution. It at first originated in England later many states like India and the United States have adopted this feature. Rule of law is purely based on the principle of common law. It denotes that law should be general in form. It should be uniformly applicable to all the citizens living in a state. There should not be different types of law for various sections of the community.

In other words the government must treat all the citizens equally as per the law. The governance and administration in a particular state should be carried on in accordance with the laws. A.V. Dicey in his “Law of the constitutions” (1885) gave a precise explanation of the rule of law.

According to him, rule of law stands for equality before the law. This implies equal subjection of all classes including the officials or common man. Law makes no discrimination between individuals. A.V. Dicey says that “Every official from Prime Minister to the attender, all are equal before the law”.

Therefore, Rule of law is helpful to all the citizens in protecting their responsibilities in a more effective manner. The constitution of India too recognizes the Rule of law as a basic feature of India constitution. The Supreme Court of India time and again declared Rule of law as a basic structure of Indian Constitution. Articles 14 to 21 of the Indian Constitution have incorporated the spirit of this concept. However, the scope of this concept is gradually shrinking owing to the overburdening of legislative work with enormous functions.

On the whole, the cardinal virtue of Rule of law is that “All are equal before law and no one must arbitrarily be punished”
constitutes the core value of any democratic system in the world.

Question 12.
What is Parliamentary form of Government?
Answer:
In a parliamentary system a clear distinction is made between the head of the state and the head of the Government here, the head of the state King or Queen in Britain or President of India, possesses nominal or titular authority whereas real authority rests with the government of which Prime Minister is the head. Example : Australia, Canada, Japan etc.

Parliamentary form of government as a system in which the real executive, the cabinet, is Immediately and legally responsible to the Legislature for its political policies änd acts, ultimately responsible to the elechifate.

Features of Parliamentary Governments:
a) Nominal and Real executives: In parliamentary form of government there should be two kinds of prexcutive in the Political system one of them, National Executive is the Head of the state and other one is real executive, Head of the government is president and Head of the government is the Prime Ministers.

b) Coordination between the Legislature and executive: Another important feature among, is that, there is a close relationship and co-ordination between legislature and executive bodies of the government. The executive members are selected from the Legislature and so executive remained as responsible for the Legislature for all its acts.

e) Significant role of the prime minister: In parliamentary form of government, the prime minister holds the real executive authority. He holdsthe government as cornerstone. He has the authority to from the council of ministers and also has right to reshuffle and dissolve the government.

d) Collective responsibility: The most important feature of the parliamentary government works on the principle of collective responsibility. It means the ministers enjoy the office only as long as they have confidence of the parliament.

e) Individual responsibility: In a parliamentary government, every minister is individually responsible to the Legislature for the efficient conduct of his department or office. In case there is any lapse in the administration, the ministers are personal answerable to parliament.

f) Dissolution of Lower House: The head of the state can dissolve the lower house on the recommendation of the prime minister. If deadlock rises between cabinet and Legislature they can appeal to the electorate through elections.

g) Effective opposition: In a parliamentary form of government opposition party is considered as soul of the democracy. If the ruling party loses its confidence in the Legislature, opposition party is the alternative to form a government and it works against ruling party through questioning the acts.

Section – C
15 × 2 = 30 Marks

Note: Answer any FIFTEEN of the following questions in not exceeding 5 lines each. Each question carries 2 Marks.

Question 1.
Unitary form of Government.
Answer:
The word ‘Unitary’ consists of two words, namely, ‘Uni’ and ‘Tary’, urn means one and tary means ‘rule’. A unitary Government is a single integrated government with all executive powers. The Constitution vests all powers in the Central Government.

Question 2.
Separation of Powers.
Answer:
Theory of separation of powers is propounded by Montesquieu in his famous book The Spirit of Laws. The powers among the three organs of the Government in the presidential executive will be distributed on the basis of the theory of separation of powers. Its main feature is Checks and Balance, which means the three organs of the Government possess equal powers and each organ checks the other two organs from crossing their limits.

Question 3.
Senate.
Answer:
Senate is the Upper House of United States Congress, which is a small group of elected people who decide the laws of the country. It consists of loo members. Every U.S. state elects two people to represent them in the U.S. Senate. These people are called senators.

TS Inter 1st Year Political Science Model Paper Set 8 with Solutions

Question 4.
Write about Flexible Constitutions.
Answer:
A flexible constitution is one whose provisions can be amended easily. It requires no special procedure for changing its provisions. It can be amended by the authorities by adopting the same procedure of ordinary laws. So we do not find differences between ordinary and constitutional laws. Flexible constitutions were prevalent in the ancient period. Ex.: British Constitution.

Question 5.
Define Secular State.
Answer:
D.E. Smith defined Secular State as None while guaranteeing individual and corporate freedom of religion, which deals with the individual as a citizen irrespective of his religion.

Question 6.
What is Recall?
Answer:
Recall means To call back. The representatives will be called back by the people in case they are inefficient. Hence, this method helps the representatives in discharging their responsibilities properly for fear of being called back on the grounds of inefficiency.

Question 7.
What is dual citizenship?
Answer:
Dual Citizenship means possession of two citizenships in two States. Ex: Children born to American citizens in other States acquire citizenship in both the States – one in their parent State and the other in the State, where there are born. Dual citizenship applies to the children until they attain adulthood. Later they have to choose citizenship of any one of the two states.

Question 8.
What are Civil Rights?
Answer:
Civil rights aim at providing basic conditions for individuals to lead a happy and dignified social life. These rights are considered vital for a civilized society Civil rights are described as the gift of democracy. Democracy flourishes wheh the citizens are provided these rights. Civil rights are those rights which provide opportunity to each person to lead a civilized social life. These fulfil basic needs of human Life in society Right to Life, liberty and equality are civil rights. Civil rights are protected by the state.

Question 9.
What are the objectives of Human Rights?
Answer:
The following are the various objectives of human rights.

  1. Provision of independence to the People against discrimination.
  2. Freedom from poverty.
  3. Freedom for availing the latent abilities of individuals.
  4. Freedom from feat
  5. Freedom of protection.
  6. Freedom from injustice.
  7. Freedom of speech and expression.
  8. Freedom of protection.
  9. Freedom of association.
  10. Freedom for carrying one’s activities on dignified lines.
  11. Freedom against exploitation.

Question 10.
J.S. Mill.
Answer:
John Stuart Mill was the most prominent political thinker of 19th century He was the most influential intellectual who propagated the Ideas of Liberty, Equality, and Welfare State. He paved the way form the emergence and spread of individualism against the hitherto imperialist and despotic tendencies in European politics of Nineteenth century. He played a decisive role in propounding the theory of Individualism in Western political thought. He tried to disposal the misconception among the masses in regard to the validity and relevance of Benthams’ utilitarianism.

Question 11.
Meaning of Socialism.
Answer:
Hughan regarded socialism as the political movement of the working class which aims to abolish exploitation by means of collective ownership and democratic management of the instruments of production and distribution. Some writers regarded socialism as a democratic movement meant for promoting justice and liberty and for managing the society on efficient principles.

Question 12.
What are the formal organs of power in the state?’
Answer:
These are three formal organs of power in the state.
They are:

  • Legislature
  • Executive and
  • Judiciary.

Legislature prepares, Amends Laws and Repeals old laws. Executive implements the different laws prepared by the legislature and maintains law and order in the state. Judiciary interprets and analyses the prevailing laws and provides justice. The will of the state is formulated, expressed and realised through these formal organs.

Question 13.
What are the views of John Rawl’s on Social Justice?
Answer:
John Rawl’s Admitted that:

  • Social Justice implies equal access to the liberties, rights and opportunities to the deprived sections of the society.
  • Social Justice is built around the idea of a social contract committed by the people for obeying certain rules.

Question 14.
What is equality of opportunity?
Answer:
The idea of equality refers to the equality of rights and opportunities.

  1. Harold Laski, in his book, ‘A Grammar of Politics’, mentioned that Equali4y implies fundamentally a leveling process.
  2. According to Barker, equality implies “Equal rights for all the people and abolition of all special rights and privileges’.

Question 15.
What do you know about National Liberty’
Answer:
National Liberty implies the Freedom and Independence of the state. It also denotes the sovereignty of the state. Every state enjoys this liberty and remains free from the political domination of other states. Ex: The Indian masses under the leadership of Mahatma Gandhi waged a non-violent struggle to secure National Freedom from the British Rule.

Question 16.
‘Rule of Law in the constitution of India.
Answer:
Rule of law is purely based on the principle of general law. It denotes that law should be general in form. It should be uniformly applicable to all the citizens living in a state. There should not be different types of law for various sections of the community.

Question 17.
What is meant by Nationality’.
Answer:
Nationality is derived from the Latin word Natio”, which means birth It is a spiritual sentiment or feeling of oñeness. This emotional feeling of oneness or unity is caused due to factors like common race, common language, common religion, common territory, common history and culture, common political aspirations, etc. Nationality is not politically organised. There can be no nation without, nationality.

TS Inter 1st Year Political Science Model Paper Set 8 with Solutions

Question 18.
‘De-Jure’ Sovereignty
Answer:
The term ‘De-Jure denotes authority exercised according to law. Dejure sovereign is the power possessed and exercised by a legally competent authority. It issues orders and enjoys command over all persons, institutions and organisations. In the state. The Queen in Britain and the president in India are example of the De-Jure sovereignty.

Question 19.
Mention any four points of significance of Political Science.
Answer:

  1. Political Science explains concepts and theories.
  2. Political Science enlightens on Rights and Duties.
  3. Political Science provides knowledge of the political thinkers.
  4. Political Science deals with International Relations.

Question 20.
Master Science.
Answer:
Aristotle used the term ‘Politics’ to designate the science of the state. He called ‘Politics’ a “Master science” as it covered almost all the activities of Individuals in Society that includes Political and Social Institutions.

TS Inter 1st Year English Model Paper Set 10 with Solutions

Thoroughly reviewing TS Inter 1st Year English Model Papers Set 10 helps in understanding the examiner’s expectations.

TS Inter 1st Year English Model Paper Set 10 with Solutions

Time: 3 Hours 15 Minutes
Maximum Marks: 100

Section – A

Question 1.
Annotate ANY TWO of the following in about 100 words. 2 × 4 = 8M
a) Though the trees grown by her are worth several crores of rupees today, her life has no respite from poverty.
b) In thought, in talk, in action, I think you will find that you can separate life into these two divisions-the dark side and the bright side, the discouraging side and the encouraging side.
c) A voice shouting ‘Relax’ penetrated into me above the noise of the crowd.
Answer:
a) Introduction : These touching words are extracted from the inter-net-based inspired write-up, The Green Champion – Thimmakka. It shows the magnificent achievement of an ordinary woman with an extraordinary commitment to conserve nature.

Context & Explanation : Born into a poor family. Thimmakka did not go to school. She worked as a labourer. As she grew up, she was married to Chikkayya, a labourer. When they came to know that they could not beget children, they were not disappointed. They came up with the idea of planting saplings and nurturing them as their own children. It became their life mission. But, she suffers from poverty. Her sole source of income is the pension of Rs. 500/- given by the government Thus, she is an example of simple living and high thinking.

Critical Comment : These words describe her pains and problems.

b) Introduction : This sentence is taken from the prose piece, “Two Sides of Life”, penned by Booker T Washington. This is a speech extracted from his popular book, Character Building.

Context & Explanation : While analyzing a number of divisions in human life, he finds two most important ones which are significant. They are the dark side of life and the bright side of life or the discouraging side and the encouraging side. These two are found in thought, in talk and in action of a person.

Critical Comment : Here, Booker T Washington discusses two sides – the bright, the dark of life though many more divisions are possible.

c) Introduction : This sentence is taken from Roger Bannister’s inspirational essay The First Four Minutes’. It is his personal experience.

Context & Explanation : Bannister looked at the flag as he lined up for the start. The flag swayed gently. The race started. He understood that he was going very slow. He himself shouted ‘Faster’. His worry -increased when he heard the first lap time 57.5 seconds. In that excitement his knowledge of pace had deserted him. At one and a half laps he was still worrying about the pace. Then a voice shouting “Relax” penetrated into him above the noise of the crowd. He followed it and started relaxing. There was no pain and stress. Later, he came to know that it was his coach ‘Stampfl’s advice.

Critical Comment : Dr. Roger Bannister narrates his glorious moments and second to second experience while running for the goal of one mile race.

TS Inter 1st Year English Model Paper Set 10 with Solutions

Question 2.
Annotate ANY TWO of the following in about 100 words. 2 × 4 = 8M
a) It was the plant and flower of light.
b) They all shook their heads and gave me a smile as though. I was trying to fool with them.
c) His stomach is full of infinite void.
Answer:
a) Introduction : This line is taken from the poem, The Noble Nature penned by Ben Jonson. He is regarded as one of the major dramatists and poets of the seventeenth century.

Context & Explanation : The poem says leading a meaningful life even for a short while is worthier than leading a long life with neither charm nor value. The core meaning of the poem centres round this single idea. The lily plant has a short life. It blooms in May and is very beautiful. Although the flower has the life span of a day and falls and dies by nightfall, it spreads beauty and delight in that short period. The poet feels, that a meaningful life like lily flower though short is what makes a man noble. Even though a man’s life is short, it can be a perfect life.

Critical Comment : The poet advises one to lead a meaningful life-of light-like that of a lily.

b) Introduction : These lines are extracted from the poem, ‘Happiness’ written by Carl Sandburg, a famous American poet. This poem is from his collection of poems, Chicago Songs. It is a simple poem with a valuable message.

Context & Explanation : The narrator seeks to know what happiness is. He enquires with many professors but in vain. Even the top executives are consulted to help him in this regard. But they express their inability. The professors and the executives smile to him as a reply of the question asked by the narrator. They look at him as if he is trying to fool them.

Critical Comment : The narrator asks professors and top executives to tell the meaning of happiness, but in vain.

c) Introduction : This heart touching line is taken from the thought provoking poem, ‘The Beggar’, penned by Dr. Ammangi Venugopal, a famous Telugu poet. His original Telugu poem, Bichchagadu is rendered into English by Elanaaga (Dr. Surendra).

Context & Explanation : The poet tries to draw the attention of readers to the gravity of farmers’ problems. It is because farmers work hard. They help others. They are the food providers to all yet the irony is that they struggle to survive. They starve. They don’t find food for themselves, even a morsel! Their stomachs get no food. They suffer from empty stomachs. Their emptiness is infinite. Thus, the poet highlights farmers’ woes and worries in a touching way. He also compels the readers to ponder over possible solutions.

Critical Comment : Here the poet depicts the pathetic condition of farmers in a touching way.

Question 3.
Answer ANY TWO ofthe following questions in about 100 words each. 2 × 4 = 8M
a) What is the attitude of teachers towards learners as illustrated in Father, Dear Father ?
b) Box and Cox fulfills all the characteristics of a one act play. Explain.
c) Why does the speaker feel it unfortunate about the students who fail in assessing property the personality of their teachers ?
Answer:
a) Raj Kinger’s Father, Dear Father is a thought provoking commentary on the present education system. It highlights the defects in the mind sets of parents, learners, teachers and the government bodies. It sets all to a new wave of thinking. However, the attitude of teachers towards learners are rude and adamant. When Rahul seeks advice of his Biology teacher to save his rose plant, she gets irritated. She thinks it a question out of their syllabus and asks him to approach a gardener for advice. Her response to Rahul reveals her crossness, irritability and rudeness. She serves as a warning to all those teachers who do not show any interest or reverence towards their profession. The letter also illustrates Rahul’s experience with his English teacher who was adamant.

b) Box and Cox by John Maddison Morton is a comic one-act play. It is regarded as the best farce of the 19th century. It was translated into many European languages. It is a humorous drama. It is short. It fulfills all the characteristics of a true one-act play. It has just three characters. It follows the unity of place. That is the action takes place in one location.

It observes the unity of time too. Its action does not last for a long. It has humour in abundance. It also serves a social purpose by exposing certain follies we suffer from. The play, Box and Cox, thus proves itself to be a perfect comic one-act play. It has all the characteristics of a one-act play.

c) Booker T Washington is a world-famous Afro-American writer, activist and educator. His collection of speeches comes in the form of Character Building. A selection from that celebrated book is our present lesson. It discusses the two sides of life. It recommends the positive side. Washington lists some mistakes students make about their teachers. He makes it clear that every teacher makes mistakes at times. It is human to err. He advises students to ignore such mistakes. He asks them to see the positive aspects of the lesson as well as the teacher. He emphasises the need to see the good and forget the bad. Good advice indeed !

TS Inter 1st Year English Model Paper Set 10 with Solutions

Question 4.
Answer ANY TWO of the following questions in about 100 words each. 2 × 4 = 8M
a) Keep Going is a classic inspirational poem, claim many critics. Substantiate.
b) What does the speaker promise in A Red Red Rose ?
c) Explain with example of the Lily that size matters not but beauty counts a lot.
Answer:
a) Edgar Albert Guest is very popular as a people’s poet. This poem, Keep Going, is undoubtedly an inspirational poem of valuable advice. Through out the poem the poet advises the reader never to quit. With the help of convincing images and commendable comparisons, the poem encourages the reader to keep on the struggle till the goal is attained. It openly admits that life may be a mixture of more pains and less pleasures. Yet, one must continue with one’s effort though with intervals of rest, till success greets one with smiles the poem emphatically announces that the faint see goals afar and the weak give up in the middle. It asserts that winnersiiever quit. It is no surprise that the world welcomes it as a classic inspirational poem.

b) The poem, A Red Red Rose, is written by Robert Burns. It is one of the best lyrics of English poetry. The speaker shares his romantic lone for his beloved. He promises different things to his beloned. He vones to love his beloved until the seas have dried up, the fire of the sun has melted the ice, and human life is over. He uses these examples to express his feelings. Thus, promises his eternal love to his bomy lass and that no matter how far he might go, he will always return to her side.

c) Ben Jonson, in the poem, The Noble Nature talks about what makes a man noble. He compares man to a sturdy Oak and to a delicate Lily in order to explain this point. The Lily plant has a short life. It blooms in May and is very beautiful. Although, the flower has the span of a day and dies by nightfall it spreads beauty and delight in that short period the poet feels that a meaningful life like the Lily flower, though short, is what makes a man noble and even though a man’s life is short it can be perfect life. People will continue to talk good about him even after he is gone. This is what actually makes a man noble, thus, beauty counts a lot.

Question 5.
Answer ANY TWO of the following questions in about 100 words each. 2 × 4 = 8M
a) Is the title, “The Short-sighted Brothers” apt to the story ? Explain.
b) The young man feels that the five boons are not gifts, but merely lendings. Justify his opinion with reference to his experiences.
c) “With all these atrocities, we cannot live”, cries a woman of Ramasagaram. Explain the atrocities the villagers were subjected to.
Answer:
a) Yes. The name “The Short-sighted Brothers” and the content of the story look like made-for-each-other things. The title matches perfectly well with the theme. Three brothers are the lead characters. All the three brothers were very short-sighted. It was a physical disability. And they all suffered from the related mental disability too. They failed to see the possible result of their crooked plans.

They were selfish. They were greedy. They tried to cheat one another. Finally their follies were exposed. As the entire story deals with their physical and mental short-sightedness, the title is appropriate to this ancient Chinese folk tale.

b) Mark Twain is the pen name of Samuel Langhorne Clemens. The world celebrates him as an eminent humourist and a great writer. The present story, “The Five Boons of Life” present his pessimistic view of life. In the story, a fairy offers a young man a boon. The fairy asks him to choose from “Fame, Love, Riches, Pleasures and Death”. She warns him that only one of those five boons is really precious. The youth chooses ‘pleasures’ first. He very soon realises that each pleasure is followed by pain. Next he chooses ‘Love’ that ends him in grief. ‘Fame’ leads him into ‘envy and pity’. ‘Wealth’ throws him into poverty. So, he rightly feels that they are not boons but mere lendings.

c) “Sanghala Panthulu”, an insightful Telugu story by Suravaram Pratapa Reddy, portrays the plight of the innocent poor. English rendering of the story by Elanaaga (Dr. N. Surendra) captures its spirit well. The story lists the atrocities Ramasagaram villagers were subjected to by the police. They (atrocities) were innumerable and unjust. They (villagers) were forced to supply to the police fowls, eggs, groceries, todday and nuts and fruits. The police demanded drudgery. The poor had to clean their toilets, press their legs, get for them firewood, etc. The wages the poor got for their services or supplies were beatings and scoldings. They were branded on the cheeks. An old, tired and hungry woman was beaten to death. The list is endless.

TS Inter 1st Year English Model Paper Set 10 with Solutions

Section – B

Question 6.
Read the following passage carefully and answer ANY FOUR questions given after it in a word or a sentence each. 4 × 1 = 4M
The country is India. A colonial official and his wife are giviqg a large dinner party. They are seated with their guests-army officers and government attaches and their wives, and a visiting American naturalist-in their spacious dining room, which has a bare marble floor, open rafters and wide glass doors opening onto a veranda.

Questions:
i) In which country is this story set ?
ii) Who is the host of the party ?
iii) Where is the party arranged ?
iv) What is synonym for the word porch in the passage ?
v) Who is the special guest in the party ?
vi) Describe the place where the dinner is hosted.
Answer:
i) in India
ii) a colonial official and his wife
iii) in the spacious dining room of the hosts
iv) veranda
v) a visiting American naturalist
vi) a spacious dining room with a bare marble floor, open rafters and wide glass doors opening onto a veranda

Question 7.
Read the following passage carefully and answer ANY FOUR questions given after it in a word or a sentence each. 4 × 1 = 4M
Here are two unique novels – Pharaoh and the King and The Victorian-that made two heroes fans of their writer. The heroes are Chiranjeevi and Hrithik Roshan. And their writer is Dasari Venkata Vishwanath. But the reason ? Their writer is a visually challenged, Telugu Medium student who started writing his first English novel when he was just 11 (2004). Nystagmus or photophobia is the disease that disables him from focusing his sight on any object for more than a second.

Born into a farmer’s family of Gokavaram in East Godavari district, the boy was inspired by JK Rowling. Helped by his English teacher Buchibapayya and younger sister Sija, Verikat worked hard for seven years to complete his first novel in 2011, which was launched by Chiranjeevi. After that, another five years of dedicated work and the second masterpiece was out in 2016. Venkat dedicated it to his favourite hero Hrithik Roshan. That made the hero Venkat’s fan and he invited Venkat to his birthday party as a special guest. Thus, he won the hearts of two heroes, besides those of millions of readers the world over.

Questions:
i) What is unique about the two novels, according to the passage ?
ii) Who are the heroes who turned fans of that writer ?
iii) How does the disease, Nystagmus impact Venkata Vishwanath ?
iv) Name the three persons who helped, directly or indirectly, Venkata Vishwanath in writing his first novel ?
v) How long did Venkata Vishwanath take to write those two novels ?
vi) What made Hrithik Roshan a fan of Venkata Vishwanath ?
Answer:
i) that made two heroes fans of their writer
ii) Chiranjeevi and Hrithik Roshan
iii) disables him from focusing his sight on any object for more than a second.
iv) JK Rowling, his English teacher Buchibapayya and his younger sister Sija
v) twelve (7 + 5) years
vi) Venkat dedicating his second novel, The Victorian, to Hrithik Roshan

Section – C

[Note : Answers of this section must be written at one place in the same Serial Order.]

Question 8.
Match ANY EIGHT of the following words in Column – A with their meanings in Column – B. 8 × 1/2 = 4M

Column A Column B
i) resistance a) silence
ii) pounce b) suffering, agony
iii) flutter c) surround, cover
iv) anguish d) natural, on the spot
v) haven e) dared
vi) spontaneous f) refusal to obey; opposition
vii) lull g) confuse, puzzle
viii) ventured h) move lightly and quickly, tremble
ix) engulf i) safe place
x) bewilder j) jump, leap

Answer:

Column A Column B
i) resistance f) refusal to obey; opposition
ii) pounce j) jump, leap
iii) flutter h) move lightly and quickly, tremble
iv) anguish b) suffering, agony
v) haven i) safe place
vi) spontaneous d) natural, on the spot
vii) lull a) silence
viii) ventured e) dared
ix) engulf c) surround, cover
x) bewilder g) confuse, puzzle

TS Inter 1st Year English Model Paper Set 10 with Solutions

Question 9.
Identify the parts of speech of ANY EIGHT of the following underlined words. 8 × 1/2 = 4M
It is a very (1) bad habit (2) to get into (3), that of being continually (4) moody (5) and discouraged, and (6) of (7) making the atmosphere (8) uncomfortable for everybody who comes (9) within ten (10) feet of you.
Answer:
1) very – adverb
2) habit – noun
3) into – preposition
4) continually – adverb
5) moody – adjective
6) and – conjunction
7) of – preposition
8) atmosphere – noun
9) comes – verb
10) ten – adjective (numeral)

Question 10.
Fill ANY EIGHT of the following blanks with a, an or the. 8 × 1/2 = 4M
There are quite_____(1)_____number of divisions into which life can be divided, but for_____(2)_____purposes of this evening I am going to speak of two;_____(3)_____bright side of life and_____(4)_____dark side.

You will not accomplish_____(5)_____task which we expect of you go with_____(6)_____moody, discouraged, fault-finding disposition.
Yes, my first rank slipped to ……………… (7) second.
From his talk, it seems studies were_____(8)_____ancillary subject; and living and experiencing,_____(9)_____major subject. Father, is he fibbing? Or is it possible that_____(10)_____world turned topsy-turvy in just about 70 years ?
Answer:
1) a
2) the
3) the
4) the
5) the
6) a
7) the
8) an
9) the
10) the

Question 11.
Fill in ANY EIGHT of the following blanks with suitable prepositions. 8 × 1/2 = 4M
I want you to go out_____(1)_____this institution so trained and so developed that you will be constantly looking_____(2)_____the bright, encouraging and beautiful things _____(3)_____life.

Everything that comes_____(4)_____their mouths is unpleasant, _____(5)_____this thing and that thing, and they make the whole atmosphere _____(6)_____them unpleasant_____(7)_____themselves and _____(8)_____everybody_____(9)_____whom they come_____(10)_____contact.
Answer:
1) from
2) for
3) in
4) from
5) about
6) around
7) for
8) for
9) with
10) into

Question 12.
Fill ANY FOUR of the following blanks with suitable forms of the verbs given in brackets. 4 × 1 = 4M
1. While I ……………….. (teach) grammar, a student raised a doubt.
2. A philanthropist ……………….. (think) about the welfare of others.
3. My nephew ……………….. (do) business in Hyderabad for the last ten
years.
4. The moon ……………….. (wax) and ……………….. (wane) during the cycle of a month.
5. I ……………….. (know, not) the right answer right now.
6. We ……………….. (have) hot coffee one hour ago.
Answer:
1) was teaching
2) thinks
3) has been doing
4) waxes… wanes
5) do not know
6) had

TS Inter 1st Year English Model Paper Set 10 with Solutions

Question 13.
Rewrite ANY FOUR of the following sentences as directed. 4 × 1 = 4M
i) Vegetables are washed before cooking.
(Change the sentence to active voice.)
ii) She had answered it already.
(Change the sentence to passive voice)
iii) The shopkeeper said to the customer, “Shall I show you the latest model ?”
(Change the sentence to indirect speech.)
iv) A visitor said, “What sultry weather !”
(Change the sentence to indirect speech.)
v) Natural flowers appeal more to our senses than artificial flowers.
(Change the sentence to positive degree.)
vi) 1 can face challenges, ………………. ?
(Add a question tag.)
Answer:
i) We wash vegetables before cocking.
ii) It had already been answered by her.
iii) The shopkeeper asked the customer if he could show him the latest model.
iv) A visitor exclaimed that it was very sultry weather.
v) Artificial flowers do not appeal to our senses as much as natural flowers.
vi) can’t I ?

Question 14.
Rewrite ANY FOUR of the following sentences correcting the underlined errors. 4 × 1 = 4M
1. It is my self.
2. She and me are twins.
3. Each of these girls have wide eyes.
4. Neither of these cars are worth the money.
5. Both didn’t attend the meeting.
6. We all didn’t go.
Answer:
1. It is I.
2. She and I are twins.
3. Each of these girls has wide eyes.
4. Neither of these cars is worth the money.
5. Neither attend the meeting.
6. None of us went.

Question 15.
Supply the missing letters to ANY EIGHT of the following words. 8 × 1/2 = 4M
i) thr_ _gh
ii) sli_ _tly
iii) gr_ _nd
iv) wo_ _y
v) sp_ _d
vi) ang_ _sh
vii) prev_ _us
viii) mi_ _ t
ix) rec_ _ve
x) p_ _ce
Answer:
i) through
ii) slightly
iii) ground
iv) worry
v) speed
vi) anguish
vii) previous
viii) might
ix) receive
x) piece / peace

TS Inter 1st Year English Model Paper Set 10 with Solutions

Question 16.
Identify the silent consonant letters in ANY EIGHT of the following words. 8 × 1/2 = 4M
i) align
ii) ghost
iii) leader
iv) straight
v) calf
vi) plumber
vii) wrap
viii) thistle
ix) attempt
x) burden
Answer:
i) align – g
ii) ghost – h
iii) leader – r
iv) straight – gh
v) calf – l
vi) plumber – b, r
vii) wrap – w
viii) thistle – t
ix) attempt – t
x) burden – r

Question 17.
Write ANY FOUR of the following transcriptions using ordinary English spelling. 4 × 1 = 4M
TS Inter 1st Year English Model Paper Set 10 with Solutions 1
Answer:
i) emphatic
ii) appearance
iii) mention
iv) gentleman
v) tremble
vi) sleep

Question 18.
Circle ANY FOUR of the words that sound different from the other words in that set with regard to the sounds of the bold letters. 4 × 1 = 4M
i) call cinema cute
ii) leisure sugar sure
iii) arm about aloud
iv) exit exam exercise
v) meal measure meant
vi) dread break bread
Answer:
i) cinema
ii) leisure
iii) arm
iv) exercise
v) meal
vi) break

Question 19.
Mention the number of syllables in ANY FOUR of the following words. 4 × 1 = 4M
i) popular
ii) today
iii) side
iv) plant
v) rainwater
vi) condition
Answer:
i) 3 – trisyllabic
ii) 2 – disyllabic
iii) 1 – monosyllabic
iv) 1 – monosyllabic
v) 3 – trisyllabic
vi) 3 – trisyllabic

TS Inter 1st Year English Model Paper Set 10 with Solutions

Question 20.
a) Analyse the bar graph given below and write about it in a paragraph. 1 × 4 = 4M
TS Inter 1st Year English Model Paper Set 10 with Solutions 2
b) Read the following paragraph and transfer the information into a tree diagram.
There are so many species of animals that we find living on the earth. Scientists grouped these animals into different classes . based on certain similarities they share. Animals are divided into vertebrates, ones with backbones and invertebrates, those without backbones. The vertebrates are basically divided into five classes. They are commonly known as mammals, birds, fish, reptiles and amphibians. Arachnids and insects are the two commonly known classes in the invertebrates group.
Answer:
a) Marks of students
The bar chart presents marks of three students in three subjects. Meena scored 70 in Telugu, 65 in Maths and in English just 50. Mala scored 65 in Maths, 50 in Telugu and only 40 in English. Megha secured 70 each in English and Maths but scored 60 in Telugu.
(OR)
b) Tree Diagram showing Species of Animals
TS Inter 1st Year English Model Paper Set 10 with Solutions 3

TS Inter 1st Year English Model Paper Set 9 with Solutions

Thoroughly reviewing TS Inter 1st Year English Model Papers Set 9 helps in understanding the examiner’s expectations.

TS Inter 1st Year English Model Paper Set 9 with Solutions

Time: 3 Hours 15 Minutes
Maximum Marks: 100

Section – A

Question 1.
Annotate ANY TWO of the following in about 100 words. 2 × 4 = 8M
a) “Cox, I shan’t want you today – you can have a holiday.”
b) One might think that growing trees is not a big deal but one would know the reality of it only when they do it on their won.
c) Education is not what a person is able to hold in his head, so much as it is what a person is able to find.
Answer:
a) Introduction: This line is taken from the one-act play, Box and Cox written by-John Maddison Morton. This play is regarded as the best farce of the nineteenth century.

Context & Explanation : Mr. Cox says these words. He reports the words of his master. That day, the owner permits Cox a holiday. So, Cox returns to his room. This is unusual for him. But, this fact gives a twist to the play. For the first time, Mr. Cox and Mr. Box meet each other in the room. Each finds fault with the other initially.

Critical Comment : Mrs. Bouncer’s folly is exposed. Thus the words play a crucial role.

b) Introduction : These touching lines are taken from the internet-based article. The Green Champion – Thimmakka. It depicts the magnificent achievements of an ordinary couple with a great commitment to conserve nature.

Context & Explanation: Every one feels that it is not difficult to grow trees. It is because they never grow any sapling in their life. Here we havd to remember the saying that empty vessels make much noise. Such type of people can say that growing trees is not a big deal. But, people who really try to grow trees can understand the foil and trouble undergoes. If it is an easy thing why do our governments spend crores of money on planting trees. It is because no one bothers about nature that is why governments take up such programmes. So, we have to appreciate Thimmakka and her husband to take up free planting mission.

Critical Comment: The essay describes the attitude of people here.

c) Introduction: This beautiful sentence is taken from the character building speech. “Two Sides of Life” delivered by Booker T Washington. This speech is extracted from his popular book ‘Character Building’.

Context & Explanation : The writer gives very forceful explanation of character building. He inspires teacher traines how they should be in their profession. He tells them to be honest If they don’t know anything, they have to accept it frankly and honestly. Their students will respect them for it It is because education is not possible to hold in one’s head. It is what a person is able to find It is not the correct notion,that the teacher should know everything.

Critical Comment : Here, the writer addresses the teacher trainees and advises them to have the character of frankness and honesty.

TS Inter 1st Year English Model Paper Set 9 with Solutions

Question 2.
Annotate ANY TWO of the following in about 100 words. 2 × 4 = 8M
a) 0 my Luve’s like the melodie
That’s sweetly pla’d in tune.
b) And in short measures life may perfect be.
c) I asked the professors who teach the meaning of life to tell me what happiness is.
Answer:
a) Introduction : This couplet is taken from the beautiful lyric A Red Red Rose, written by Robert Burns. It is one of the best lyrics of English poetry. It blends the eternity of love with the mortality of life.

Context & Explanation : The poet compares his love to a melody that is sweetly played in tune. His love is a song that is sung just so right in fact that it’s kind of sweet. His feelings are very profound. Critical Comment: Here, the poet compares his beloved to a sweet melody which is nice to hear.

b) Introduction: This is the concluding line of the beautiful lyric. The Noble Nature written by Ben Jonson. He is very well known for his comedy of humours like Every Man in His Humour.

Context & Explanation : The poet talks about what makes a man noble. He compares man to a sturdy oak and to a delicate lily in order to do this. He says that a person doesn’t become great or honourable by having long life or huge body. His greatest is analysed by his deeds. And to make man better or life perfect, the poet advises one to dead a meaningful life of light-like that of a lily. Thus, the focus of the poem is clearly on making man better. A person’s life is meaningful only if he does some acts of benefaction. Otherwise life is meaningless. Critical Comment: Leading a meaningful life makes it noble, Jonson asserts. He employs analogies from nature.

c) Introduction : This line is taken from the poem ‘Happiness’ written by Carl Sandburg. He is a famous American poet. The poem is extracted from his collection of poems, ‘Chicago Songs’.

Context & Explanation : It depicts the narrator’s experience. He wants to know what happiness is. First, he consults the professors for the answer. They represent the intelligence and success. But, they can’t answer it. They claim that they teach the meaning of life. Finally, he finds the real meaning of happiness from a crowd of Hungarians with their women and children under a tree.

Critical Comment : Here the line describes the narrators experience. He asks the professors about the meaning of happiness.

Question 3.
Answer ANY TWO of the following questions in about 100 words each. 2 × 4 = 8M
a) What gave Dr. Bannister strength in the final spurt ?
b) What does the boy think of his grandparents in his letter ?
c) Why was Thimmakka called Saalumarada ?
Answer:
a) Dr. Roger Bannister was the first man to run the race of one mile in 3 minutes 59.4 seconds. He narrates his eventual victory of race in this essay. In the final spurt, he got the strength from his will power. He had1 turned the last bend. There was only 50 yards more to be run.

His body tired and consumed all his energy. But, it went on running. That strength came from great will power. At that juncture, determination, dedication and strong will power lead him ahead. Therefore, he could succeed with his will power.

b) Raj Kinger’s Father, Dear Father is a thought provoking commentary on the education system prevailing now. This short write-up is a letter to a father.

Rahul is a school boy. He is very much happy with his grandparents who enjoy life.

He says that his grandfather had a carefree and beautiful childhood. His grandfather recollects how enjoyed in the mango and guava gardens. He says that his grandfather studies were secondary and living and experiencing
was the major subject. He asks his father very innocently whether his grandfather is lying. And his grandmother is semi-literate. But, she is happy with her kitchen work, gardening and reading the Bhagavad Geeta and other holy books. Thus, he thinks of his grandparents.

c) The present internet-based essay, the Green Champion – Thimmakka describes the magnificent achievements of an ordinary woman with an excellent commitment to conserve nature. Thimmakka, a woman more than 100 years in age, from Karnataka has been launded globally as the green champion for her planting mission. Thimmakka along with her husband planted over 8000 other trees. Even after her husband’s death, she continued her mission of planting trees. Her outstanding work earned her the name Saalumarada, which means a row of trees in Kannada. Thimakka is popular as Saalumarada Thimmakka due to her work. She continues her fight against deforestation. Her contributions are truly remarkable. With her achievements, she is called Saalumarada Thimmakka.

TS Inter 1st Year English Model Paper Set 9 with Solutions

Question 4.
Answer ANY TWO of the following questions in about 100 words each. 2 × 4 = 8M
a) “Dr. Ammangi Venugopal’s creativity is rooted deeply in the complexities and contradictions of modern life”, say observers. Explain the statement, taking ‘The Beggar’ as a reference point :
b) Life is queer with its twists and turns…. List a few twists as mentioned in the poem.
c) How is the feeling of love expressed in the poem A Red Red Rose ?
Answer:
a) The poem, The Beggar is written by Dr. Ammangi Venugopal. He is a creative genius. Actually, he has penned it in Telugu as Bichchagadu. It is translated into English by Elanaaga (Dr. Surendra). Dr. Ammangi is well aware of the complexities and contradictions of modern life. The complex problems farmers today face form the central idea of his moving poem. Farmers are the food providers to all. They struggle to survive. They starve, yet, they toil. Their feet bleed. Their eyes are full of shadows of their sad stories. Their hunched backs tell us how hard they work. Yet, their stomachs get no food. They are capable of feeding millions. And those millions do not include in them those farmers. How cruel the modern society responsible for this irony is! Thus, the poem shows the complexities of current times.

b) Edgar Albert Guest is very popular as a people’s poet. His poem, keep going is undoubtedly an inspirational poem. All through its twenty four lines of the poem the poem keeps on advising the reader never to quit. It openly admits that life may be a mixture of more pains and less pleasures. Goals may stand beyond your reach. Funds may be low. Needs may be more. Things do not always go the way we plant them. There are times when you will be over whelmed in a given aspect of your life. You have to face changes. Your journey is all about climbing up hill. Life is like a journey whereby some roads are tough and tiresome. Your circumstances deny your happiness. Your life has so many low moments that you lack humour. Instead of a smile, you let out a sigh. When you feel like you can’t go on take rest. But, dont’ quit your effort. Thus, life is full of twists and turns.

c) The poem A Red Red Rose is written by Robert Burns. It is one of the best lyrics of English poetry. It blends the eternity of love with the mortality of life. It is an address to the speaker’s lover to whom he swears eternal love and loyalty.

The speaker shares his romantic love for his beloved in this poem. His feelings are very profound. He com¬pares his beloved with a fresh and beautiful rose sprung in June and to a sweet melody as well. He also makes several promises to live his beloved forever. He makes a promise that he will return to her life after their temporary separation. He promises to be with her, no matter how long the journey takes.

Question 5.
Answer ANY TWJD of the following questions in about 100 words each. 2 × 4 = 8M
a) What are the thoughts in the mind of the youth when he chooses wealth ? What is the result ?
b) Helping the old is as good as playing the game. Elucidate with reference to the story “Playing the Game”.
c) “A spirited discussion springs up between a young girl and a colonel”. Discuss.
Answer:
a) “The Five Boons of Life”, by Mark Twain, presents a philosophical approach to life. It shows us how foolish we are in our priorities at times. The youth stands for man’s follies. He gets a chance, from a fairy to choose from ‘Fame, Love, Riches, Pleasures and Death’. He is led by false appearances. The fairy’s warning fails to correct him. He chooses ‘Pleasures’ first. He soon realises how painful those pleasures are ! He than opts for ‘Love’. He understands how grief follows love. Then, he goes for Fame. Again, it proves to be a wrong choice. Then he thinks ‘Wealth’ will make him happy. He plans to spend, shine, and feed his hungry heart with his mockers envy. He thinks he can buy everything the earth can offer. He is wrong, once again.

b) Arthur Mee is known for his humanism. He expresses it artistically. “Playing the Game” exhibits that rare quality. Alan is the central character. He loved cricket. Once, he got a chance to represent his school in cricket. On the scheduled day, Alan started for the ground early. But on the way, he noticed an old man struggling to walk. He was move. He held his helping hand to that aged man. Hence, he couldn’t reach the ground in time. He missed the much-awaited opportunity. He felt sad. But for this kind act, his parents presented him with his favourite bicycle. His classmates cheered him. The story proves, thus, that helping the old is better than playing the game.

c) “The Dinner Party”, by Mona Gardner, showcases the emotional strength of women. Deeds, not empty words, prove this point beyond anyone’s doubt. A colonial officer hosts the dinner party. The guests are just twenty. During the party, an animated discussion arises between a young girl and a colonel. The girl says women have advanced a lot from their earlier era of screaming at the sight of a mouse. The colonel contradicts her stand. He asserts men have an extra ounce of nerve control in a crisis. And he adds that ounce counts a lot. But the girl is right. The story proves at the end how strong a woman is in crises!

TS Inter 1st Year English Model Paper Set 9 with Solutions

Section – B

Question 6.
Read the following passage carefully and answer ANY FOUR questions given after it in a word or a sentence each. 4 × 1 = 4M
The news about the arrival of the elderly man from city had spread in the village by morning. The villagers said he was a tall stout man. He helped form associations in villages. He sported a kerchief like lawyers do. He brought a leather suitcase which was full of books. He knew all the bigwigs in the city. He would do away with all our troubles.

Questions:
i) Name the story from which this passage is taken.
ii) Who does the phrase the elderly man refer to ?
iii) How would the elderly man help the villagers ?
iv) What did he bring with him ?
v) The villagers talked about the elderly man … … … (fill in).
1) adversely.
2) appreciatively
3) accusingly
4) arrogantly
vi) Find the Phrasal Verb used in the passage that means put an end to; eliminate.
Answer:
i) Sanghala Panthulu
ii) Sanghala Panthulu
iii) He would help the villagers form associations and solve their problems.
iv) a leather suitcase, full of books
v) (2) appreciatively
vi) do(es) a way with

Question 7.
Read the following passage carefully and answer ANY FOUR questions given after it in a word or a sentence each. 4 × 1 = 4M
A Modern Marvel Taking Shape on Out Soil at Sangareddy
Can you’conceive of a construction without cement and steel ? Unimaginable ? But, a modern marvel-a 32 foot high Sri Chakra shaped temple is taking shape on a sprawling one and a half acre site with just natural elements like lime, jaggery, jute, gum, myrobalan fruit (karakkaya) paste, Indian bael (Maredu/Bilwa) juice, sand and stone (15 lakh pieces, some weighing 5 tons each).

With Lord Shiva as the presiding deity, this Sri Kailasa Prastara Mahameru (human body) Panchamukha Umamaheswara Devasthanam, at Phasalvadi village, near Sangareddy, is being devised, planned and executed by jyothir- vaasthu Vidyapeetham. Modelled after ancient temples like Konark, Hampi, this 20 crore rupee mammoth miracle is expected to have a life of 6,000 years, says Maheswara Siddhanthi, the man behind this project. Hundreds of expert engineers, skilled sculptors, eminent architects and famous artisans have been toiling since day one i.e., 14 June 2017 and are determined to complete it at the earliest. With concepts from epics as its fountain of inspiration, this temple looks certain to flourish as a spiritual centre with unique architectural features !

Questions:
i) Mention the unique feature of this temple in terms of construction materials used.
ii) Which organization is executing the work of this temple ?
iii) Where is this temple located ?
iv) Without using cement and steel, can the structure last long ? Support your answer with a sentence from the passage.
v) Name the temples that were studied to design this temple.
vi) Who is supervising this major project ?
Answer:
i) Without using steel and cement, it uses only natural elements like lime, jaggery, jute, gum,….
ii) Jyothirvaasthu Vidyapeetam
iii) Phasalvadi village, near Sangareddy
iv) Yes. It is expected to have a life of 6,000 years.
v) Konark, Hampi temples
vi) Maheswara Siddhanthi

Section – C

[Note : Answers of this section must be written at one place in the same Serial Order.]

Question 8.
Match ANY EIGHT of the following words in Coiumn – A with their meanings in Column – B. 8 × 1/2

Column A Column B
i) stretch a) understand
ii) monsoon b) eating grass by animals
iii) capture c) clearly
iv) cherish d) huge, enormous
v) realize e) humble
vi) evidently f) unbelievable
vii) modest g) rainy season
viii) graze h) an area of land
ix) incredible i) value
x) massive j) catch hold of something

Answer:

Column A Column B
i) stretch h) an area of land
ii) monsoon g) rainy season
iii) capture j) catch hold of something
iv) cherish i) value
v) realize a) understand
vi) evidently c) clearly
vii) modest e) humble
viii) graze b) eating grass by animals
ix) incredible f) unbelievable
x) massive d) huge, enormous

TS Inter 1st Year English Model Paper Set 9 with Solutions

Question 9.
Identify the parts of speech of ANY EIGHT of the following underlined words. 8 × 1/2 = 4M
Ah (1), then you (2) mean to say that this (3) gentleman’s smoke (4), instead of emulating the example of all (5) other sorts of (6) smoke, and (7) going up the chimney, thinks (8) proper to affect a singularity (9) by taking the contrary (10) direction.
Answer:
1) Ah – interjection
2) you – pronoun
3) this – adjective (determiner)
4) smoke – noun
5) all – adverb
6) of – preposition
7) and – conjunction
8) thinks – verb
9) singularity – noun
10) contrary – adjective

Question 10.
Fill ANY EIGHT of the following blanks with a, an or the 8 × 1/2 = 4M
1. Amitabh Bacchan is …………….. famous actor.
2. Are you …………….. vegetarian ?
3. It is …………….. absurd story indeed.
4. I don’t believe him. He is …………….. liar.
5. What …………….. lovely Villa !
6. It is …………….. one-man show !
7. I never witnessed such …………….. long queue for Covaxin.
8. He is …………….. United Nations exployee.
9. We have quite …………….. few books onYoga.
10. I saw …………….. accident this morning.
Answer:
1) a
2) a
3) an
4) a
5) a
6) a
7) a
8) a
9) a
10) an

Question 11.
Fill in ANY EIGHT of the following blanks with suitable prepositions. 8 × 1/2 = 4M
When you go_____(1)_____your classrooms, I repeat, try to forget and overlook any weak points that you may think you see. Remember, and dwell_____(2)_____the consideration that has been given_____(3)_____ the lesson, the faithfulness_____(4)_____which it was prepared, and the earnestness_____(5)_____which it is presented.

So it seems! Far be it_____(6)_____me. Bouncer, to hurry your movements, but I think it right to acquaint you_____(7)_____my immediate intention_____(8)_____divesting myself_____(9)_____my garments, and
going_____(10)_____bed.
Answer:
1) into
2) upon
3) to
4) with
5) with
6) from
7) with
8) of
9) of
10) to

Question 12.
Fill ANY FOUR of the following blanks with suitable forms of the verbs given in brackets. 4 × 1 = 4M
1. Don’t get off the train till it …………….. (stop).
2. Listen ! Somebody …………….. (scream).
3. A parrot …………….. (repeat) our voice as soon as it listens to it.
4. Vinay …………….. (lose) the job lastyear because of his misbehaviour.
5. Dhirubai Ambani …………….. (not, live) in a costly house even when he was a famous industrialist.
6. …………….. he …………….. (play) tennis dally ?
Answer:
1) stops
2) is screaming
3) repeats
4) lost
5) did not live
6) Does…play

Question 13.
Rewrite ANY FOUR of the following sentences as directed. 4 × 1 = 4M
i) Who could help him ?
(Change the sentence to passive voice.)
ii) Hardik said to Annu, “Go and study.”
(Change the sentence to indirect speech.)
iii) The mother said to her son, “When will you have your breakfast ?”
(Change the sentence to indirect speech.)
iv) Robert Frost is one of the best American poets.
(Change the sentence to comparative degree.)
v) Jupiter is bigger than any other planet.
(Change the sentence to positive degree.)
vi) Let’s play tennis, …………….. ?
(Add a question tag.)
Answer:
i) By whom could he be helped ?
ii) Hardik asked Annu to go and study.
iii) The mother asked her son when he would have his breakfast.
iv) Robert Frost is better than many other American poets.
v) No other planet is as big as Jupiter.
vi) shall we

TS Inter 1st Year English Model Paper Set 9 with Solutions

Question 14.
Rewrite ANY FOUR of the following sentences correcting the underlined errors. 4 × 1 = 4M
1. They are staying in the same flat for the last many years.
2. How long are you waiting here ?
3. He is interested to do a job.
4. They have moved to the new house last week.
5. He is having many imported clothes.
6. As soon as I opened the doors, the birds fly away.
Answer:
1. They have been staying in the same flat for the last many years.
2. How long have you been waiting here ?
3. He is interested in doing a job.
4. They moved to the new house last week.
5. He has many imported clothes.
6. As soon as I opened the door, the birds flew away.

Question 15.
Supply the missing letters to ANY EIGHT of the following words. 8 × 1/2 = 4M
i) sa_ _ling
ii) rup_ _s
iii) hu_ _and
iv) res_ _rces
v) s_ _rce
vi) su_ _icient
vii) ma_ _ive
viii) vi_ _age
ix) init_ _tive
x) a_ _roval
Answer:
i) sappling
ii) rupees
iii) husband
iv) resources
v) source
vi) sufficient
vii) massive
viii) village
ix) initiative
x) approval

Question 16.
Identify the silent consonant letters in ANY EIGHT of the following words. 8 × 1/2 = 4M
i) thorough
ii) who
iii) benign
iv) receipt
v) rhythm
vi) diversity
vii) nursery
viii) column
ix) curd
x) kneel
Answer:
i) thorough – gh
ii) who – w
iii) benign – g
iv) receipt – p
v) rhythm – h
vi) diversity – r
vii) nursery – r
x) kneel – k
viii) column – n
ix) curd – r

TS Inter 1st Year English Model Paper Set 9 with Solutions

Question 17.
Write ANY FOUR of the following transcriptions using ordinary English spelling. 4 × 1 = 4M
TS Inter 1st Year English Model Paper Set 9 with Solutions 1
Answer:
i) success
ii) effort
iii) excitement
iv) worry
v) previous
vi) athletic

Question 18.
Circle ANY FOUR of the words that sound different from the other words in that set with regard to the sounds of the bold letters. 4 × 1 = 4M
i) suggest beggar luggage
ii) cap tap tape
iii) shake take talk
iv) house hour mouse
v) buy try ray
vi) thin this thick
Answer:
i) suggest
ii) tape
vi) this
iii) talk
iv) hour
v) ray

Question 19.
Mention the number of syllables in ANY FOUR of the following words. 4 × 1 = 4M
i) punctual
ii) increase
iii) room
iv) mantelpiece
v) breakfast
vi) gracious
Answer:
i) 2 – disyllabic
ii) 2 – disyllabic
iii) 1 – monosyllabic
iv) 3 – trisyllabic
v) 2 – disyllabic
vi) 2 – disyllabic

TS Inter 1st Year English Model Paper Set 9 with Solutions

Question 20.
a) The given below bar graph shows how much dietary fibre is found in certain fruits. Convert the information into a paragraph. 1 × 4 = 4M
TS Inter 1st Year English Model Paper Set 9 with Solutions 2
(Or)
b) Draw a flow chart based on the information given below.
The following process is the description of how a post office transfers a letter from a sender to a receiver. First, the sender posts the letter in a post box. Next, the box is opened. Then the – contents in it are sorted out. Then they are kept in a bag and the bag is tied. The destination is written on the bag. The bags are sent to the district post office. The district post office sends the bags to the destination village / town post offices. The destination post office receives the letters. The received letters are arranged and sorted out. The post man delivers the letters to the addressees.
Answer:
a) Fibre Content in Fruits
The given bar graph presents the details of fibre content in various fruits. The guava stands tall with six (6) grams of dietary fibre per a serving of one cup. Next comes the pear with five (5) grams per unit. The third in the order is the apple with four (4) grams per a cup. The banana and the orange have almost the same quantity of dietary fibre – three (3) grams per cup.
(Or)
b) Flow Chart depicting the Process of Delivering Letters
TS Inter 1st Year English Model Paper Set 9 with Solutions 3

TS Inter 1st Year Political Science Model Paper Set 7 with Solutions

Thoroughly analyzing TS Inter 1st Year Political Science Model Papers Set 7 with Solutions helps students identify their strengths and weaknesses.

TS Inter 1st Year Political Science Model Paper Set 7 with Solutions

Time: 3 Hour
Max Marks: 100

Section – A
3 x 10 = 30 Marks

Note: Answer any THREE of the following questions in not exceeding 40 lines each. Each question carries 10 Marks.

Question 1.
Define Political Science and explain its scope.
Answer:
Introduction:
Political Science is a premier social science. It is mainly concerned with the study of the state in its relation with Society.
Citizens, Associations and the world at Large. Aristotle is regarded as the Father of Political Science. He wrote famous book ‘THE POLITICS”.

Meaning:
The word politics is derived from the ancient Greek word “POLIS” meaning city. State and polity from ‘Poletieia’ meaning government of constitution. Politics came to mean the study of state and government and the Institutions of the state.

Definitions:
Political Scientists gave various definitions on Political Science.
They are as follows:

  1. J.W. GARNER: ‘Political Science begins and Ends with the State”.
  2. R.G. GETTLE: Political Science is” The historical investigation of the state in the past, an analytical study of the state of present and what the state ought to be in the future”.
  3. ROBERT DAHL: “Political Science is the Scientific study of importance of power, Authority and influence.”

Scope of Political Science: The scope of Political Science means the subject matter covered by it or the topics which are
included in its study It may be explained in the following ways:
i) Study of man in relation to the Society, State and Government: Political science is concerned with the perennial and central issue of establishing proper relationship among state, society and government with individual. Aristotle stated that Man is a social and political animal as well.

ii) Study of State: Political science explains the origin, evolution and purpose of the state and its intimate relationship between the state and the citizens. It explains the various theories of the origin of the state and it also studies the nature, functions and various theories of the state.

iii) Study of the Government: Scope of political science includes the study of government. Political science explains the relationship between state and government. The state realises it’s aims and objectives through the government. Government formulates various policies, programs and their implementation for well being of the people. Political science also studies various forms and structures of the government and their merits and demerits.

iv) Study of Associations and Institutions: There are several associations and institutions which influence the life of the individual. Political science studies various associations, institutions and their relationship with the state. Political science explains structure, nature and functions of the various associations and institutions. It also studies voluntary oranisations and their role in the political processes.

v) Study of Rights and Duties: Scope of Political Science includes the study or rights and duties of citizens. In recent times, issues relating to civil rights, human rights and civil society got significance in the study of political science.

vi) Studies of National and International Issues: The scope of Political science comprises the study of national and international affairs. The political science deals with the matters relating to nationstate, territorial integrity and its sovereignty it also studies international aspects like armaments and disarmaments, balance of power, defence and security studies. It also covers international law, international organisations etc.

vii) Study of Comparative Government and Polities: The importance of the comparative study of government and politics has been increased in recent times. Political science covers the study of various world governments, their structurés and functions. It studies the relationship among the different political systems in the world.

viii) Study of Modern Political Analysis: The 20th century political science is regarded as a study of sharing and shaping of power, and it’s execution in a day to day political process. Political science studies the modern concepts like, political socialisation, political participation. political development, political culture and political communication.

ix) Study of Public Policies: Modern political scientist like David Easton, Gabriel A. Almond, Charles Merriam argued that
political science is a policy science. They considered political science as the study of formulation, execution and evaluation of public policies. They also emphasised the study of political parties, pressure groups, mass media and organs of the governments and their influence in the formulation of the public policies. It also studies major polices like Agliculture policy,industrial policy. Environmental policy, Reservation policy and Education policy etc.

Thus, the scope of political science has, in recent times, extnded to the study of above concepts and theories and has become one of the most relevant social sciences.

TS Inter 1st Year Political Science Model Paper Set 7 with Solutions

Question 2.
Explain the differences between State and Associations.
Answer:
Differences between State and Associations:

State Associations
1. The state is permanent. 1. Associations are temporary.
2. The state has sovereign power. 2. Associations cannot have sovereignty.
3. The state has fixed boundaries. No state is universal or world-wide. 3. The associations cannot have fixed territorial boundaries. Some associations are international and universal in character. Ex: U.N.O. Red Cross Society The Lions international etc.
4. The membership of state is compulsory. Every citizen naturally becomes the member of the state. 4. But the membership of an association is optical. It depends on the will and wish of the people.
5. A man can become a member of one state only at a time. 5. But they can be member of any number as associations as he desires.
6. The state has multifarious functions concerning almost the whole of man’s life. 6. The functions of an association are singular and common to its members only.
7. The state makes the laws, violation of which is visited by punishment. 7. The associations cannot make laws, but makes their own rules and regulations.
8. The state can impose compulsory taxes on its people. 8. Associations cannot impose taxes on its members. They can only thrive on voluntary contributions of their members.
9. The aim of state is broader. 9. The aim of the Associations is limited.
10. State can enforce its decisions upon recalcitrant members or punish them for disobedience. 10. The associations cannot enforce is decisions up on members.
11. The state is superior to all associations. 11. Many associations in a state which live and depend on the mercy and pleasure of the state.

Question 3.
Define Nationality Explain the essential elements of Nationality.
Answer:
Introduction: The concepts of Nation and Nationality have become important components in the domain of International Relations and political science respectively Both inspired the people of several countries with patriotic feeling prior to the two world wars.

The Events that took place in the erstwhile Soviet Union, ethnic Riots between serbians and croatians in the former Yugoslavia, the unification of East and west Germanies, the peace talks between Israel and Palestine Liberation Organization (PLO) on West Asia etc., refled the serious concern of the people for realising Nationality and Nation States.

Meaning: The Word “Nation’ is derived from a latin word “NATIO” which means ‘BORN’ (BIRTH) or “Common Descent”.

Definitions:

  1. LG. Gettle: “Nationality is a population having the common bonds of Race, Language, Religion, Traditions and History
  2. J.H. Rose: “Nationality is a union of Hearts once made and never unmade”.
  3. J.W Garner: “Nationality is a group or portion of population which is united by Raal and other bonds”.

Essential Elements of Nationality:
1. Purity of Race: Racial purity helps in the formation and strengthening of the idea of Nationality. Race is a physical phenomenon. It depends on certain distinctions of skull, stature, hair; complexion,etc. These distinctions serve as a cementing bond among the members of a group.

But we should remember that cômmon race is not an indispensable factor in the growth of Nationality Modern races are so mixed that none of them can daim to be pure. Pure races have disappeared because of wars and migrations. Racial purity is now a myth only Ex: Canada and United states have transformed into single nations inspite of their racial diversities in their respective populations. Similarly, Australia and Britain are two distinct Nations although they belong to one racial stock.

2. Common Language: Language plays a key role in the promotion of nationality. The philosophers and scientists said that common language is essential for the development of nationality. Language is a medium to express all their feelings. It helps to express one’s own selves to have cordial relations and to share the miseries and happiness in a group languages also promotes common feelings anti traditions. Common language promotes the feeling of oneness and keeps the entire race on single track.

3. Common Religion: Religion is one important factor to strengthen nationality. There are many instances when people of different nationalities with common religion remain citizens in the same state. For instance, the main reason for the partition of Indian subcontinent into India and Pakistan in 1947 lies in the religion.

4. Geographical Unity: Geographical unity is necessary for the emergence of nationality. Nationality sentiments prevail and develop among the people living in a single geographical area. The people residing in such an area love, worship their country and make sacrifices for the sake of their motherland. People, who belong to one religion, converse the same language, same race living in a geographical area inculcate and improve their nationality sentiments.

The formation of Israel in 1946 was purely due to the feelings of the hitherto wandering Jewish people to live in a single geographical area. Hence their desire of live in a territory made them united. This ultimately transformed them as patriotic persons.

5. Common History: Common History is considered as an important element of Nationality It invokes an inspiration among the people and binds them together. Some historical incidents may give a chance to the people to develop national sentiments. Ex: Indians have learnt the lessons of Nationalism from the British legacy.

6. Common Culture: Culture in its broad sense means a way of life. It is reflected through certain common elements like dress, customs, conventions, food habits, religious beliefs, ethical values, etc. They easily develop into a single Nation. These elements bind the people together and hold to gether.

7. Common Political aspirations: Nationality sentiments prevail and develop ìmong the people having common political aspirations. The political ideas, conventions and institutions which were formed due to. the single political rule will have a considerable impact and influence over the people. For instance, the Swiss people love very much their direct democratic devices in political matters.

Similarly, the Americans express the feeling of worship towards their constitution. The British people also feel proud of their political and judicial institutions like rule of law, parliamentary democracy and judicial review etc.

8. Common Economic ties: This element of nationality has been stressed by ‘Karl Marx. Since then onwards the importance of this element has been increasing. The Russians have great regard for their economic system, even though there exist diversities. Their unflinching love for socialism inspired nationalism among them. They successfully repulsed the attacks of Germany during the Second World War. Thus the common economic ties made them united and integrated them into a nation.

TS Inter 1st Year Political Science Model Paper Set 7 with Solutions

Question 4.
Explain different kinds of law.
Answer:
Many political philosophers gave their classifications on laws in different ways. Of them the classification given by Maciver is mentioned as worthy which can be explained in the following points.

1. Natural Law: Natural law is also known as divine law. It is abstract. It is not created by any human agency. It is considered as the gift of nature, based on metaphysical power. It refers to the use of reason to analyse human nature. It is written in the heart of human beings by the ringer of God.

2. Positive Law: Positive law is created by the human agency. It is also known as political law. It is framed on the basis of the existing social and political conditions. It is sanctioned by the Sovereign Political Authority. Violation of positive law leads to punishment.

3. Constitutional Law: It is a basic law of any state. It defines the political system. All the basic principles of administration are included in this type. All other laws in the Statç are subservient to constitutional law. It is framed by the constituent assembly.

4. Ordinary Law: It determines the relation between the state, administration and people. These laws are framed by a group of officials authorised by law.

5. Public Law: It regulates the relation between people and state. These laws are formulated by state for society.

6. Private Law: It regulates the relation between citizens. It protects the rights of citizens. It also called civil law.

7. Administrative Law: It regulates the administrative relations between the authorities and people. Administrative law brings discipline among the personnel in the government. Now it is implementing in France and India.

8. General Law: It deals with the private affairs of individual in relation to the State. It covers the laws relating to Marriage, Divorce, Contract etc.

9. Statutory Law: Statutory law is the greater part of modern law. It is enacted by the legislature of a State for the day-to-day administration.

10. Common Law: Common law is a customary law. It is a product of customs and traditions which are popular among the people. The courts accept common law as a part of the legal system.

Question 5.
Define Rights. Describe the Civil and Political Rights.
Answer:
Introduction: Rights are the essential conditions for the development of the personality of individuals. They are upheld by the laws of the state. They are regarded as a power or privilege which the law invests in a person. They are treated as the sum total of the opportunities meant for enhancing one’s personality. Individuals cannot achieve progress in the absence of the rights.

Definitions of Rights: Political scientists have defined the term Right in several ways. Some of their definitions are explained below:
1. Earnest Barker: ‘Rights are the external conditions necessary for the development of the capacities of the personality of the individual.’

2. Beni Prasad: ‘Rights are nothing more and nothing less than those social conditions which are necessary for the development of personality of individuals.”

3. Bosanquet: “A right is a claim recognised by the society and enforced by the state.’

4. T.H. Green: ‘Rights are those powers claimed and recognized as contributory to the common good.”

5. H.J. Laski: “Rights are those conditions of social life without which no man can seek in general to be himself at his best.”

civil Rights: Civil rights aim at providing basic conditions for individuals to lead a happy and dignified social life. These rights are considered vital for a civilized society Social life becomes impossible in their absence. Individuals in a civilized society enjoy the following Civil rights.

These are
1. Right to life:
This is the most important civil right. T.H. Green considered it as the most fundamental civil right. This right provides security to the individual’s life. Individuals cannot lead their lives in the absence of this right. This right is based on the premise that the life of an individual is valuable not only to himself, but also to the society and the state as a whole. Hence it prescribes at large the state to extend protection to the life of individuals. However, it empowers the state to impose some reasonable restrictions upon the individuals. The state can insist any person to sacrifice his life for the sake of the nation This right also includes the right of self-defence.

2. Right to liberty: This right enables individuals to have freedom in various walks of life, It makes their lives worth living. It enables them to develop their personality in various spheres. It includes various freedoms such as freedom of movement, speech, expression, thought, residence etc.

3. Right to equality: This right implies that individuals are equal before law. It forbids discrimination on the basis of ones caste, colour, creed, education, region, race, religion, wealth etc. It enables equal treatment to all persons provides scope for uniform application of laws. It enables equal opportunities to all persons in social, economic and political fields.

4. Right to property: This right enables every individual to acquire, enjoy, donate or inherit the property. It is essential to the individual for securing higher standards of living. This right is crucial for the growth of individual’s personality.

5. Right to family: Family is a fundamental social institution. This right enables individuals to maintain family relations in society. Consequently. individuals will have freedom to marry persons of their choice. They will have choice to procreate children and rear their offspring. However, the state can impose certain restrictions upon this right keeping in view the national interests. For example. until recent’ times China imposed severe restrictions against their citizens in the size of their families. Recently it has made some amendments in this regard.

6. Right to religion: This right allows the individuals to have freedom to practice, propagate and profess any religion of their choice. Every individual is at liberty to preach or practice the religious doctrines as they like. The secular states provide religious freedoms to their citizens.

7. Right to contract: This right provides freedom to every individual to enter into contract or legal arrangements with others regarding his life, property and work. It regulates the two parties in carrying their contracts in letter and spirit. The state recognizes only those contracts which are helpful to the common well being of the people.

8. Right to education: In the modem era education is regarded as vital to every individual. Uneducated and innocent individuals cannot play an active role in public affairs. Similarly, illiterate persons cannot fully make use of their abilities. Education and literacy enable the people to understand the problems of the society and policies of the government. This right guarantees a minimum level of education to every citizen in democratic states.

9. Right to form associations and unions: This right enables individuals to form associations and unions for releasing some specific objectives. Individuals may join, continue or keep away from the membership of associations according to their will and pleasure. The State is empowered to impose restrictions against those associations which ignore the welfare of the nation.

10. Right to constitutional remedies: Civil rights are meaningless in the absence of this right. This right is essential to every individual for safeguarding his rights. This right empowers a person (who was deprived of his liberty due to the intervention or manhandling by others including the government) to seek justice and relief from the concerned judicial organizations. The affected individuals are authorised to approach an appropriate, court for correcting such imbalance. in this regard the higher judicial organizations issue several writs and effectively check such tendencies. These writs are in the form of Habeas Corpus, Mandamus, Prohibition, Quo-warranto and Certiorari etc. Political Rights: Political rights are those rights which enable the individuals to participate in the political affairs of the state.

The following are the important political rights:
1. Right to vote: Right to vote is the most important political right enjoyed by the citizens in modern democratic states. It serves as a powerful weapon for adult citizens in choosing their representatives to various legislative bodies. It makes them as real sovereigns. All the citizens are entitled to this right without any discrimination based on creed, colour, language, race, region, religion, sex etc. However, persons such as aliens and minors are deprived of this right.

2. Right to contest in elections: This right empowers the citizens to contest as candidates to various legislative bodies in the state. Especially this right enables those, who have political sagacity, enthusiasm and dynamic nature, to actively participate in the political dynamics of the state. As a result, it increases political enthusiasm among the citizens. Such an element is considered as a base of democratic polity.

3. Right to hold public offices: This right provides opportunities to the citizens to hold various public offices for a definite period. It gives no scope for exclusion of citizens or conferring special privileges to some at the cost of others. This helps the citizens to exercise authority in a dignified manner.

4. Right to petition: This right enables the citizens to forward competitions denoting their requirements or grievances. It is considered as a vital political right in the modem state. The citizens could be able to find solutions to their immediate or long pending issues by bringing them to the notice of the government through this right. It also helps the public authorities to know the grievances of the people and attend to them properly and promptly.

5. Right to criticism: This right gives opportunity to the citizens to criticize the various public policies and programmes. It also enable them to highlight the omissions and commissions of the leaders, and administrative personnel at various levels. It also gives scope for the citizens to render positive and constructive criticism about the on goings in the government from time to time. Ultimately it keeps the administrative authorities and policymakers to be vigilant in discharging their obligations.

TS Inter 1st Year Political Science Model Paper Set 7 with Solutions

Section – B
8 x 5 = 40 Marks

Note: Answer any EIGHT of the following questions in not exceeding 20 lines each. Each question carries 5 Marks.

Question 1.
What is the nature of Political Science?
Answer:
The traditional writers considered the study of state and government as the proper sphere of political science. The state and government are both closely related. There can be no state without government. The sate and government, therefore, have been the central subjects of the study of political science.”

In the period after the Second World War political scientists in the West developed new theories and concepts in the discipline of political science. American political scientist. Harold Laswell, in the 1930s, defined politics as the science of the study of power because the state is a structure of power and, the business of the political scientists is to examine ‘Who gets Power, When and How? Twenty years later during 1950 s, a school of thinking shifted the emphasis and redefined politics as a policy-making science. Modem writers have expanded the scope of poLitical science. They describe political science ‘as the science of the study of power’-or a policy-making science; or a fundamental activity in every organized group of human
life.

Broadly speaking, the study of Political Science developed in two strands: the normative and the empirical. In the normative studies of political science, the main focus is on norms, values and goals or ends of political life and activity. Most of the studies of the concepts such as liberty, equality, justice and empowerment and the study of ideologies are covered under normative studies. The empirical studies, both quantitative and qualitative, focus on facts and the actual processes or means that subscribe to the achievement of goals.

The study of the functioning of government and other institutions and the human interactions in different capacities come under empirical studies. These include, the studies on public policy, voting behaviour, political parties, pressure groups and social movements.

Question 2.
Explain any four kinds of Sovereignly.
Answer:
Introduction: Sovereignty is the most important characteric of the modem state. The state is distinguished from other Associations or institutions only by sovereign power.

Definition:
Willoughby: “Sovereignty is the supreme will of the state”.
Jean Bodin: “Sovereigñty is the supreme power of the state over citizens and subjects unrestrained by law”.

Kinds of Sovereignty: Sovereignty is classified into several kinds as mentioned below;

  1. Nominal sovereignty
  2. Real sovereignty
  3. Legal sovereignty
  4. Political sovereignty
  5. Popular sovereignty
  6. Dejure sovereignty
  7. Defacto sovereignty

1. Nominal sovereignty:
Nominal sovereignty is also known as titular sovereignty. Nomial sovereignty implies possession of sovereign powers only in name. The Queen in Britain, the Emperor of Japan, the President of India etc., are some examples of nominal sovereignty.

2. Real sovereignty:
The real sovereign actually possesses the sovereign power. He discharges such authority bn behalf of the nominal sovereign head. Accordingly, the nominal soverign person exercises his powers only on the advice of the real sovereign persons or body of persons. The Prime Ministers of England and India etc., are some best examples of this kind of sovereignty.

3. Legal sovereignty:
Legal Sovereignty denotes the supremacy in terms of formal law. Only the legal sovereign is competent to issue the highest orders. It transcends even the divine law or the normal laws and the public opinion. The court of law recognizes only the legal sovereign and accepts its orders. The best example of legal sovereign is the king-in-parliament in England. The legal sovereignty is definite, deliberate, decisive and precisely known. The president of india is also an example of legal sovereignty.

TS Inter 1st Year Political Science Model Paper Set 7 with Solutions

4. Political sovereignty: Behind the legal sovereignty there is the political sovereignty. In a democratic state while the legal sovereign is the supreme lawmaking power; there is behind it another sovereign whose will is the ultimate and final source of authority. Gamer observes: “Behind the legal sovereign, however, is another power, legally unknown, unorganized, and incapable of expressing the will of the state in the form of legal command, yet with a power to those mandates the legal sovereignty will in practice bow and whose will must ultimately prevail in the state.

5. Popular sovereignty: The concept of popular sovereignty is different from that of political sovereignty. It means that sovereignty ultimately resides in the people. The doctrine of popular sovereignty is the product of the sixteenth and seventeenth centuñes. It emerged as an expression of resentment of the people against the despotic authority of the kings and their reliance on the theory of Divine Right.

Popular sovereignty attributes ultimate sovereignty to the people. This theory, first hinted by the John Locke, was later expounded by Rousseau and it becomes the slogan of the French Revolution.

6. Dejure sovereignty: The term ‘De Jure’ denotes authority exercised according to law. De Jure sovereignty is the power possessed and exercised by a legally competent authority. It issues orders and enjoys command over all persons, institutions, and organizations in the state. The Queen in Britain and the President in India are some examples of the De Jure sovereignty.

7. De facto sovereignty: The French term ‘De facto’ implies ‘real’. De facto sovereign is a person or a body of persons who exercise such authority in the last resort and at the final stage. De facto sovereign may not be a legal sovereign. Hs authority is based not on law but on physical force. De facto sovereign may be a king, dictator or religious priest.

Question 3.
Describe the merits of Nationalism.
Answer:
The Nationalist character of the Modem State has the following merits.

  1. Nationalism removed the mutual differences, personal animosities and internal feeds. It promoted unity integrity and solidarity among the people of a nation. It enabled them to understand the neighbour’s point of view. So, it promoted closer understanding among the people.
  2. Nationalism made the people obey the government.
  3. It helped in achieving the progress of a nation in a short period.
  4. It accelerated the pace of the development of the state. It provided a democratic bare to the government and those strengthened the administrative system.
  5. It is anti-imperialistic. So it does not allow economic exploitation.
  6. It secured political stability and peacefull social atmosphere.

Question 4.
What are the characteristics of Liberty?
Answer:
Meaning: The term Liberty is derived from the Latin word “LIBER” which means free from restraints.
Definition: “Liberty means the absence of restraints”. – J.R. Seely
Characteristics of Liberty: The following are the important characteristics of Liberty.

  1. Liberty is a dynamic concept. its interpretation varies according to time, place, and wishes of the people.
  2. Liberty always opposes political subjugation, imprisonment, and slavery
  3. It always aims at realizing the aspirations of the individuals.
  4. Liberty always means absence of irrational restraints and presence of favourable conditions.
  5. It is the product of rights.
  6. It is essential for the realization of human personalities.
  7. It is found only in democratic states.
  8. It is manifest in the form of rights.
  9. It does not mean license to do whatever a person wants. It is always subject to limitations.

Question 5.
Discuss Individualism.
Answer:
Meaning: Individualism means the state should leave the individual alone. This theory is also known as the Laissez Faire theory. Laissez Faire is a French term which means ‘leave alone’. It regards the individual as the centre of social life. According to this theory, the individual freedom should be given maximum scope and the state interference should be reduced to the minimum.

The individualists regard state as a necessary evil’. It is necessary because it has to protect the individual from violence and fraud. It is an evil because its existence is a threat to individual freedom. So it is desirable to have state’s interference as little as possible. Lesser the functions performed by the state, the more is the liberty enjoyed by the individual.

The state should perform the following limited functions:

  1. Protection of the individual and of the state from foreign aggression.
  2. Protection of the individual against one another.
  3. Protection of property from robbery and damage.
  4. Protection of individual from false contracts and breach of contracts.

Question 6.
How is naturalised citizenship acquired?
Answer:
Naturalized Citizenship: Citizenship may also be acquired through naturalization. According to this method, an alien will become a citizen after fulfilling certain conditions. These conditions vary from state to state. Some of them may be summed up as follows.
1. Residence: An alien who resides in a state for a prescribed period automatically become its citizen. Residence in any part of the state is a must for an alien. The period of residency varies from state to state. For instance it is 5 years in Britain and United States and 10 years in France respectively.

2. Choice: The children of alien parents could receive citizenship of the state according to their option and choice.

3. Application: An alien in a state may apply for the citizenship of that state. Then the government of that state considers his application on its merits. It grants citizenship to him with or without some conditions. These prescribed conditions refer to a minimum period of residence, good moral character, financial capability and knowledge of one of the national languages. Besides, an alien must take an oath of allegiance before he assumes the citizenship of another state.

4. Fixed Assets: An alien who buys some portion of land or acquires some fixed property can acquire citizenship in a state.

5. Service Public or Private: An alien who service in government of a state or in a private recognised enterprise could
become the citizen of that state. He is entited for which citizenship the serves in the public or private authorised departments. He may also be given Citizenship if the renders meritorious service in another state.

6. Marriage: An alien woman acquires citizenship of a state when she marries the citizen of that state. In some countries when a person marries an alien, citizenship of either of the husband or wife is acquired. For instance, a British lady will acquire Indian citizenship if she marries an Indian citizen. Japanese women do not lose their citizenship even if they marry persons of alien states. The alien person on the other hand, acquires the citizenship of Japan if he marries a Japanese lady.

Question 7.
Explain any four essential conditions for the success of Democracy.
Answer:
1. Sound system of Education: The success of democracy requires adequate education for the citizens. Ignorance. innocence and undereducation prevent them from adopting right attitudes and large-scale reforms. Education sharpens the intellect of individuals. It develops a proper understanding of various things. It makes the citizens vigilant. Besides, this enables them to assess. and criticise the policies of government.

2. Enlightened Citizenship: Enlightened citizens are an asset to the democratic state. They can exercise proper vigilance. They can actively participate in public affairs and help their fellow citizens in the exercise of their rights and discharge of their responsibilities. They extend co-operation to the government in all its good work.

3. Independent Press: An independent press is a prerequisite democracy. It enables the people to receive accurate and unbiased information regarding the activities of the government. It not only keep’ s the people in touch with government activities but also ventilates their grievances. It strives to promote a harmonious relationship between the people and the government.

4. Strong Opposition: The success of parliamentary democracy depends to a great extent on the strong and effective opposition. Such an opposition will act as a check against the government by pointing out its lapses. in this regard, the role of opposition in some advanced states like Australia, Britan and United States is liable. In India to the opposition parties performed great role in many times.

5. Decentralization of powers: Decentralization of powers and establishment of democratic institutions at the grassroots level is indispensable for the healthy organisation of democratic institutions. The representative bodies at the grass roots level (as known as Panchayat Raj Institutions (PRIs) in India) will act as the mini legislatures. The residents of local areas will be able to know how to exercise their franchise.

6. Absence of economic dispartities: Democracy can not function smoothly when there are economic disparities in a country When a country comprises a large number of poor people and a few wealthy persons, democracy could not work successfully.

7. Social Equality: Social equality is another pre-requisite of democracy Caste, class and racial differences will impede the healthy working of democracy. Such elements encourage of democratic polity To be successful, democracy must open its doors to everybody on equal ‘basis by providing equal social opportunities to all in social sphere. Social equality must not only be proclaimed but also be practiced.

8. Faith in democracy: Certain democratic beliefs and values like individuals worth, need for tolerance of differences, decisions through discussions etc., should be inculcated among the people.

9. Sagacious Leadership: Sagacious leadership is another essential condition of democracy. Sagacious leaders, by dint of their administrative acumen, political propriety, social ‘commitment and economic perspective, will be able to lead the democratic state to greater heights of glory.

10. Honesty and transparency: Honest persons belonging to various walks life, when entrusted with major responsibilities of the government, will strive for the success of democracy. Similarly transparency in administration also acts as a basic ingredient for the success of democracy.

11. Absences of militarism: Democracy functions mostly in countries which are relatively free from militarism. In rules out the use of force and believes in the worth of individuals. It provides adequate opportunities to the people basing on worth, ablility and dedication militarism, on tlie other hand, demands concentration of authority and favours despotism.

TS Inter 1st Year Political Science Model Paper Set 7 with Solutions

Question 8.
What is Indian Secularism?
Answer:
It is asserted that India is a secular state. The addition of the word secular to our constitution by 42 amendment in 1976 proves this fact. Even from the beginning of the Indian constitution, India is a secular state. Several provisions of the Indian Constitution also prove this fact.

  1. According to Article 25, every Indian citizen enjoys the freedom to profess, practice and propagate the religion of his own choice. It also allow them to donate their properties for the religious purposes.
  2. Article 26 guarantees every person to:
  • Establish and maintain religious and charitable institutions
  • Manage his (or) her religious affairs
  • Own and acquire movable and immovable properties and
  • Maintain such properties in accordance with law.

3. Article 27 states that state shall not impose any tax upon the individual for the development of religions. It also implies that state. shall not impose taxes on the basis of the religious principles of individuals.

4. Article 28 forbids the imparting of religious teachings in the educational institutions which are wholly partly aided by
the government. No religious prayers or discussions shall be conducted in educational institutions.

Question 9.
Explain the features of Constitutions.
Answer:
Introduction: The age of Democracy led to political civilisation. Now-a-days every civilised state possess a constitution. A Constitution is a condition of modern state. The constitution is a living text of a political system. It represents the political character of the state and its constituents.

The term constitution implies a written document embodying the provisions relating to the powers and functions of the government organs, the rights and duties of the citizens.

Meaning: The term Constitution is an English word. It was
derived from a Latin word ‘Constitution, which means to Establish’ Definitions:

  1. Aristotle: “Constitution is the arrangement of offices in a state, especially the highest of all”.
  2. ord Bryce: “Constitution is a set of established rules embodying and enacting the practice of Government.
  3. Stephen Leacock: “Constitution is the form of Government’.
  4. K.C. Wheare: “Constitution is that body of rulés which regulates the ends for which governmental power is exercised’.

Features of the Constitution:
1. Preamble: Every Constitution will have a preamble. The preamble denotes the aims and aspirations of the Constitution. it is like the soul of the Constitution. Hence, preamble is considered as an important feature of the Constitution.

2. Clarity: Clarity is another important feature of the Constitution. The Constitution clearly explains about the different policies and methods of governance. It is written in a simple and clear language.

3. Incorporation of Fundamental Rights: Every Constitution includes some fundamental rights. These fundamental rights are meant for safeguarding the freedoms of the citizens. They enable the citizens to realise their personality in various spheres. They help the citizens for leading a happy and honorable life in the state.

4. Brevity: Brevity is another feature of a Constitution. Brevity avoids confusion among the individuals in understanding and interpreting provisions. Unnecessary elements are not included in the Constitution. It should be precise. It must not contain large number of clauses.

5. flexibility: The Constitution must be flexible for adapting the wishes are aspirations of the people from time to time. There must be a scope of amending the provisions of the Constitution if necessary. Frequent changes in the Constitution tend to weaken the spirit of the Constitution. But, at the same time, the Constitution of a modern state should be adaptable to the progressive changes.

6. permanence: Permanence is one more feature of the Constitution. The Constitution must have everlasting values for the welfare of the whole nation. It represents the actual structure of the state and its political institutions. it obliges the customs of the people.

7. Mode of Amendment: The Constitution specifies the mode of amendment. It will be relevant to the contemporary conditions of the state. it contains a special chapter on the constitutional amendment procedures. Usually, the constitutional amendments are of three types, namely

  • Rigid
  • Flexible and
  • Half rigid and Half flexible.
  • On the whole, the constitution of every state comprises both rigid and flexible elements.

8. Explanatory: The Constitution is explanatory in nature. It denotes and discusses almost all elements relating to the People, Government and State. It contains separate provisions of the structure, powers and limitations of state activity.

Question 10.
What is Judicial Activism?
Answer:
According to the idea of judicial activism judges should use their powers to correct injustices, especially when the other branches a Government do not act to do so the courts play an active role in shaping social policy on such issues as civil rights, political unfairness protection of individual rights and public morality.

Judicial activism is policymaking function of judiciary in competition with policymaking by the legislative and executive. This element is associated with the doctrine of judicial review. The essence of true judicial activism lies in rendering decisions by the judiciary which are in tune with the temper and tempo of the times.

Behind every judicial decision, judicial activism and judicial restraint are the two aspects that describe the philosophy and motivation. The concept of judicial activism is the polar opposite of judicial restraint. Judicial activism refers to a theory of judgement that takes into account the spirit of the law and the changing times, where as judicial restraint relies on a strict interpretation of the law and the importance of legal precedent.

Judicial activism is dynamic process of judicial outlook in a changing society. Arthur Schlesinger Jr. introduced the term Judicial activism in 1947. According to Black’s Law Dictionary, “Judicial activism is a judicial philosophy which motivates judges to depart from traditional precedents in favour of progressive and new social policies”.

Question 11.
Explain the merits and demerits of Parliamentary form of Government.
Answer:
Merits of Parliamentary Government:
a) Harmony and co-ordination between Legislature and executive: a parliamentary government, it secures cooperation
and coordination from the Legislature because it was formed with the single majority party from Legislature. It enacts laws and implementing laws through taking confidence of legislators.

b) A Scrub on Autocracy: Parliamentary government effectively checks the despotic attitude of the majority party in the lower house of the Legislature. Legislators prevent the government from the making mistakes against public interest through its questioning and a vote of no-confidence in the other motions.

c) Governmentally Able and Experienced: In a parliamentary government, Government should be consisted with the top Leaders of the majority party. Able and experienced people from the party have a hold on the party and government.

d) Responsible Government: In a parliamentary from a government, council of ministers along with prime minister hold the offices and enjoys the powers and acts on the collective responsibility to the Legislature. In a parliamentary system, opposition party controls the ruling party through its vigilance in the outside of the Legislature.

e) Flexible Government: Flexibility is another merit of the parliamentary from of a government, according to Bagehot, under this government; the people can choose a ruler for the occasion who may be especially qualified to successfully pilot the ship of the state through motional crisis.

f) Alternative to Government: Parliamentary system is in the real sense a government by criticism; The majority from the government the majority continues the opposition. The opposition must criticises the, government. The lapses of the government are its opportunities and ruling party lost confidence opposition party is ready to hold the office.

Demerits of the Parliamentary Government:
a) Unstable Government: The government has no fixed life. It remains in office only so long as it can retain parliamentary majority which is sentient to the vagaries of the representative actives.

b) Violation of the theory of separation of powers: In a parliamentary of government, there is a combination of executive and Legislature functions in the same set of individuals lead top while the same men may be at one members of the Legislature and the executive, their functions in the two roles are distinct but in political experience, they worked in no distinction.

c) Executive becomes Autocratic/tyranny of majority: When the executive is confident of support by majority members in the Legislation, at is likely to become autocratic. The opposition feels helpless in correcting the erratic behaviour of the government because all decisions taken on the basis of voting.

d) Unsuitable for emergencies: A national crisis cannot meet with promptness in parliamentary government because much of his time wasted in discussions. Get emergency needs prompt action, while taking decisions, be fear of the opposition and the masses at large.

e) Bureaucratic Dictatorship: a parliamentary government, bureaucracy becomes unduly important. The ministers being amateurs heavily rely on bureaucratise for everything.

f) National Interests Ignored: In a parliamentary government, political parties often ignore the interests of the nation for the sake of interest of the party in power. All national aspects are divided in the interest of the party only.

Despite all their defects, parliamentary government is very popularity is considered more democratic and a true reflection of the public opinion.

TS Inter 1st Year Political Science Model Paper Set 7 with Solutions

Question 12.
Explain three types of Authority.
Answer:
The sociologist and philosopher Max Weber distinguishes three types of Authority – namely:

  1. Charismatic
  2. Traditional and
  3. Legal-rational.

Each of which corresponds to a brand of leadership that is operative in contemporary society
i) Charismatic Authority: The charismatic authority points to an individual who possesses certain traits that make a leader extraordinary. This type of leader is not only capable of but actually possesses the superior power of charisma to rally diverse and conflict prone people behind him. His power comes from the massive trust and almost unbreakable faith people put in him.

ii) Traditional authority: Traditional authority indicates the presence of a dominant personality. This leader is some one who depends on established tradition or order. While this leader is also a dominant personality, the prevailing order in society gives him the mandate to rule. This type of leadership, however, is reflective of everyday routine and conduct.

iii) Legal-rational authority: Legal-rational authority is one that is grounded in clearly defined laws. The obedience of people is not based on the capacity of any leader but on the legitimacy and competence that procedures and laws bestow upon persons in authority Contemporary society depends on this type of rationalization, as the complexities of its problems require the emergence of a bureaucracy that embodies order and systematization.

Section – C
15 × 2 = 30 Marks

Note: Answer any FIFTEEN of the following questions in not exceeding 5 lines each. Each question carries 2 Marks.

Question 1.
Behaviouralism.
Answer:
It is a modern interdisciplinary approach in Political Science. It seeks to make political Science a real Science. It originated in 1925 but became popular in the USA after the second World War David Eason, Robert Dahl, Gabriel Almond are important supporters of this approach. It studies political behaviour of people by using Scientific methods of data collection, It is a protest against the Traditional Approach.

Question 2.
Pluralistic theory of Sovereignty.
Answer:
Pluralism opposes Austin’s theory of sovereignty. It says state ‘sovereignty is not absolute. State is also an Association and the individual is benefited by many Associations. Pluralists want decentralisation of authority and limited functions of the state. Sovereignty is divisible and it is not the source of law. Von Gierke, H.I. Laski, Ernest Barker, Maciver, G.D.H cole, Mait land were the exponents of pluralism.

Question 3.
De-Facto Sovereignty.
Answer:
The French term DeFacto’ implies Real. Defacto sovereign is a person or a body of persons who exercise such authority in the last resort and at the final stage Defacto sovereign may not be a legal sovereign. His authority is based not on law but on physical force. De Facto sovereign may be a king. Dictator or Religious priest.

Question 4.
Mention any two demerits of Nationalism.
Answer:

  1. Nationalism makes the people extremely proud. Jealous and arrogant as was clear from the history of Germany and Italy.
  2. It leads to unnecessary and unhealthy competition among the nations in economic matters.

Question 5.
What do you mean by the term equality?
Answer:
Equity means fairness or justice. It is also a kind of Jude-Made law. It is an informal method of making a new law or altering an old law to new conditions. Sometimes, the courts may be confronted with the disputes about wliich the law is silent. under such situation, the Judge will give relief to the aggrieved party by using principles of social justice and humanism common sense. In course of time, they acquire the status of law. In equity Judge is adding to the law what is missing therein and creating a new one.

Question 6.
What is International Equality’
Answer:
International equality: International equality means that all the states are treated equally irrespective of theIr geographical, economic or military composition. According to this element all nations of the world are equal whether they are large or small. For instance, the United Nations have extended equal dignity and status to all the nations in its charter. International Equality reflects the traits of humanism.

It emphasizes the peaceful settlement of disputes between the nations. Some cautioned about the occurrence of Third World War in the absence of respect to international law. They expressed apprehensions thinking that human beings will go back to the stone ages.

TS Inter 1st Year Political Science Model Paper Set 7 with Solutions

Question 7.
What do you know about Social Justice?
Answer:
Social Justice envisages a balance between rights of individuals and social control. It facilitates the fulfillment of the legitimate expectations of the individuals under the existing laws. It ensures several benefits and extends protection to the individuals against the interference or encroachment from others in society. It is consistent with the unity and the integrity of the nation. It fulfills the needs of the society.

Social Justice enforces the principle of equality before law. It also ensures eradication of social evils like poverty, unemployment, starvation, disease etc. It also extends protection to the downtrodden and weaker sections of society. Ultimately it provides those conditions essential for the allround development of individuals.

Question 8.
Meaning of Socialism.
Answer:
Hughan regarded socialism as the political movement of the working class which aims to abolish exploitation by means of collective ownership and democratic management of the instruments of production and distribution. Some writers regarded socialism as a democratic movement meant for promoting justice and liberty and for managing the society on efficient principles.

Question 9.
Civil – disobedience – National movement.
Answer:
This movement is a landmark in the constitutional history of India. The Indian National Congress Launched this movement on March 12, 1930 under the guidance of Gandhiji. Gandhiji started the civil disobedience movement by taking salt laws for violation. Along with 78 standards supporters Gardhiji began to March towards Dandi, a remote village for about 240 miles from Sabarmati Ashram on March 1930 to 6th April 1930. Gandhiji Planned to violate the salt Laws of the British government by making salt. Hence this movement is also popularly known as salt Satyagraha Movement.

Question 10.
What does the terms Jus Soli mean?
Answer:
Jus Soli means, acquisition of citizenship by the principle of place of birth. According to this method, a child acquires the citizenship of a State, where it borns. It is the place of birth which determines citizenship. This method is not more popular in modern times. At present, this method is observed exclusively in Argentina.

Question 11.
What is initiative?
Answer:
An initiative is a request made by the people to the legislature inframing a law on certain national problem or policy as such. After making the law, the same shall be presented for referendum. In this aspect, people in a specified number present a petition in written form to the legislature proposing a legislation. It is also of two kinds. They are:

  • Formulative initiative
  • Non-formulative initiative.

Question 12.
In what ways does Secularism enables religious freedom to individual?
Answer:
Secularism does not recognize any particular religion as the state religion. Secular state adopts neutral policy in religious matters. It implements various laws and social welfare measures without basing on the religious feelings of the people.

Question 13.
What is an Unwritten Constitution?
Answer:
An unwritten constitution is one whose provisions are not written in a single document. It includes several customs and traditions which are manifested in the form of Laws. The constitution of Britain is the best example of an unwritten constitution.

Question 14.
House of lords.
Answer:
The House of Lords, also known as The House of Peers, is the upper House of the parliament of the United Kingdom. Membership is granted by appointment or else by heredity. Like the House of common, it meets in the palace of West Minister. Currently, there are 300 members in the House of Lords members of whom 240 are Elected members and 60 appointed. Independent members upto 12 Church of England Bishops may sit in the House as ex officio Lords spiritual.
Elected members will serve a single non-renewable term of 15 years.

TS Inter 1st Year Political Science Model Paper Set 7 with Solutions

Question 15.
Parliamentary Executive.
Answer:
A parliamentary System of government means that the Executive branch of government has the direct or indirect support of the parliament. This support is usually shown by a vote of confidence. The Relationship between the executive and the Legislature in a parliamentary system is called Responsible government. According to this system, there is a President who is the formal Head of the State and the Prime Minister and the council of ministers which run the government.

Question 16.
Aristotles classification of Governments.
Answer:
Aristotle classified governments on the basis of two elements, namely:

  • i) Number of rulers
  • ii) Aims of the State.

He again classified Governments into normal and perverted forms. He says monarchy, aristocracy, and polity as the normal form of governments. Tyranny, oligarchy and democracy are the perverted form of Government.

Question 17.
Separation of Powers.
Answer:
Theory of separation of powers is propounded by Montesquieu in his famous book The Spirit of Laws’. The powers among the three organs of the Government in presidential executive will be distributed on the basis of the theory of separation of powers. Its main feature is Checks and Balance’, which means the three organs of the Government possess equal powers and each organ checks the other two organs from crossing their limits.

Question 18.
Prime Minister.
Answer:
Parliamentary government is described as Prime Ministerial government. The Prime Minister in this system acts as the Real executive head of the government. He acts as the Leader of the Majority party or coalition Ministry in the lower house of the legislature. He remains as the main pillar to the structure of union cabinet and unioncouncil of Ministers. He is central to the formation, continuance and survival of the Ministiy He Presides over the meetings of the union cabinet and decides its agenda. He enforces the Principle of Collective Responsibility. All the Ministers ake oath of office assume powers and discharge their Public and political obligations under his stewardship.

Question 19.
What are the views of Plato on Justice?
Answer:
Justice is giving to every man his due. It is a combination of reason, courage, appetite and will in terms of the state.

Question 20.
Equality and Social change.
Answer:
Equality as an idea of Social Change: With advance of scientific knowledge and technology, more and more areas of natural in equality are coming within the alterable sphere. We know that health and bodily strength can be improved by proper nutrition, and mental make up can be considerably developed by proper education and training. But availability of these benefits to an individual is dependent on his socioeconomic status.

TS Inter 1st Year English Model Paper Set 8 with Solutions

Thoroughly reviewing TS Inter 1st Year English Model Papers Set 8 helps in understanding the examiner’s expectations.

TS Inter 1st Year English Model Paper Set 8 with Solutions

Time: 3 Hours 15 Minutes
Maximum Marks: 100

Section – A

Question 1.
Annotate ANY TWO of the following in about 100 words. 2 × 4 = 8M
a) And she was cross. She said go ask the guy who keeps gardening things.
b) It is not the case only with the coals, Mrs. Bouncer, but I’ve lately observed a gradual and steady increase of evaporation among my candles, wood, sugar, and matches.
c) Her intentions are evidently good as she has planted trees rich in biodiversity.
Answer:
a) Introduction : This sentence is taken from the prose piece, Father, Dear Father written by Raj Kinger. Actually this is an article published in the English daily, The Hindu.

Context & Explanation : Rahul has an unpleasant experience with his Biology teacher. When his rose plant is attacked by pests he seeks advice of his teacher to save his plant. But, the teacher gets irritated as she thinks it a question out of their syllabus and asks him to approach a gardener for advice. The teacher serves as a warning to all those teachers who do not show any interest or reverence towards their profession. Therefore, Rahul criticizes such an education system ’ which curbs independent thinking and encourages blind adherence to whatever the teacher teaches.

Critical Comment : Here, Rahul narrates the incident of his biology teacher not able to help him with a practical science related problem.

b) Introduction: We come across these interesting words in Box and Cox, a one-act play written by John Maddison Morton.

Context & Explanation : Mrs. Bouncer a greedy landlady, rents out her room to two persons at the sametime, without letting one know of the other person. They are Mr. Box, the printer and the other man, Mr. Cox. Mr. Box works all night and lives here only during the day. Mr. Cox is employed in a hat shop where he spends all day. Thus, Mrs. Bouncer manages to ensure that they do not meet each other in the room. But, they suspect something is wrong there. They notice their things being used up by others. The given words from Cox complain about this loss of things. It speaks alot about Cox’s nature.

Critical Comment : Here, Mr. Cox addresses these words to Mrs. Bouncer.

c) Introduction : These are the concluding words taken from the internet – based article, The Green Champion-Thimmakka. It describes her magnificent achievements in preserving the environment.

Context & Explanation: Thimmakka and her husband started planting saplings and nurturing them as their own children. Even after the death of her husband, she pursuded her mission with the same determination and courage. She is 100 plus now and still cherishes the dream of planting more trees. She continues her fight against deforestation. Her contributions are truly remarkable. She proves that age is not a big problem if we aspire to do anything. So, she is a true inspiration to us to have good intentions towards society. Please plant a sapling and make the world a better place for our children. Even thousand mile journey begins with a single step.

Critical Comment : The words describe her passion for planting trees and expanding her mission.

TS Inter 1st Year English Model Paper Set 8 with Solutions

Question 2.
Annotate ANY TWO of the following in about 100 words. 2 × 4 = 8M
a) You may succeed with another blow.
b) O my Luve’s like a red, red rose.
That’s newly sprung in June.
c) It is not growing like a tree.
In bulk, doth make Man better be;
Answer:
a) Introduction : This optionistic line is taken four the inspirational poem, ‘Keep Going’, written by Edgar Albert Guest. He was very well-known as a people’s poet. The poem is universally acknowledged as one of the best inspirational poems.

Context & Explanation : This simple inspirational poem speaks volumes about the need to keep going, despite difficulties in life. If openly admits that life may be a mixture of more pains and less pleasures. But, one must continue with one’s effort till success greets one. It is because you may succeed the next time. So, you try again and don’t give up even if you fail many times. If another blow fails, try another and another. But, stop not.

Critical Comment : The poem encourages and inspires the reader to keep on the effort till the goal is attained.

b) Introduction : This couplet is taken from the poem, A Red Red Rose written by Robert Burns. It is one of the best lyrics of English poetry. It blends the eternity of love with the mortality of life.

Context & Explanation : The poet begins by using a simile to com¬pare his love to a rose. In other words, his love is like a flower that has just bloomed in June. His love is fresh and is bursting with life. His feelings are very profound.

Critical Comment : It is an address to the speaker’s lover to whom he swears eternal love and loyalty.

c) Introduction: These are the opening lines of the impressive poem, ‘The Noble Nature’ written by Ben Jonson. He is regarded as the second most popular of English dramatists, after Shakespeare.

Context & Explanation : The poet employs examples from flora to drive home his point. He straight away introduces the main idea how to become a better man. But, mere bulk doesn’t make one great. Smartness, even in small measure, impresses and impacts everyone. Neither long life nor large size can help one attain nobility. Quality counts more than quantity. Motherwords, matter matters, not the magnitude!. To explain this, the poet compares man to both an Oak tree and a Lily.

Critical Comment : The poem seeks to explain what makes Man noble in his life.

Question 3.
Answer ANY TWO of the following questions in about 100 words each. 2 × 4 = 8M
a) How do the two types of persons react to an overcast morning ?
b) Why did Dr. Bannister feel that the moment of the lifetime had come ?
c) “………………. So that I’m getting double rent for my room, and neither of my lodgers is any the wiser for it” says Mrs. Bouncer. Is she right in her estimate of her lodgers ? Support your answer with details.
Answer:
a) Booker T Washington’s addresses are very forceful explanations of character building. In his speech Two Sides of Life he describes how people react to an overcast morning. To a person who constantly looks at the dark side of things in life, the morning appears gloomy, dull and the streets full of muddy water. Everything looks disagreeable to him. Whereas for a person who always looks at the bright side of things in life, the morning appears beautiful in all aspects. He speaks of the beauties in the rain drops, of the freshness in the newly bathed flowers, shrubs and trees.

b) Bannister was the first man to run the race of one mile in 3 minutes 59.4 seconds. In this essay,he narrates his eventual victory of the race. He says that there was no force from anyone to make him participate in this running race. It is entirely his own decision. He passed the half mile in 1 minute 58 seconds. Later, he had to run the last lap in 59 seconds. He felt a moment of mixed joy and anguish. His mind raced, well ahead of his body. It drew his body compellingly forward them, he felt that the moment of life time had come. There was no worry. He felt confidence in his success. It because there was only 200 yards of track under his feet. He realized that he was going to create a record of life time.

c) John Maddison is an English playwright. His play Box and Cox is a one-act farce. It is hilarious. It has just three characters. Mrs. Bouncer is a greedy landlady. She rents out her room to two persons at the same time. The tenants, Box and Cox do not know it. It shows her greediness.

She boasts of her capital idea. She feels that neither of her lodgers finds it. Even though she feels like that she is always in tremble of fear. Initially, she may succeed in deceiving them for a while we can observe it when she gives various excuses when they suspect something is wrong. In order to escape from their doubts, she gets busy to put things out of their notice. That is why they fail to know her deceptive nature. Later, they come to know her deceitful dealings. Thus, her estimate of her lodgers is not completely right.

TS Inter 1st Year English Model Paper Set 8 with Solutions

Question 4.
Answer ANY TWO of the following questions in about 100 words each. 2 × 4 = 8M
a) Explain the narrator’s experience in finding out what happiness is.
b) How does the poem, ‘The Beggar’ describe the farmer’s pathetic physical condition?
c) If may be near when it seems afar; what seems after and why ?
Answer:
a) The poem ‘Happiness’ is written by Carl Sandburg. It conveys a beautiful message. It is extracted from his collection of poems. Chicago songs. The poem is an expression of the narrator’s search for the meaning of happiness and his ultimate realization.

The narrator seeks to know what happiness is. He enquires with many professors but in vain. Even, the top executives are consulted, but to no avail. One Sunday afternoon, he wanders along a river. There, he sees a group of Hungarians with their women and children under the trees. They are spending happy moments there. He at once understands what happiness is. Happiness is living in the present. It is not wealth or success or fame.

b) The poem, The Beggar, by Dr. Ammangi Venugopal portrays the pathetic condition of farmers. It depicts the difficulties farmers face in a touching way. The poet talks about, the farmer as ‘my farmer’. It shows that the poet also belongs to the family of a farmer. So, he describes the pitiable physical position of farmers. It forms an important part of the poem. It identifies farmers with dark eyes that are filled with the shadows of their struggles and sufferings. Their backs are bent with burden. Their hands are soiled and severed. Their feet bleed. Yet, their ability to produce food and satisfy other’s hunger remains fully active. They work hard and help others. Yet, they struggle to survive. They starve. Their stomachs get no food. They suffer from empty stomachs. Their faces are filled with wretchedness. Thus the reader is forced to understand and sympathise with farmers.

c) Edgar Albert Guest is very popular as a people’s poet. His poem, Keep Going, keeps on advising the reader never to quit. It encourages the reader to keep on the struggle till the goal is attained. Sometimes, a goal situated near may appear far often when eyes are tired because of exhaustion. You may think that you are not going to succeed, yet you are close to success. Therefore, you must continue with your efforts till success greets you. Life is a fight. It will often present you with pain or hardships. You may be hit with many challenges. You should not lose the fighting spirit. Don’t quit and go through your hardships. Success is yours. Sure ! thus, the poem inspires us to acheive our goals.

Question 5.
Answer ANY TWO of the following questions in about 100 words each. 2 × 4 = 8M
a) Write a paragraph on how Alan and his parents felt excited when he was chosen to play for the school cricket match.
b) “The years have taught you wisdom …. surely it must be so”, remarked the fairy. Is she right ? Explain.
c) Describe the result of the declaration by the “Mohathemeem”.
Answer:
a) Arthur Henry Mee is famous as an eminent educator and journalist. His short story “Playing the Game” is at once didactic and entertaining. Its gripping narration offers a pleasant reading experience. Alan is the lead character. He was a schoolboy. He loved cricket. His parents supported and encouraged him. Alan’s father actually helped Alan practise bowling. He commented that Alan was shaping as a good bowler. Then, Alan was selected to play in his school team. That was a well deserved opportunity. Hence Alan felt excited. That made Alan’s parents doubly excited. That is just natural and justifiable on their part.

b) The short story “The Five Boons of Life” is a bundle of boons in learning. Mark Twain shows us how difficult it is to select from among options. The story also exposes the deceptive nature of appearances. The fairy presents to the youth her five boons. They are : “Fame, Love, Riches, Pleasures and Death”. She asks him to be careful in his selection. She adds that only one of them is valuable. The youth falters and chooses ‘Pleasures’. He regrets his choice. Then he opts for ‘Love’. He feels sad about his wrong decision. Then, the fairy says that years must have taught him wisdom. Yes, experience is the best teacher. But the youth stays a bad learner. So, he hasn’t picked up any wisdom.

c) Suravaram’s social story, “Sanghala Panthulu”, presents the pathetic plight of Ramasagaram villagers. Elanaaga translated this moving Telugu story into English. The police went on exploiting the innocent villagers ruthlessly. Sandhala Panthulu came to the rescue of the poor. The police were angry with Panthulu. When they tried to arrest Panthulu, a good number of youth revolted against the police. The police complained against them. The Mohathemeem came to enquire into the incident. He found the police were guilty. He declared the dismissal, suspension and scaling down of different police personnel. The villagers felt happy. Their joy knew no bounds. Feasts followed, justice prevailed.

TS Inter 1st Year English Model Paper Set 8 with Solutions

Section – B

Question 6.
Read the following passage carefully and answer ANY FOUR questions given after it in a word or a sentence each. 4 × 1 = 4M
The fairy came, bringing again four of the gifts, but Death was wanting. She said:
T gave it to a mother’s pet, a little child. It was innocent, but trusted me, asking me to choose for it. You did not ask me to choose.”
“Oh, miserable me ! What is left for me ?”
“What not even you have deserved: the wanton insult of Old Age.”

Questions:
i) The fairy brought the gift, Death too. Say Yes or No.
ii) Who does the word I refer to ? ,
iii) What did the fairy give little child ?
iv) What is the epithet used to describe the little child ?
v) The man didn’t repose faith in the fairy. Write true or false.
vi) What was left for the man ?
Answer:
i) No
ii) The word I refers to the fairy.
iii) Death
iv) a mother’s pet
v) true
vi) not even what he deserved; the wanton insult of old age

Question 7.
Read the following passage carefully and answer ANY FOUR questions given after it in a word or a sentence each. 4 × 1 = 4M
A woman is a full circle. Within her is the power to create, nurture and transform, goes a glorious compliment, highlighting woman’s power. Then, if many women are at the helm of a village, can you imagine the degree of transformation ? A lively example is unfolding itself at Madhavaram village of Suryapet district. With the initiative from the sons of the soil like Sri Koti Reddy, Superintendent of Police, Ram Sudheer, School Assistant (Teacher), the entire village Panchayat was unanimously occupied by an all-women team.

Besides, women’s committees were formed for each important village development activity like education, health, drinking water and sanitation. Ms Vijayafakshmi, Sarpanch and Ms Janakamma, Vice-Sarpanch were all smiles when asked about their achievements in a short span of time. The long list includes a library, purified water, a gymnasium, English Medium Sections in ZP School, greenery etc. Their honesty in admitting that their move towards prohibition of liquor still awaits results stuns everyone. The village serves as a model, showcasing women’s power.

Questions:
i) Frailty, thy name is woman, says Shakespeare. But, what does this passage say in this regard ? (Frailty means weakness.)
ii) Mention the unique feature of Madhavaram’s present panchayat.
iii) What do other all-women committees have for their function ?
iv) Why were the Sarpanch and Vice-Sarpanch all smiles ?
v) Which particular area still awaits satisfactory progress ?
vi) What does the village seek to showcase ?
Answer:
i) power, thy name is woman.
ii) It is occupied by an all-women team.
iii) education, health, drinking water, sanitation, etc.
iv) as they recorded many achievements in a short span of time
v) the prohibition of liquor
vi) women’s power

Section – C

[Note : Answers of this section must be written at one place in the same Serial Order.]

Question 8.
Match ANY EIGHT of the following words in Column – A with their meanings in Column – B. 8 × 1/2 = 4M

Column A Column B
i) resistance a) silence
ii) pounce b) suffering, agony
iii) flutter c) surround, cover
iv) anguish d) natural, on the spot
v) haven e) dared
vi) spontaneous f) refusal to obey; opposition
vii) lull g) confuse, puzzle
viii) ventured h) move lightly and quickly, tremble
ix) engulf i) safe place
x) bewilder j) jump, leap

Answer:

Column A Column B
i) resistance f) refusal to obey; opposition
ii) pounce j) jump, leap
iii) flutter h) move lightly and quickly, tremble
iv) anguish b) suffering, agony
v) haven i) safe place
vi) spontaneous d) natural, on the spot
vii) lull a) silence
viii) ventured e) dared
ix) engulf c) surround, cover
x) bewilder g) confuse, puzzle

TS Inter 1st Year English Model Paper Set 8 with Solutions

Question 9.
Identify the parts of speech of ANY EIGHT of the following underlined words. 8 × 1/2 = 4M
Box : Stop ! (1) Can you (2) inform (3) me who (4) the individual (5) is that I invariably (6) encounter (7) going downstairs when I’m coming up (8), and (9) coming upstairs (10) when I’m going down ?
Answer:
1) stop – verb
2) you – pronoun
3) inform – verb
4) who – pronoun
5) individual – noun
6) invariably – adverb
7) encounter – verb
8) up – adverb
9) and – conjunction
10) upstairs – adverb

Question 10.
Fill ANY EIGHT of the following blanks with a, an or the. 8 × 1/2 = 4M
It is not _____ (1) _____ case only with _____ (2) _____ coals, Mrs. Bouncer, but I’ve lately observed _____ (3) _____ gradual and steady increase of evaporation among my candles, wood, sugar and matches.

Why _____ (4) _____ gentleman who has got _____ (5) _____ attic is hardly ever without _____ (6) _____ pipe in his mouth and there he sits with his feet upon _____ (7) _____ mantelpiece. From _____ (8) _____ appearance of his outward man, I should unhesitatingly set him down as _____ (9) _____ gentleman connected with _____ (10) _____ printing interest.
Answer:
1) the
2) the
3) a
4) the
5) the
6) a
7) the
8) the
9) a
10) the

Question 11.
Fill in ANY EIGHT of the following blanks with suitable prepositions. 8 × 1/2 = 4M
_____ (1) _____ one and a half laps I was still worrying _____ (2) _____ the pace. Advoice shouting ‘Relax’ penetrated _____ (3) _____ me _____ (4) _____ the noise _____ (5) the crowd.

As we lined up_____ (6) _____ the start I glanced _____ (7) _____ ts he flag again. It fluttered more gently now, and the scene _____ (8) _____ Shaw’s Saint Joan flashed _____ (9) _____ my mind, how she, _____ (10) _____ her desperate moment, waited _____ (11) _____ the wind to change.
Answer:
1) at
2) about
3) into
4) above
5) of
6) for
7) at
8) from
9) through
10) at
11) for

Question 12.
Fill ANY FOUR of the following blanks with suitable forms of the verbs given in brackets. 4 × 1 = 4M
1. People ……………….. (speak) Telugu in Telangana and Andhra Pradesh.
2. Mary ……………….. (eat) her supper by 7.00 p.m.
3. Don’t disturb ! The child ……………….. (sleep)
4. If I ……………….. (be) a bird, I would fly high in the sky to have a beautiful view of the earth.
5. It is time we ……………….. (start) working hard.
6. If I were you, ……………….. (construct) an independent house.
Answer:
1) speak
2) will have eaten
3) is sleeping
4) were
5) started
6) would construct

TS Inter 1st Year English Model Paper Set 8 with Solutions

Question 13.
Rewrite ANY FOUR of the following sentences as directed. 4 × 1 = 4M
i) He is buying a TV set at the moment.
(Change the sentence to passive voice.)
ii) Hardik said to Annu, “Go and study.”
(Change the sentence to indirect speech.)
iii) A visitor said to me, “Are there any places worth seeing in Wa- rangal ?”
(Change the sentence to indirect speech.)
iv) For many Indians, cricket gives greater pleasure than football.
(Change the sentence to positive degree.)
v) LIC is one of the most popular insurance companies in India.
(Change the sentence to comparative degree.)
vi) There’s a problem here, …………….. ?
(Add a question tag.)
Answer:
i) A TV set is being bought by him at the moment.
ii) Hardik asked Annu to go and study.
iii) A visitor asked me if there were any worth seeing places in Warangal.
iv) For many Indians, football doesn’t give as much pleasure as cricket.
v) LIC is more popular than many other insurance companies in India.
vi) isn’t there

Question 14.
Rewrite ANY FOUR of the following sentences correcting the underlined errors. 4 × 1 = 4M
1. If I will Stand on my own legs, my parents will feel happy.
2. If you consult me. I would have advised you what to do.
3. He is visiting the library daily.
4. He walks very fastly.
5. We don’t hardly believe it.
6. They don’t do anything careful.
Answer:
1. If I stand on my own legs, my parents will feel happy.
2. If you had consulted me, I would have advised you what to do.
3. He visits the library daily.
4. He walks very fast.
5. We don’t believe it./We hardly believe it.
6. They don’t do anything careful.

Question 15.
Supply the missing letters to ANY EIGHT of the following words. 8 × 1/2 = 4M
i) pers_ _de
ii) flu_ _er
iii) p_ _nee
iv) ex_ _ode
v) a_ _empt
vi) br_ _th
vii) hu_ _red
viii) pa_ _ive
ix) co_ _apse
x) thr_ _ten
Answer:
i) persuade
ii) flutter
iii) pounce
iv) explode
v) attempt
vi) breath
vii) hundred
viii) passive
ix) collapse
x) threaten

TS Inter 1st Year English Model Paper Set 8 with Solutions

Question 16.
Identify the silent consonant letters in ANY EIGHT of the following words. 8 × 1/2 = 4M
i) bustle
ii) although
iii) parliament
iv) fight
v) knee
vi) brought
vii) bomb
viii) could
ix) hymn
x) which
Answer:
i) bustle – t
ii) although – gh
iii) parliament – r, i
iv) fight – gh
v) knee – k
vi) brought – gh
vii) bomb – b
viii) could – l
ix) hymn – n
x) which – h

Question 17.
Write ANY FOUR of the following transcriptions using ordinary English spelling. 4 × 1 = 4M
TS Inter 1st Year English Model Paper Set 8 with Solutions 1
Answer:
i) acquaint
ii) attic
iii) horizontal
iv) gridiron
v) curb
vi) vengeance

Question 18.
Circle ANY FOUR of the words that sound different from the other words in that set with regard to the sounds of the bold letters. 4 × 1 = 4M
i) deer feet street
ii) barely money rely
iii) look book door
iv) ant above apple
v) one orange ox
vi) late bat make
Answer:
i) deer
ii) rely
iii) door
iv) above
v) one
vi) bat

Question 19.
Mention the number of syllables in ANY FOUR of the following words. 4 × 1 = 4M
i) popular
ii) today
iii) side
iv) plant
v) rainwater
vi) condition
Answer:
i) 3 – trisyllabic
ii) 2 – disyllabic
iii) 1 – monosyllabic
iv) 1 – monosyllabic
v) 3 – trisyllabic
vi) 3 – trisyllabic

TS Inter 1st Year English Model Paper Set 8 with Solutions

Question 20.
a) The pie chart given below shows how people spend their time on smart phones. Convert the information into a paragraph. 1 × 4 = 4M
TS Inter 1st Year English Model Paper Set 8 with Solutions 2
b) The following tree diagram depicts the classification of Vitamins. Present the information in a paragraph.
TS Inter 1st Year English Model Paper Set 8 with Solutions 3
Answer:
a) Time spent on Smart Phones
Time spent on smart phones is presented in the given pie chart. The lion’s share, i.e. 35% of the time goes to games. Social networking follows games with its share of 29% of the time. Utilities consume 20% time. The share of music and videos is 8%. Others take 5% time. News comes last with just 3% time.
(Or)
b) Classification of Vitamins
The given tree diagram presents the classification of vitamins. Vitamins are broadly of two types. They are : 1) Soluble vitamins in water and 2) Soluble in fats. Vitamin B and Vitamin C fall in the category of ‘Soluble in water’. Vitamins A, D, E and K (four) belong to the group of vitamins soluble in fat and Vitamin B is sub-divided into Bl, B2, B3, B6 and B12 (five) types.

TS Inter 1st Year Political Science Model Paper Set 6 with Solutions

Thoroughly analyzing TS Inter 1st Year Political Science Model Papers Set 6 with Solutions helps students identify their strengths and weaknesses.

TS Inter 1st Year Political Science Model Paper Set 6 with Solutions

Time: 3 Hours
Max. Marks:100

Section – A
3 x 10 = 30 Marks

Note: Answer any THREE of the following questions In not exceeding 40 lines each. Each question carries 10 Marks.

Question 1.
Discuss the relationship of Political Science with History and Economics.
Answer:
Political Science has intimate relation with other social sciences like History and Economics. Such inter-relation between political science and History as well as relation between political science and Economics can be detailed as below.
a. Political Science – History
b. Political Science – Economics.

a) Political Science – History: History describes the past. The development of mankind and society can be known through History. History being the story of man, functions as a treasure house of human experiences. It is like a laboratory to all social sciences. The political, economic, social, cultural, religious and literary activity of man can be known only through History It describes different associations of man from earliest times. History conveys information to the present society, the developments in the past in the areas like state, civilization, culture, religion and economic activity. History is a written record of different events, movements, their causes and interrelations.

History provides information to study the political activity in the past. The birth and development of political ideas and institutions is known through history.

“History without Political Science has no fruit.
Political Science without History has no root”

There has been continuous transformation and development of political institutions since the earliest period of History. The evolution of different political institutions through the ages is recorded in History History is the foundation of Political Science. A comparative study of the previous political institutions and the contemporary political activity provides a scope to find ideal and stable political institutions in future.

The knowledge of political activity is very much essential to understand the events like founding of the Indian National congress, the French Revolution, the Russian Revolution and the theories like the twö Nation Theory of the Muslim League and also to estimate their impact.

In the same way it is possible to study the concepts proposed by Plato, Aristotle, and other Philosophers in the light of the knowledge of history of Ancient Europe. Different political thinkers like Machiavelli, Montesquieu, and Lord Bryce developed their respective theories basing on the information found in history As Robbon opined, it is essential for a student to know about the history of his own race to study the constitution and foreign policy of his race.

The knowledge of Political Science is essential to history, in the same way as the knowledge of History to Political Science. History and Political Science can contribute for the development of any Civilised society in the spirit of mutual co-operation.

b) Political Science – Economics: Economics ’studies the aspects Like wealth, production, distribution and exchange of goods. It studies about various methods to accumulate wealth. Economics is a sociological study of the aspects like wealth, production and distribution. All the social institutions nd political theories place the human life on a right track. A clearly defined political system is very essential for a man to become a good and ideal citizen.

Economics helps in different ways to study the human welfare. Economics tries to coordinate the methods of satisfying unlimited wants with limited resources. Lack of peace and dissatisfaction prevail in a society when the economic needs are not satisfied.

The Primary needs like food, clothing, shelter, education, and medical aid are to be satisfied. Other wise life becomes sorrowful. If the basic needs are not satisfied, the individual has to spend all his energy for that purpose only. A poverty-stricken society gives scope for the prevalence of immorality and anti-social elements.

An individual suffering from hunger, ignorance, ill-health cannot be in a position to assess his political aims and responsibilities. He resorts to many crimes to satisfy his hunger. Such individual entertains a spirit to adopt illegal means for his progress. He cannot use his rights properly and discharge his duties. A citizen without basic needs cannot understand the value of right to vote. The communists feel that democracy cannot be successful without a socialist economy. Aristotle warned that economic inequalities lead to social revolutions.

Even though Political Science and Economics are two different disciplines, their common aim is the welfare of people. The policies related to the production, consumption, proper use of exchange units, removing inflation: contributing for the accumulation of national wealth, promotion of industrial development are very much a part of the activities of a modern State. The impact of economic policies is very much considerable on all the above policies. it is quite possible to solve many economic problems through a political system only.

TS Inter 1st Year Political Science Model Paper Set 6 with Solutions

Question 2.
Discuss pluralistic theory of Sovereignty.
Answer:
Pluralism or pluralistic theory of sovereignty was proposed and popularized as an attack on monism. Elements like democracy, federal spirit, individual freedoms, separation of powers, decentralization of authority etc. are considered as the factors which influenced the proponents of pluralism. This theory finds its origin during the middle age.

Main Principles: Pluralism comprises the following main principles:
1. The state is one of the many organizations in society. It is not the only organization having supremacy. It has no differ
ent features, status and significance apart from other organizations.

2. The state has not created the society or any organization, The various cultural, economic, religious and political associations were not created by the state. State cannot dissolve the above organizations. It has no such power.

3. Society is federal in structure. State too must distribute its powers on the basis of federal principles.

4. Sovereignty is not the exclusive possession of the state. The state is not supreme over all other organizations.

5. Sovereignty is not absolute, unlimited and unquestionable. It has some internal and external limitations. While the customs and traditions of the people will act as internal limitations, international covenants and agreements will impose some restrictions on the sovereignty of the state.

6. Sovereignty is not discretionary in nature. It has to be enforced keeping in view the various constitutional acts and
covenants. It will not vest in the state alone.

7. It is not correct to say that. the commands of the sovereign are laws. Sovereignty has to be exercised keeping in view the customs, traditions and the constitutional provisions.

8. Pluralists criticized that Austin’s theory assigned more importance to the state. This may threaten the very origin, existence, survival and progress of the remaining associations in human society Hence Krabbe and laski pointed out that it is better to ignore the criticism of sovereignty.

Criticism:
1. The pluralist’s contention that sovereignty is possessed by all associations along with the state has been condemned by the critics. The critics expressed their apprehension stating that the pluralist’s ideas may lead to the creation of anarchy and instability in the state.

2. The pluralist’s assumption of similarity of state and other associations in regard to their aims objectives, and aspirations is not appropriate.

3. The pluralist failed to identity the distinction between the community, society, and state. There prevails more similarity rather than differences between the state and society. Both the state and society are independent in their sphere. They are more heterogeneous in their nature.

4. The critics viewed pluralism as unreal and impractical. They felt that it is not possible to achieve coordination between
the state and associations.

Conclusion: Inspite of the above criticism, pluralistic theory of sovereignty is recognized as very important in political science. Similarly, the concept of state and Monistic theory of sovereignty have acquired special place in political science.

Question 3.
What is Equality? Explain its characteristic features.
Answer:
Meaning and explanation of Equality: The concept of Equality is of great significance in the study of political science. The term Equality became an important slogan and inspired the people of France, America, Russia and India during their struggle for freedom and independence.

The term “Equality” implies absolute equality of treatment. In Political science, the term ‘Equalitÿ refers to a state which
grants its citizens equality before the law and equal opportunities to develop their personality But it may be noted that individuals are not equal in many respects. While some of them are strong, some others may be weak. Similarly some are more intelligent than others. In this way, men differ in many respects. Hence, equality of treatment is not possible. In other words, it implies that state should grant to its citizens equality before law and equal protection by law.

Essential Features of Equality:
1. Equality as a Claim of Right: Equality is a prescriptive term, not a descriptive one. We argue that human beings must be treated as equal, not that they are in fact equal. Equality aims at widening the base of social benefits lest these benefits are cornered by a small and vocal minority impoverlshing the rest of the community.

2. Equality as a Modern Idea: Large inequalities of wealth, prestige and power have always remained prominent and almost universal features of social structure throughout human history With beginning of scientific age, men learned to control natural phenomenon by rational and empirical knowledge of nature. This paved the way for removing such social inequalities as were not reasonable and which were also removable by human effort.

3. Equality as an Idea of Social Change: With advance of scientific knowledge and technology, more and more areas of natural inequality are coining within the alterable sphere. We know that health and bodily strength can be improved by proper nutrition, and mental up can be considerably developed by proper education and training. But the availability of these benefits to an individual is dependent on his socioeconomic status.

4. Essential for Social Justice and Liberty: Equality is essential for social justice. Equality is closely connected with liberty. This is due to the fact that without liberty people cannot have equality. Liberty remains insignificant in the absence of equality.

5. Social distinctions can be based only upon public utility: Unequal treatment would not be resented in society unless the more gifted persons employ their natural qualities to exploit others. And also, so long as authority and division of labour are based on rational grounds, inequality of status and position does not become objectionable. Inequality is not an issue so long as authority is exercised in the general interest of the society.

6. Equality does not imply literal Equality: It means equality of opportunities. Equality demands a progressive reduction of inequalities where they are thought to be unreasonable. It does not imply literal equalization. It implies giving equality of opportunities for the development of personal qualities and capacities. It does not mean ‘equality of outcome’.

TS Inter 1st Year Political Science Model Paper Set 6 with Solutions

Question 4.
Write an essay on basic ideas of Gandhism.
Answer:
Gandhism – basic ideas:
Gandhi – A brief biography: Mohandas Karamchand Gandhi was born at Porbandar in Gujarat in 1869. He was called the Mahatma. (The Great Soul) by Rabindranath Tagore. He was the father of our nation. He applied age-old ancient Indian ideals like Truth, Non-violence, and Satyagraha as political weapons and won the freedom not by late but by loving the enemy. He preached only whatever he practiced.

Basic ideas of Gandhism:
1. Meta Physical idealism: The Upanishadic concepts like ‘The Divine”. The universal soul, manifested in all living and non-living things of the entire universe, or ‘The Divine light illuminating everywhere” are the basis for the Gandhian Philosophy. His metaphysical idealism was a unique combination of the values based on Non-violence, ethics, Vedanta, Spiritual, Meta Physical, Jam, Buddhist and Vaishnava.

2. Ethical absolutism: Gandhi believes the superiority of moral and ethical values. The roots of Ns ethical absolution can be traced in the “Rita’ of the Vedanta. This Rita is universal, omnipresent, and ethical in values is ruling the Men and the Gods.

3. Doctrine of Non-Violence: The literal meaning of Non-violence is “not doing violence. In a Nutshell it means “Not to kill”. “Not to do harm is its wider meaning.

He applied nonviolence as a means and a weapon in politics. Truth and fearlessness are the essential conditions of Non-violence. Gandhi regarded and equated non-violence on par with self-torture of the Soul, Mercy, Love, Fearlessness, innocence, Soul Force, kindness, Selflessness and non-indulgence. Gandhi used non-violence as a potent weapon not only against the British colonialism but also in the movements waged against all types of deeply entrenched evils of the society.

4. Doctrine of Satyagraha: Gandhi explained Satyagraha not as a philosophical doctrine, but as a means to fight against the foreign rule and to achieve social and economic justice.

Gandhi formulated the word satyagraha when he was in South Africa. He called satyagraha as Love Force” and ‘Sóul Force’. Truth cannot tolerate violence. Even the guilty should not be punished with violence. A sin for one may not be to the other. At once, the search for truth must be only on non-violent means. We have to try to remove the holds on untruth and injustice from his ways by inflicting suffering upon himself. By satyagraha means, Gandhi said that inflicting suffering not on the evil-doer but upon himself.

Question 5.
Identify the safeguards of Rights.
Answer:
Introduction: Rights are the essential conditions for the development of the personality of individuals. They are upheld by the laws of the state. Individuals cannot achieve progress in the absence of the lights.

Definition:

  1. T.H. Green: “Rights are those powers claimed and recognized as contributory to the common good.”
  2. H.J. Laski: “Rights are those conditions of social life without which no man can seek in general to be himself at his best.”

Safeguards of Rights: Individuals enjoy their rights only when they were fully protected or safeguarded by the State. In this regard, the following elements act as the safeguards of the rights.
1. DemocratIc Rule: Democratic rule safeguards the rights of the people to a great extent. People can enjoy their rights perfectly in democratic states only. This system makes constitutional and legal provisions for safeguarding the right of the people.

2. Written and Rigid Constitution: A written constitution clearly defines the powers and functions of the government. It also explains about the various limitations of governmental authority. Besides, a rigid constitution will guarantee the rights of the people by making it difficult for the rulers and legislators to make amendments on flimsy grounds.

3. Constitutional Incorporation: The incorporation of fundamental rights in the constitution will prevent the encroachment of individual rights by the government. Such an arrangement protects the rights of the individuals to a great extent.

4. Separation of Powers: The powers of the government should be separated among the three organs of the government. Such as measure would act as a check against other organs. Ultimately, it serves as a safeguard of individual liberty.

5. Decentralisation of Powers: Individuals enjoy their rights, when powers are decentralised among the governmental institutions. This involves allocation of powers at various levels – national, provisional, local either on functional or territorial basis.

6. Rule of Law: Rule of law implies equality before law. It also denotes equal application of laws to the citizens. It gives no scope for discrimination between citizens on the grounds of region, religion, caste, colour, community etc.

7. Independent and Impartial Judiciary: Independent and impartial judiciary is another safeguard of rights. Judges in higher judicial bodies will deliver judgement with impartial and independent outlook. In the process of delivering justice, they issue certain writs for immediate protection of the rights.

8. Independent Press: Independent and honest press is another essential safeguard of rights of individuals. Such agency will be able to disseminate news and views impartially and without fear or favour to anybody. In this regard the state should not try to threaten and silence the press. Then only individuals enjoy their rights to the maximum extent.

9. Social and Economic Equalities: Social and economic equalities are necessary for enjoying one’s rights. People will be able to utilize their rights properly and positively when there are social and economic equalities in the state. These equalities include absence of casteism, communalism, linguism, widespread economic inequalities, exploitation, etc.

10. External Vigilance: Eternal vigilance is said to be the most important safeguard of rights of individuals. Individuals must be vigilant and cautious about the policies of the government. They should oppose the despotic tendencies of the government through democratic and constitutional methods. Under no circumstances they should allow the self-seeking politicians to acquire power. Besides several other elements like judicial review, recall, strong opposition etc., are considered as the safeguards of rights.

Seciton – B
8 x 5 = 4o Marks

Note: Answer any EIGHT of the following questions in not exceeding 20 lines each. Each question carries 5 Marks.

Question 1.
Write a brief note on the Evolution of Political Science.
Answer:
The beginning of a study of politics (or political thinking) is traced to the ancient Greek philosophers like Socrates, Plato and Aristotle. Among these, Aristotle is regarded as the father of Political Science because of his objective and systematic (scientific) study of the affairs of the government and politics. Aristotle used the term ‘politics’ to designate the science of the state. He called ‘politics’ a Master Science’ as it covered almost all the activities of individuals in society that includes political and social institutions In fact, politics was a comprehensive concept for the Greeks. However, the Greek concept of politics underwent changes over the centuries.

In other words, the Greek concept of politics is no more valid today. The study of politics has acquired a wider meaning in the modern times. As a matter of fact, with the evolution of the systems of production through different stages (e.g., hunters and food-gatherers, to agriculture and then to industry/factory) social and economic institutions become important, and so also political organizations, evolved and progressed.

This resulted in a distinction between three spheres of human activities, namely, ‘politics’, ‘social’ and ‘economics’. Hence forth, Political Science came to be defined as ‘science of state and government. Thus, the detailed study of the evolution of the State, its functions and the government constituted the subject matter of the study of Political Science.

With the influence of the study of Behaviourism in natural Sciences, the behavioural Movement has been iñitiated in Social Sciences. After the Second world war the behavioural movement which had emerged in 1920’s, became very popular movement in 1950’s. This movement was led by American Political Scientists like Charles Marrium, Gabriel Almond, David Easton etc.

Behaviouralism emphasises on the study of political behaviour, and perceptions of the individuals towards their own political system. Hence the study of political science confined itself to the study of political behaviour in an organised society. The study area of political science also extends to policy formulation, implementation and evaluation of the political systems. Hence, the political science is also called as policy science. The concept of policy science was propounded by J. Lasswell.

TS Inter 1st Year Political Science Model Paper Set 6 with Solutions

Question 2.
Explain the differences between State and Government.
Answer:
Differences between State and Government:

State Government
1. State is broader. It consists of all the people. 1. Government is narrower. It is a body of few citizens.
2. State is permanent. 2. But Government is transitory. Government keeps on changing. For ex: In India NDA at the national level has been replaced by the UPA.
3. State has sovereign power. 3. Government does not have sovereignty. But, Governments exercise the sovereign power in the name of state.
4. All states are universal and similar in nature and characteristics. 4. Government differs from state to state depending upon the wishes of the people or the constitutions of the respective states. For ex : Parliamentary, Presidential Govt etc.
5. Loyalty of the citizens to their state is compulsory. 5. In a democracy, people have the right to critizen the acts of omission and commission of the Government.
6. State is a whole. it consists of four essential elements like population, territory, government and sovereignty. 6. Government is one of the four elements of the state. Hence, it is the part and parcel of the state.
7. The state is master; it can place and replace the governments according to their efficiency and popularity. 7. The status of the government in relation to the state is that of a master and servant. The survival of the Government depends on the pleasure of the state.
8. Membership of the State is Compulsory No one is exempted from its membership. 8. Membership of the Government is not compulsory ‘it depends upon the will of the person concerned to become the member of the Government or not.

Question 3.
Describe any three types of Justice.
Answer:
1. Natural Justice: Natural Justice is based on the notion that every person in the world possesses some rights for availing the natural resources. Natural resources provide support to the life of each and every creature on Earth. As the human beings are the only rational creatures, it is their responsibility to see that natural resources have to be judiciously exploited. Human beings must keep in mind the requirements of future generations in this regard.

2. Social Justice: Social Justice envisages a balance between rights of individuals and social control. It facilitates the fulfillment of the legitimate expectations of the individuals under the existing laws. It ensures several benefits and extends protection to the individuals against the interference or encroachment from others in society. It is consistent with the unity and the integrity of the nation. It fulfills the needs of the society Social Justice enforces the principle of equality before law. It also ensures eradication of social evils like poverty unemployment, starvation, disease etc. It also extends protection to the downtrodden and weaker sections of society. Ultimately it provides those conditions essential for the all-round development of individuals.

3. Political Justice: Political Justice symbolises political equality It implies provision of political rights to all the adult citizens in a state. It facilitates free and fair participation of the citizens in the governance of the country. It is manifested to the full extent in times of elections. It allows the citizens for their active participation in day-to-day administration. It is based on the premise that everyone is counted as one and none for more than one. It may be noted that political justice prevails in the State when the following conditions are prevalent:

  1. Rule of law
  2. Independent Judiclaiy
  3. Popular elections to the representative bodies.
  4. Political parties.
  5. Freedom of press and assembly
  6. Democratic rule etc.

Question 4.
Gandhiji’s views on Religion and Politics.
Answer:
The moral concepts of Gandhi can be founded in his ideas expressed very frequently. Politics without morals character create a degenerated state and Government in a human society indulged in material pleasures. All the political means are used only to gratify power. But Gandhi described the theory of authority. He says that both authority and ethics should become the focal points of politics.

He mixed humanistic, political religious and ethical values in politics. He opposed the segregation of religion from politics. In his opinion both of there are equally essential. Politics without religion is like a dead corpse, not useful to a country except to burn. He says that his search for truth pulled him into politics and his moral strength helped him to remain very firm in politics.

Even a political programme is intended for the social and ethical advancement of the people. We cannot segregate politics from life, most importantly from religion. Segregation means nurturing religious fundamentalism’, bigotry and evil in politics. Religion according Gandhi was not Rituels and blind faith, but a coordinated moral values of all religions. A religion should not promote sectarianism. But Gandhi never supported a state religion. He wants to use the religion to oppose all types of evils in the society.

TS Inter 1st Year Political Science Model Paper Set 6 with Solutions

Question 5.
Discuss the various types of Duties.
Answer:
Duty is an obligation of an individual towards. Other individuals residing in the Society. It is regarded as an obligation or duty towards others. The term Duty’ denotes what one is bound to do. Every individual must abide by certain rules of behaviour in society for his own good and for the good of others. These include some do’s and don’ts. Duties are both positive and negative in nature.

Everyone in society must perform these duties in the larger interests of society and state. Everyone must behave in such a way that promotes common good and social welfare: Duties in turn contribute to the public good. They establish peace and order in society. Duties always precede rights.

Types of Duties: Duties are broadly of two types :
Moral and Legal.
i. Moral Duties: Moral Duties are those which bound the individuals together on moral grounds. They may not be upheld and supported by the laws of the state. They are based on the moral beliefs of the people. They are sanctioned by the community basing on some customs, traditions and usages. Any violation of moral Duties does not lead to punishment. Helping the needy and the sick is regarded as an example of moral Duties.

ii. Legal Duties: Legal Duties are implemented through the courts and with the support of the statutory laws. They carry statutory significance. They are very clear and precise. They are compulsory and coercive in nature. So those who violate these Duties will be punished. Obeying the laws of the state, paying taxes, assisting the administrators in the maintenance of law and order etc., are some of the important legal Duties of a citizen. Legal Duties are further classified into positive and negative.

1. Positive Duties: When a citizen exercises his Duties to strengthen the social progress and welfare, they are known as positive Duties. Obedience to the laws of the state, defending the country paying taxes, etc., are some of the examples for positive aspects of legal Duties. These Duties aim at extending cooperation to the government in realizing the objectives of the state.

2. Negative Duties: When a citizen abstains from doing an activity as prohibited by the laws, it is said to be an example of negative responsibility. Negative responsibilities keep the people from not doing certain activities. The government, on behalf of the state, makes several regulations in this regard.

Question 6.
How citizenship is lost?
Answer:
Citizens loose their citizenship under the following conditions:
1. Renunciation: A person is deprived of his citizenship, if he wishes to become the citizen of another state. One will lose the citizenship of ones parent state and may become the citizen of a foreign state by naturalization. In India, the Constitution prescribes that a person Who voluntarily acquires citizenship of any other state will no longer be an Indian citizen.

2. Marriage: Generally a woman lose her citizenship when she marries an alien. However, some states allow retention of citizenship. For instance in Britain, there is an option to retain British citizenship who marries an alien.

3. Accepting Foreign Service: A person may lose his citizenship when he enters into the service of another state. If a person accepts a permanent job in the government of a foreign state, he foregoes the citizenship of his native state.

4. Obliging Foreign Decorations or Titles: When a citizen obliges to receive foreign decorations or titles, it may lead to the forfeiture of his Citizenship.

5. Prolonged Absence: Prolonged absence in the native state beyond a certain period may lead to the loss of citizenship. In some states like France and Germany citizens who are absent themselves from their native country for more than ten years will loose their citizenship.

6. Treason or Crime: Involvement of a citizen in a serious crime and subsequent proof of his action will aIo lead to the loss of citizenship. Especially those persons who directly or indirectly participate or extend assistance to anti-state, anti-social, and anti-governmental activities, will loose their citizenship by a special notification’ to that effect.

7. Desertion from Army: Desertion from Army thereby jeopardizing the security of a state leads to the forfeiture of citizenship.

Question 7.
What are the two types of Democracy?
Answer:
Democracy is an important and most significant form of government. The term Democracy is derived from two Greek words namely Demos and Kratio. In Greek language Demos means the people and Kratio means power. Hence Democracy means power of the people.

Definitions of Democracy:
Abraham Lincoln: ‘ Democracy is a government of the people, by the people and for the people’.
J.R. Seeley: “Democracy is a government in which everyone has a share.”

Lord Bryce: ‘Democracy is that form of government in which the ruling power of the state is vested not in a particular class but in the members of the community as a whole”.

Types of Democracy: Democracy is mainly classified into two types, namely Direct Democracy and indirect Democracy. These two types are explained as below.

1. Direct Democracy: When the people themselves directly express their will on public affairs, this type of government is called direct democracy. In direct democracy the citizens are the real makers of the state policy and programme of action. The will of the state is directly formulated by them and not through their elected delegates.

Some direct democratic checks such as referendums and initiatives are in operation today in Switzerland and in a few states of the United States. In some small Cantons of Switzerland, adult citizens meet in any Sunday in April or May and by show of hands, elect their representative offices as also approve the measures they needed.

2. Indirect Democracy: Indirect democracy is also known as representative democracy. In this type of democracy a clear distinction is made between the immediate sovereign and the ultimate sovereign. The legislature which consists of the elected representatives of the people formulates and expresses the will of the state. Hence, the legislature is the immediate sovereign authority. In this type of democracy, the people elect their representatives periodically and review their activities during their full term.

If their activities are proved to be unsatisfactory, the people can withdraw their trust in them and choose new representatives. Representative democracy thus combines efficient administration with popular sovereignty. In representative democracy the parties articulate and organize the will of the people and act as the transmission belt between the government and the governed. In a representative democracy the ultimate source of authority remains the people.

TS Inter 1st Year Political Science Model Paper Set 6 with Solutions

Question 8.
What is essence of secularism?
Answer:
Secularism is essentially an advocacy in independent, separate or distinct ideologies separate from religion. It is a view of Life based on the principle that morality, education, and government should not be related to the religion. It contends that ethical standards of society and norms of governance should be determined exclusively on the basis of the present life and world, not of the divine.

Secularism does not aim at repudiation of religion. It merely affirms that church and state are two separate entities. Secularism enables the individuals to enjoy their religious freedom to their full extent. The state will not interfere in the religious affairs of individuals. Secularism preaches tolerance and kindness. It believes in universal brotherhood of man. Secularism serves as the best means for fostering national units and integrity feelings among the people.

Question 9.
Explain the differences between Written and Unwritten constitutions.
Answer:

Written Constitution Unwritten Constitution
1. Written constitution implies a document or few documents in which the rules regulating the main institutions of Government are written down. 1. Unwritten constitution denotes a sum of customs, conventions, and usages which have not been systematically documented.
2. All the basic principles of the State are clearly written. 2. All the basic principles of the State exist in the form of customs and traditions.
3. Written constitution is framed by a special assembly convened at a particular point of time. 3. Unwritten constitution contains some written elements also in the form of enactments of fundamental charters made from time to time.
4. It is suitable to the educated and literate people. 4. It is suitable to the uneducated and illiterate people.
5. Courts of law protect the liberties of the citizens. 5. Courts of law cannot provide much protection.
6. It is formulated at a particular time. 6. It is evolutionary in nature.
7. It provides political Stability. 7. It could not ensure political stability.
8. It cannot be easily amended. 8. It can easily be amended.
9. It is useful to federal states. 9. It is advantageous to the unitary states.

Question 10.
What is Judicial Activism?
Answer:
According to the idea of judicial activism judges should use their powers to correct injustices, especially when the other branches a Government do not act to do so. The courts play an active role In’ shaping social policy on such issues as civil rights, political unfairness protection of individual rights, and public morality.

Judicial activism is policy-making function of judiciary in competition with poilcy making by the legislative and executive. This element is associated with the doctrine of judicial review. The essence of true judicial activism lies in rendering decisions by the judiciary which are in tuhe with the temper and tempo of the times.

Behind every judicial decision, judicial activism and judicial restraint are the two aspects that describe the philosophy and motivation. The concept of judicial activism is the polar opposite of judicial restraint. Judicial activism refers to a theory of judgement that takes into account the spirit of the law and the changing times, whereas judicial restraint relias on a strict interpretation of the law and the importance of Legal precedent.

Judicial activism is a dynamic process of judicial outlook in a changing society. Arthur Schlesinger jr. intrõducéd the term ‘Judicial activism’ in 1947. According to BlacWs Law Dictionary, ” Judicial activism is a judicial philosophy which motivates judges to depart from. traditional precedents in favour of progressive and new social policies”.

Question 11.
What is Parliamentary form of Government?
Answer:
In a parliamentary system a clear distinction is made between the head of the state and the head of the Government here, the head of the state King or Queen in Britain or President of India, possesses nominal or titular authority whereas real authority rests with the government of which Prime Minister is the head. Example : Australia, Canada, Japan etc.

Parliamentary form of government as a system in which the real executive, the cabinet, is immediately and legally responsible to the Legisture for its political policies and acts, ultimately responsible to the electorate.

Features of Parliamentary Governments:
a) Nommai and Real executives: In the parliamentary form of government there should be two kinds of executive in the Political system one of them, National Executive is the Head of the state and other one is real executive, Head of the government is president and Head of the government is the Prime Ministers.

b) Coordination between the Legislature and executive: Another important feature a mong, is that, there is a close relationship and coordination between legislature and executive bodies of the government. The executive members are selected from the Legislature and so executive remained as responsible for the Legislature for all its acts.

c) Significant role of the prime minister: In a parliamentary form of government, the prime minister hold the real executive authority. He holds the government as cornerstone. He has the authority to from the council of ministers and also has right to reshuffle and dissolve the government.

d) Collective responsibility: The most important feature of the parliamentary government works on the principle of collective responsibility. It means the ministers enjoy the office only as long as they have confidence of the parliament.

e) Individual responsibility: In a parliamentary government, every minister is individually responsible to the Legislature for the efficient conduct of his department or office. In case there is any lapse in the administration, the ministers are personal answerable to parliament.

f) Dissolution of Lower House: The head of the state can dissolve the lower house on the recommendation of the prime minister. If deadlock rises between cabinet and Legislature they can appeal to the electorate through elections.

g) Effective opposition: In a parliamentary form of government opposition party is considered as soul of the democracy. If the ruling party loses its confidence in the Legislature, opposition party is the alternative to form a government and it works against ruling party through questioning the acts.

Question 12.
Define power and explain its different kinds of power.
Answer:
The concept of power has become a key concept covering all aspects of politics. Suppose the politics is viewed as the process of resolution of the conflict. In that case, the distribution of power within a political community determines how the conflict is to be resolved and whether the resolution is to be effectively accepted by all parties.

Definition of power:

  • H.V. Wiseman defined power as “the abilits’ to get one’s wishes carried out despite resistance”.
  • Hans J.Morgenthan defined power as “Man’s control over the minds and actions of other men”.
  • Edward A.Schiils defined power as ‘The ability to influence the behaviour of others in accordance with one’s own ends”.

Different kinds (or) Forms of power:
1. Political Power: Political power refers to the influence exercised by formal and informal órgans of the state. Power, in politics, is always political power, power of the state, power of the government, and power of the laws through which government operates. But these formal organs, in turn, are influenced by the informal organs which not only take the form of political parties in power and in opposition, but also large number of pressure groups, public opinion, popular movements, mass media, etc. Therefore, the comprehensive analysis of power goes beyond formal organs of the state and includes informal organs of the state.

2. Technological Power: Technology, in modern times, has become an important element in the exercise of power. In recent times, the down of intelligent machine in the form of Artificial intelligence (AI) revolution will have immense influence on man, society, and politics. The computers, unintelligeñt machines, are already doing much of the work in contemporary democracies. Political parties rely on large automated database fo help run their campaigns.

Governments increasingly utilize big data systems to manage and deliver health care and other public services. Our dependence on the technology leaves us ripe for exploitation. Its visible signs are fake news and the micro-targeting of voters with machine-generated messages designed to trigger their individual prejudices so as to influence voting behavior. Therefore, whoever gains paper hand technologically will have a decisive influence on politics.

3. Economic Power: Economic power is the power derived from the possession of wealth, especially the major means of
production and distribution. Economic power plays a vital role in decision-making processes in liberal democracies. If a nation possesses abundant natural and other resources, it will have more economic power. The major newspapers and TV channels are owned by a handful big business houses who take full advantage of these media to promote opinion which suits their opinion. Consumer culture is promoted in a big way to suit their business interests.

4. Ideological Power: Ideological power represents the manipulative power of the dominant or ruling classes which hold sway on the thinking and emotions of the people and try to create an illusion of consent. Thus, the people are led to believe that they are governed with their approval while they are actually continued to be governed according to the designs of the ruling classes.

This ideological domination by the ruling class through the consent of the ruled is conceptualized as ‘hegemony by the Italian Marxist, Antonia Gramsci (1891 – 1937). Thus, an outstanding feature of political ideology is that it provides legitimacy to the ruling classes and helps them maintain their stronghold or political power. When people are made to believe that a particular system of government is the best system, they will not be inclined to challenge the authority of the ruling classes.

5. National Power: From the point of view of realist’s perspective, politics is a struggle for power’. Whatever may be the ultimate aims of international politics, power is the immediate aim’. In the context of international relations, the national power implies a sovereign state (s) attempt to influence other sovereign state (s) in terms of achieving its own goals in international affairs. In this context, the terms like ‘uni-polar’, ‘Bi-polar and ‘multi-polar’ world systems represent the world politics as dominated by one, two, and many nations respectively.

There are different methods of exercising national power through force, influence, and authority. Force is the explicit threat or the use of military, economic, and other instruments of coercion. Influence is the use of instruments of persuasion in order to alter the behavior of other nations. Authority is the compliance by one nation to the directives issued by another nation nurtured by the perception of respect, solidarity, affection, affinity, leadership, knowledge, and expertise.

TS Inter 1st Year Political Science Model Paper Set 6 with Solutions

Section – C
15 x 2 = 30 Marks

Note: Answer any FIFTEEN of the following questions in not exceeding 5 lines each. Each question carries 2 Marks.

Question 1.
Aristotle.
Answer:
The Ancient Greek philosopher Aristotle is regarded as the Father of political Science because of his objective and scientific study of the affairs of the government and politics. Aristotle used the term Politics’ to designate the science of the state. He called politics a Master Science. He was the disciple of Plato.

Question 2.
What is Society?
Answer:
Society: Society is a group of men brought together by a system of common ideas, interests, and aspirations.

Question 3.
What is State?
Answer:
The term ‘State is comparatively modern. To the Greeks the term was not known. They used the word Polis’ which we translate as City State. Ancient Romans used the word Civitas’ for State. After the disintegration of the great Roman Empire towards the end of the fifth century, the. Teutonic tribes established their principalities and they used the word Status’ from which the English word ‘State’ is derived. Thus the word Status is a Teutonic word. The exact meaning of the status is society.

Question 4.
Write any two merits of Nationalism.
Answer:

  1. Nationalism made the people obey the government.
  2. It helped in achieving the, progress of a nation in a short period.

Question 5.
Write any two definitions of Law.
Answer:
Political thinkers defined law in different ways which are listed below.

  1. “Law is the command of the sovereign.” – John Austin
  2. “Law is the system of rights and obligations which the state enforces’. – T.H. Green

Question 6.
Mention the names of four types of liberty.
Answer:
Liberty is of in the following types namely:

  1. Natural liberty
  2. Civil liberty
  3. Economic liberty
  4. Political liberty and
  5. National liberty.

Question 7.
What are the implications of Legal Justice?
Answer:
Legal Justice has two implications:

  • It implies that there is just application of the laws in the society on the basis of rule of law.
  • Laws are made in accordance with the principle of Natural Justice.

Question 8.
Meaning of Socialism.
Answer:
Hughan regarded socialism as the political movement of the working class which aims to abolish exploitation be means of collective ownership and democratic management of the instruments of production and distribution. Some writers regarded socialism as a democratic movement meant for promoting justice and liberty and for managing the society on efficient principles.

TS Inter 1st Year Political Science Model Paper Set 6 with Solutions

Question 9.
Natural Rights.
Answer:
Natural rights are those rights which are enjoyed by men by birth. Men enjoyed these rights even before the origin of civilized society. The society and the state recognized and respected these rights. John Locke, who propounded the theory of natural rights, claimed that rights are pre-social and pre-political in nature. He cited the right to life, right to liberty and right to property as the basic natural rights. The state cannot deprive men of these rights.

Question 10.
List out two conditions of loss of citizenship.
Answer:

  • Renunciation: A person is deprived of his citizenship if he wishes to become the citizen of any other State.
  • Marriage: Generally a woman lose her citizenship when she marries an alien.

Question 11.
What is public opinion?
Answer:
Public opinion occupies an important place in democratic states. People express their opinion in times of elections or through the legislatures. They obey and follow those laws which are formulated in accordance with the public opinion. If the government acts against the wishes of people it has to face the consequences. Public opinion keeps the government responsible and responsive. It keeps the government alert and vigilant in its functioning.

Question 12.
What is Secularism?
Answer:
Secularism is one of the characteristic feature of a modern state. The concept of secularism was popularised by the state authority to control the religion and religious authority over the safe affairs. Secularism is an important social and political phenomenon. Secularism is essentially an advocacy in independent, separate or distinct ideologies separate from religion.

Question 13.
Write about any two features of Secular Stale.
Answer:
Features of Secular State: Secular State comprises the following features.
1. No place for religion: Secular Stateš does not assign significance to any particular religion. It will not make laws or implement them on religious grounds.

2. Equal status: Secular state accords equal status to its people. It makes no differentiation between individuals on the grounds of their caste, colour, community, religion, race, region, language etc. As a result, people will have satisfaction and extend cooperation to the government in the implementation of various policies and programmes. They live together with the fellow members of other religious denominations.

Question 14.
What is Rigid Constitution?
Answer:
Rigid Constitution is one whose provisions can be changed easily. In this system the constitutional amendment methods are different from those of ordinary laws. There will be a special procedure for amending the provisions of the Rigid Constitution. The Rigid Constitution will have firmness due to its special procedures of amendment. Ex.: Constitution of United States of America.

Question 15.
Council of States.
Answer:
Council of States is the Upper House of indian Parliament. It is also known as the Rajya Sabha. It consists of 250 members out of them 238 members are elected by the members of state legislative assemblies through the system of proportional representation by means of single transferable vote. The remaining 12 members are nominated by the President of India from the fields of Arts’, Literature, Social Service, Cooperative Movement, Science and Technology. It is a permanent house and it can never be dissolved. Each member shall remain in office for a period of 6 years. For every 2 years 1/3 of its members shall retire.

TS Inter 1st Year Political Science Model Paper Set 6 with Solutions

Question 16.
What is meant by Plural Executive?
Answer:
Among various types of Executive, Plural Executive is one. If the executive powers are exercised by a body of persons having coequal authority, it is called “Plural Executive”. The Swiss Federal Council and the President in the former U.S.S.R. are the examples of Plural Executives. This Plural Executive combines the merits of both the parliamentary and the presidential executives, avoiding their defects.

Question 17.
Federal Government.
Answer:
Governments are classified into Federal and Unitary on the basis of the distribution of powers between the Centre and the States. A federal system is one in which the powers of the government are distributed constitutionally between the Centre and the State Governments. Ex: America, Switzerland, etc. Meaning: The term “Federatloñ” is derived from a Latin word “Foedus” which means “Treaty of Agreement”.

Question 18.
No-Confidence Motion.
Answer:
No,- confidence motion is an important power of the Legislature, especially in a parliamentary system of government in which the legislature exercises control over the executive for all its decisions over policies, the Council of Ministers are directly responsible to the Lok sabha in India and to the House of Commons in Britain where the parliamentary system is in existence. The Lok Sabha in India and the House of Commons in Britain can fail the government by passing the direct vote of No – Confidence against the prime minister and his Ministers.

Question 19.
What are the Informal organs of power in the state?
Answer:
These are the eight main informal organs of power in the state. They are popularly known as agents of political socialization. They are namely:

  • The family
  • The school
  • Peer groups (or) Reference groups
  • Employment experiences
  • Mass – media
  • Government and Party agencies
  • Symbols
  • Direct contact.

Question 20.
What is equality of Opportunity?
Answer:
The idea of equality refers to the equality of rights and opportunities.

  1. Harold Laski, in his book, ‘A Grammar of Politics’, mentioned that Equality implies fundamentally a leveling process.
  2. According to Barker, equality implies “Equal rights for all the people and abolition of all special rights and privileges’.

TS Inter 1st Year English Model Paper Set 7 with Solutions

Thoroughly reviewing TS Inter 1st Year English Model Papers Set 7 helps in understanding the examiner’s expectations.

TS Inter 1st Year English Model Paper Set 7 with Solutions

Time: 3 Hours 15 Minutes
Maximum Marks: 100

Section – A

Question 1.
Annotate ANY TWO of the following in about 100 words. 2 × 4 = 8M
a) No words could be invented for such supreme happiness, eclipsing all other feelings.
b) Yes. My first rank slipped to the second.
c) It was a capital idea of mine-that it was!
Answer:
a) Introduction : This sentence is taken from Roger Bannister’s inspirational essay The First Four Minutes. It is his personal experience.

Context & Explanation : Finally Bannister reached the place where no man had yet ventured. He achieved it in his first attempt of the year. He was the first man to run the race of one mile in 3 minutes 59.4 seconds. He did on May 6th, 1954. Therefore he became free from the burden of athletic ambition. There were no words for describing his happiness. His joy was boundless. It eclipsed all other feelings.

Critical Comment : Here, he describes his elation for his success.

b) Introduction : This sentence is taken from the prose piece, Father, Dear Father written by Raj Kinger. Actually this is an article published in the English daily, The Hindu.

Context & Explanation : Rahul is the class topper in his school. His first rank slips to the second. Admitting the guilt, he writes a letter to his father. His father’s advice to think before studying, before answering the papers makes him think and think. The word think makes him reflect on several issues including many pitfalls in our education system. Further, he says that the sense of life is not taught to him. He feels that the education should give a feel of life to him and should be useful in life.

Critical Comment : Rahul, the class topper in his school, presents his anguish over the present education system through a letter to his father in this context.

c) Introduction : This line is taken from the one-act play, Box and Cox written by John Maddison Morton. This play is regarded as the best farce of the nineteenth century.

Context & Explanation : Mrs. Bouncer is by nature covetous lady. It is this trait of personality that makes her let out a single room to two different persons simultaneously, taking unadvantage of their different professions and callings. By this, she is able to earn double income from the same room. She takes the opportunity thinking it as a capital idea. Practically, nobody can imagine such a thing. As soon as Cox leaves the room, she gets busy in the room to put his thingS out of Mr. Box’s way.

Critical Comment : Here, Mrs Bouncer feels proud of herself to have got an idea to rent out the room to two different people at the same time.

TS Inter 1st Year English Model Paper Set 7 with Solutions

Question 2.
Annotate ANY TWO of the following in about 100 words. 2 × 4 = 8M
a) Have you sighted anyone.
With shadows in his dusky eyes ?
b) Success is failure turned inside out.
c) And fare thee weel, my only
Luve and fare thee weel a while !
Answer:
a) Introduction : These are the opening lines of the poem, “The Beggar” written by Dr. Ammangi Venugopal, a popular Telugu poet. He has written in Telugu as Bichchagadu. it is translated into English by Elanaaga, (Dr. Surendra).

Context & Explanation : The poem projects the intense grief and suffering of the farmers. A farmer today is misery incarnate. His eyes speak volumes about farmers’ sorrow. The poet minces no words in highlighting their woes. He opens the poem with a question. It identifies farmers with dark eyes that are filled with the shadows of their struggles. The reader, addressed as ‘you’, is forced to understand and sympathise with farmers. Therefore the lines play an important role in initiating the thought process effectively.

Critical Comment : The poet portrays the pathetic plight of farmers. He is questioning the reader to make him to think about the farmers.

b) Introduction : This wonderful line is taken from the classic inspi-rational poem, ‘Keep Going’, penned by Edgar Albert Guest, a well- known people’s poet.

Context & Explanation : The poem is all about perserverance, determination and will-power not to give up when one is swimming against the tide. Every failure is a learning opportunity to turn it into success. It is because success and failure are made of the same cloth. Beneath success there is failure and beneath failure there is success. Be optimistic that you can acquire success.

Critical Comment : The poem reminds us that there are seeds of success in every failure. That is why we mustn’t quit.

c) Introduction: This couplet is taken from the beautiful lyric, ‘A Red Red Rose’ written by Robert Burns. It is one of the best lyrics of English poetry. It blends the eternity of love with the mortality of life.

Context & Explanation : The speaker says that he will love his beloved forever. Even after the seas get dried up, all the rocks melt, and the sands of life exhaust their love stays alive. It will last forever. For the present, the speaker says good bye only to return soon, though the journey is to a far off place the poem blends the eternity of love with the mortality of life.

Critical Comment : The poet makes several promises to love his beloved forever.

Question 3.
Answer ANY TWO of the following questions in about 100 words each. 2 × 4 = 8M
a) Who is taking the noble mission of Thimmakka forward and how ?
b) The whole speech is on human traits. Comment with reference to Booker T Washington’s Two Sides of Life.
c) Write a paragraph on the present day education system as described in Rahul’s letter.
Answer:
a) Planting more and more plants is the noble mission of Thimmakka. She expanded her mission from 10 banyan saplings to over 8000 other trees. Her outstanding work earned her the name Saalumarada, which means a row of trees. Now, she is 100 plus. Her noble mission is taken forward by her faster son, Sri Umesh. He has been planting and tending to trees along the roads, in schools, public places and on the mountains, and hill tops also runs the PRITHVI BACHAO movement successfully. He maintains nursery and distributes plants to the farmers who are interested in growing plants. So, the adopted son adopts her noble mission of planting saplings.

b) Booker T Washington is a world-famous Afro-American writer, activist and educator. His collection of speeches comes in the form of Character Building. A selection from that celebrated book is our present lesson. It discusses the two sides of life. It recommends the positive side. Yes. The lesson discusses various traits of man. Indeed, it is about and from Character Building. It focuses on the optimism and pessimism mainly. It touches upon the need to be honest and frank. It dwells upon the world’s view of positive and negative sided people. It analyses man’s view of education. It highlights the desirable traits of teachers. Thus the entire piece is devoted to a debate on multiple human traits.

c) Raj Kinger’s Father, Dear Father is a heart wrenching letter addressed to a father by his son, Rahul. In his letter, Rahul condemns our educational system and explains the reason for losing his first rank. If was due to his disagreement with his teacher regarding an answer in English Grammar. Although the teacher -was wrong, he was adamant that he was correct. Rahul criticizes such an education system which curbs in-dependent thinking and encourages blind adherence to whatever the teacher teachers. Thus, he condemns the emphasis placed on examinations, marks and ranks. For him practical education matters more than theoretical.

TS Inter 1st Year English Model Paper Set 7 with Solutions

Question 4.
Answer ANY TWO of the following questions in about 100 words each. 2 × 4 = 8M
a) Discuss the aptness of the title “The Noble Nature” to the poem.
b) Seeing helps one better in understanding then listening to. Justify the statement with reference to the poem, ‘Happiness’.
c) List the abilities a farmer is endowed with, according to the poem.
Answer:
a) The Noble Nature is one of the most popular lyrics of Ben Jonson. He is regarded as one of the major dramatists and poets of the Seventeenth Century. In this poem, he seeks to explain what makes man’s life noble. The core meaning of the poem centres around this single idea. In just ten lines of the poem, the poet says twice. Man better be; and life perfect be. And to make Man better or life perfect, he advises one to lead a meaningful life of light-like that of a lily. Thus, the focus of the poem is clearly on making Man better. To explain this point, examples of the oak and the lily are used. Hence, the title, The Noble Nature, suits the poem well. The poem says leading meaningful life even for a short while is worthier than leading a long life with neither charm nor value.

b) Carl Sandburg’s poem, ‘Happiness’ conveys a beautiful message. It is extracted from his collection, Chicago songs. It shows how the narrator tries to find out the real meaning of happiness and his ultimate realization.

This seems like a more light hearted poem. The poet depicts the narrator’s experience. He asks people what they think of happiness. The first two he asks are the people who should know what happiness is. But, both look at him as if he is trying to fool them. He then ventures out to observe some of the lower class. He examplifies, what he sees, his image of happiness.

The poem centers around the difference between the lower and the upper class. He favours the lower class for their simplicity. They value the things in their lives. It is proved in the lives of Hungarians. They show him what happiness is. They enjoy then food, drink, music and fun. At last, seeing Hungarians helps the narrator in understanding how they spend happy moments under a tree. Then he realizes what happiness is. Even if they are not very well educated or wealthy, they stand as a symbol of sharing and helping mentality people.

c) Dr. Ammangi Venugopal is a creative genius. He is well aware of the abilities of a farmer. In his poem, The Beggar, the poet minces no words in depicting farmers’ abilities. They are the food providers to all. Their eyes are dark with shadows of their struggles and sufferings. Their backs are bent with burden. Their hands are soiled and severed. Their feet bleed. Yet, their ability to produce food and satisfy others’ hunger remains fully active. They work hard and help others. They are capable of feeding millions. They reduce and satisfy the hunger of even skies. Thus, the poem is endowed with the abilities of a farmer.

Question 5.
Answer ANY TWO of the following questions in about 100 words each. 2 × 4 = 8M
a) Were the three brothers successful in executing their tricks ? Support your answer.
b) Write a paragraph on how Alan and his parents felt excited when he was chosen to play for the school cricket match.
c) “A spirited discussion springs up between a young girl and a colonel”. Discuss.
Answer:
a) No. The three brothers failed in their efforts. The famous Chinese folk tale “The Short-sighted Brothers”, explains their failure. All the three brothers were very short-sighted. Once, they wanted to prove the power of their sight. They were to read an inscription. Each learnt secretly from the monk about the writing. They thought they could outsmart the others. They visited the monastery the next day. They started READING from the TABLET. Each one READ out. Then the monk came out. He told them that the TABLET was not yet put up ! They READ from the TABLET that WAS NOT there ! Their folly was exposed. They realised it!

b) Arthur Henry Mee is famous as an eminent educator and journalist. His short story “Playing the Game” is at once didactic and entertain¬ing. Its gripping narration offers a pleasant reading experience. Alan is the lead character. He was a schoolboy. He loved cricket. His parents supported and encouraged him. Alan’s father actually helped Alan practise bowling. He commented that Alan was shaping as a good bowler. Then, Alan was selected to play in his school team. That was a well deserved opportunity. Hence Alan felt excited. That made Alan’s parents doubly excited. That is just natural and justifiable on their part.

c) “The Dinner Party”, by Mona Gardner, showcases the emotional strength of women. Deeds, not empty words, prove this point beyond anyone’s doubt. A colonial officer hosts the dinner party. The guests are just twenty. During the party, an animated discussion arises between a young girl and a colonel. The girl says women have advanced a lot from their earlier era of screaming at the sight of a mouse. The colonel contradicts her stand. He asserts men have an extra ounce of nerve control in a crisis. And he adds that ounce counts a lot. But the girl is right. The story proves at the end how strong a woman is in crises!

TS Inter 1st Year English Model Paper Set 7 with Solutions

Section – B

Question 6.
Read the following passage carefully and answer ANY FOUR questions given after it in a word or a sentence each. 4 × 1 = 4M
Thus, things were moving happily. But the farmers were perturbed. They observed the lives of people on the other side of the river Krishna ruled by the British and found that people were happy there. There was no drudgery, no penalties-no beatings either. But if the people of Ramasagaram were to migrate to that side leaving the households and assets earned by their ancestors and their caste trades as well, how would they live ?

Questions:
i) “Thus, things were moving happily.” ‘Happily’ to whom ?
ii) What did the farmers observe ?
iii) What did the farmers find out ?
iv) What was the reason for the vast difference in the lives of Ramasagaram people and that of those living on the other side of the river ?
v) Was it possible for the people of Ramasagaram to migrate to the other side of the river ?
vi) Give the word from the passage that means disturbed / worried /anxious.
Answer:
i) moving happily to the police personnel
ii) observed the lives of people on the other side of river Krishna ruled by the Britishers.
iii) found that people on the other side of river Krishna were happy
iv) The reason : Ramasagaram was under the Nizam’s rule and the, village on the other side of river Krishna was ruled by the Britishers.
v) No, it was not possible for them to migrate.
vi) ‘perturbed’

Question 7.
Read the following passage carefully and answer ANY FOUR questions given after it in a word or a sentence each. 4 × 1 = 4M

Mistakes can Make Miracles

Mistakes at times can turn out to be a blessing in disguise. One such error led to the establishment of the Nobel Prize, the most coveted award in the world. When Ludwig (Alfred’s brother) died in 1888, a French newspaper erroneously confused the deceased’s identity with that of Alfred Nobel, the inventor of dynamite that brought him enormous money. As a result, it published a scathing obituary entitled The Merchant of Death is Dead. Alfred thus had the rarest opportunity of reading his own death report (obituary). His troubled conscience said to himself, “Is this how posterity is going to remember me ? No, 1 must do something. The huge fortune I made must go to promote peace in this world. “What followed is history. Nobel prizes in Peace, Literature, Physics, Chemistry and Biology/Medicine were thus born !

Questions:
i) What was the error that led to the establishment of the Nobel Prize ?
ii) How did the French paper describe Alfred Nobel ?
iii) Who died in 1888 ?
iv) What was the rarest opportunity Alfred Nobel had ?
v) How did Nobel try to change his image ?
vi) Write the word used in the passage to mean death report.
Answer:
i) confused the deceased’s identity with that of Alfred Nobel
ii) as the Merchant of Death
iii) Ludwig Nobel, Alfred’s brother
iv) to read his own death report
v) by trying to promote peace in the world and by establishing the Nobel Prize.
vi) obituary

Section – C

[Note : Answers of this section must be written at one place in the same Serial Order.]

Question 8.
Match ANY EIGHT of the following words in Column – A with their meanings in Column – B. 8 × 1/2 = 4M

Column A Column B
i) transgression a) irritation, disappointment
ii) philosopher b) nervous, rude
iii) plucking c) real meaning
iv) harbinger d) identify, be familiar with
v) essence e) pulling something out
vi) frustration f) skill
vii) recognize g) doing wrong, violation of a code
viii) nervy h) at risk to be lost
ix) craft i) truth-seeker, logician
x) at stake j) indication

Answer:

Column A Column B
i) transgression g) doing wrong, violation of a code
ii) philosopher i) truth-seeker, logician
iii) plucking e) pulling something out
iv) harbinger j) indication
v) essence c) real meaning
vi) frustration a) irritation, disappointment
vii) recognize d) identify, be familiar with
viii) nervy b) nervous, rude
ix) craft f) skill
x) at stake h) at risk to be lost

TS Inter 1st Year English Model Paper Set 7 with Solutions

Question 9.
Identify the parts of speech of ANY EIGHT of the following underlined words. 8 × 1/2 = 4M
I had (1) a moment (2) of (3) mixed joy (4) and anguish, when (5) my mind (6) took over. It (7) raced well (8) ahead of my body and (9) drew my body compellingly (10) forward.
Answer:
1) had – verb
2) moment – noun
3) of – preposition
4) joy – noun
5) when – pronoun
6) mind – noun
7) it – pronoun
8) well – adverb
9) and – conjunction
10) compelling – adverb

Question 10.
Fill ANY EIGHT of the following blanks with a, an or the. 8 × 1/2 = 4M
1. Panaji is ……………….. capital of Goa state.
2. ……………….. moon is ……………….. symbol of pleasantness.
3. I have given ……………….. one rupee coin to ……………….. beggar.
4. ……………….. rabbit runs very fast.
5. Is there ……………….. bank near here ?
6. ……………….. talent of ……………….. writer can’t be underestimated.
7. ……………….. simplicity which Gandhi followed is taken as ……………….. example everywhere.
8. I interviewed ……………….. M.P. in ……………….. evening.
9. Did you get married after leaving ……………….. university ?
10. Would you like to be ……………….. actor ?
Answer:
1) the
2) The, the
3) a, the (a)
4) The/A
5) a
6) The, a /the
7) The, an
8) an, the
9) the
10) on

Question 11.
Fill in ANY EIGHT of the following blanks with suitable prepositions. 8 × 1/2 = 4M
1. It rained ……………….. two days.
2. Switch ……………….. the light, please, it is quite dark here.
3. The trains are seldom ……………….. time.
4. We live ……………….. the fifth floor.
5. She is familiar ……………….. computer hardware.
6. I have not slept properly two ……………….. days.
7. He trembled ……………….. fear when he was caught.
8. Looking forward ……………….. seeing you at the meeting.
9. Most foreigners dream ……………….. visiting India.
10. She insisted ……………….. joining us.
Answer:
1) for
2) on
3) on
4) on
5) with
6) for
7) with
8) to
9) of
10) on

TS Inter 1st Year English Model Paper Set 7 with Solutions

Question 12.
Fill ANY FOUR of the following blanks with suitable forms of the verbs given in brackets. 4 × 1 = 4M
1. Surya Namaskar ……………….. (consist) of twelve postures.
2. Gandhi ……………….. (influence) by the writings of Tolstoy.
3. Bandla Sirisha ……………….. (be) the first Telugu woman (third Indian Origin woman) to go into the space on July 11, 2021.
4. Rohit Sharma ……………….. (bat) for two hours when rain interrupted
the match. ,
5. The meeting ……………….. (start) by 10.00 a.m. tomorrow.
6. Usually my father ……………….. (take) rice for lunch, But now he ……………….. (take) chapatis.
Answer:
1) consists
2) was influnced
3) is
4) had been batting
5) will have started
6) takes … is taking

Question 13.
Rewrite ANY FOUR of the following sentences as directed. 4 × 1 = 4M
i) The workers called off the strike.
(Change the sentence to passive voice.)
ii) Every sentence can’t be changed into the passive voice.
(Change the sentence to active voice.)
iii) The teacher said to Kavitha, “What does the word ‘corruption’ mean ?”
(Change the sentence to indirect speech.)
iv) A foolish friend can be more dangerous than a wise enemy. (Change the sentence to positive degree.)
v) You have done your homework, ……………….. ?
(Add an appropriate question tag)
vi) I am unable to answer your question, ……………….. ?
(Add an appropriate question tag)
Answer:
i) The strike was called off by the workers.
ii) One (We) can’t change every sentence into the passive voice.
iii) The teacher asked Kavitha what the word corruption meant.
iv) A wise enemy cannot be so dangerous as a foolish friend.
v) haven’t you
vi) am I (unable = not able)

Question 14.
Rewrite ANY FOUR of the following sentences correcting the underlined errors. 4 × 1 = 4M
1. Raghu is my older brother.
2. The streets of Hyderabad are wider than Warangal.
3. Suma is an popular anchor.
4. I waited for a hour.
5. Sun rises in east.
6. Onions cost Rs. 20 kilogram.
Answer:
1. Raghu is my elder brother.
2. The streets of Hyderabad are wider than those of Warangal.
3. Suma is a popular anchor.
4. I waited for an hour.
5. The sun rises in east.
6. Onions cost Rs. 20 a kilogram.

Question 15.
Supply the missing letters to ANY EIGHT of the following words. 8 × 1/2 = 4M
i) hi_ _top
ii) ba_ _an
iii) r_ _tine
iv) conc_ _ve
v) m_ _ntain
vi) mi_ _ion
vii) in_ _edible
viii) mons_ _n
ix) ca_ _y
x) reco_ _tion
Answer:
i) hilltop
ii) banyan
iii) routine
iv) conceive
v) mountain / maintain
vi) mission/million
vii) incredible
viii) monsooni
x) carry
xi) recognition

TS Inter 1st Year English Model Paper Set 7 with Solutions

Question 16.
Identify the silent consonant letters in ANY EIGHT of the following words. 8 × 1/2 = 4M
i) yolk
ii) would
iii) pneumonia
iv) consign
v) drawing
vi) what
vii) knead
viii) doubt
ix) island
x) aisle
Answer:
i) yolk – l
ii) would – l
iii) pneumonia – p
iv) consign – g
v) drawing – w
vi) what – h
vii) knead – k
viii) doubt – b
ix) island – s
x) aisle – s

Question 17.
Write ANY FOUR of the following transcriptions using ordinary English spelling. 4 × 1 = 4M
TS Inter 1st Year English Model Paper Set 7 with Solutions 1
Answer:
i) desperate
ii) lull
iii) impelled
iv) resistance
v) pride
vi) faint

Question 18.
Circle ANY FOUR of the words that sound different from the other words in that set with regard to the sounds of the bold letters. 4 × 1 = 4M
i) head bead lean
ii) time it I
iii) way waste want
iv) eye get check
v) march start nation
vi) being stage morning
Answer:
i) head
ii) it
iii) want
iv) eye
v) nation
vi) stage

Question 19.
Mention the number of syllables in ANY FOUR of the following words. 4 × 1 = 4M
i) pension
ii) source
iii) confer
iv) captivate
v) modest
vi) contribution
Answer:
i) 2 – disyllabic
ii) 1 – monosyllabic
iii) 2 – disyllabic
iv) 3 – trisyllabic
v) 2 – disyllabic
vi) 4 – polysyllabic

TS Inter 1st Year English Model Paper Set 7 with Solutions

Question 20.
a) Observe the pie chart given below. It contains information about the mode of transport used by students of a certain junior college. Write a small paragraph. 1 × 4 = 4M

TS Inter 1st Year English Model Paper Set 7 with Solutions 2
b) Read the following paragraph and transfer the information into a flow chart.
The following paragraph describes how clothes are washed. Draw a flow chart based on the information given. First, fill a bucket half full with water. Then, add a spoonful of washing powder. Stir vigorously till the power mixes with water and forms foam. Put the unwashed clothes into it. Wait for fifteen minutes. Take out clothes and scrub with a brush to remove stains. Now, rinse the clothes with clean water.

Wring out the clothes gently by twisting and compressing them. This removes excess water from the clothes. This saves the time of drying. Now dry the washed clothes by putting them on the clothes line. Collect the washed and dried clothes later.
Answer:
a) Mode of Transport of Students
The given pie chart presents the mode of transport used by students of a particular junior college. A major part of them 40% – use the public transport, i.e. bus. A half of the share of bus, that is 20% of them travel by autorickshaws. Two wheelers and cars carry 15% each of the students. Just 10% of them use the cleanest and the healthiest mode – walking.
(Or)
b) How to wash clothes ?
TS Inter 1st Year English Model Paper Set 7 with Solutions 3

TS Inter 2nd Year Commerce Question Paper March 2023 with Solutions

Thoroughly analyzing TS Inter 2nd Year Commerce Model Papers and TS Inter 2nd Year Commerce Question Paper March 2023 helps students identify their strengths and weaknesses.

TS Inter 2nd Year Commerce Question Paper March 2023 with Solutions

Time: 3 Hours
Maximum Marks: 100

Part – I (50 Marks)
Section – A
(2 × 10 = 20)

Answer any two of the following questions in not exceeding 40 lines each.

Question 1.
What are the differences between the Money market and the Capital market?

Question 2.
Explain the principles of Management.

Question 3.
Explain the functions of an entrepreneur.

Section – B
(4 × 5 = 20)

Answer any four of the following questions in not exceeding 20 lines each.

Question 4.
What are the differences between the Primary Market and the Secondary Market?

Question 5.
What is E-Banking? Explain any four advantages of E-Banking.

TS Inter 2nd Year Commerce Question Paper March 2023 with Solutions

Question 6.
Define insurance. Explain any four principles of insurance.

Question 7.
How does the government of Telangana extend special support to the SC/ST entrepreneurs in our State?

Question 8.
Explain any five objectives of SEZs.

Question 9.
Define the staffing process and explain the four steps involved in it.

Section – C
(5 × 2 = 10)

Answer any five of the following questions in not exceeding 5 lines each.

Question 10.
What do you mean by Lame duck?

Question 11.
What is Cash Credit?

Question 12.
What do you mean by Overdraft?

Question 13.
What is Fire Insurance?

Question 14.
What do you mean by Bridge Loans?

Question 15.
What do you mean by Cheap Jacks?

Question 16.
What is a Bill of Lading?

TS Inter 2nd Year Commerce Question Paper March 2023 with Solutions

Question 17.
Meaning of Control.

Part – II (50 Marks)
Section – D
(1 × 20 = 20)

Answer the following question.

Question 18.
Ram and Karan are partners sharing profits and losses in the ratio of 3 : 2 respectively. Their Balance Sheet as of 31st March 2021 was as follows:
TS Inter 2nd Year Commerce Question Paper March 2023 with Solutions Q18
On 1st April 2021, they decided to admit Mr. Sharath for 1/5th of the share in profits. The terms of admission are
(a) He has to bring ₹ 20,000 towards capital and ₹ 10,000 towards goodwill in cash.
(b) Furniture is to be depreciated by ₹ 1,000.
(c) Create a provision of ₹ 1,500 for bad debts and debtors.
(d) Appreciate the value of buildings by ₹ 5,000.
Prepare necessary ledger accounts and open the balance sheet of the new firm.

Section – E
(1 × 10 = 10)

Answer any one of the following questions.

Question 19.
On 1st January 2021, Ravi of Hyderabad consigned goods valued ₹ 60,000 to Karan of Karimnagar. Ravi paid cartage and other expenditures ₹ 4,000. On 31st March 2021, Karan sent an account sales with the following information:
(a) 50% of the goods sold for ₹ 50,000.
(b) Karan incurred expenses ₹ 3,000.
(c) Karan is entitled to commission @ 6% on sales. A bank draft was enclosed for the balance due.
Prepare the necessary Ledger Accounts in the books of Ravi.

Question 20.
Hyderabad Youth Club gives you their Receipts and Payments Account and other information and requests you to prepare their Income and Expenditure Account for the year ended 31-03-2021.
Receipt and Payments Account for the year ended 31-03-2021
TS Inter 2nd Year Commerce Question Paper March 2023 with Solutions Q20
Additional Information:
(a) Outstanding subscription on 31-03-2021 ₹ 750.
(b) Subscriptions received in advance on 31-03-2021 ₹ 250.
(c) The value of old furniture sold is ₹ 22,500

Section – F
(2 × 5 = 10)

Answer any two of the following questions.

Question 21.
Distinguish any five differences between the Consignment and Sales.

Question 22.
Rama bought a plant and machine on 1st April 2017 for ₹ 23,000 and paid ₹ 2,000 for its installation. Depreciation is to be allowed at 10% under the straight-line method. On 31st March 2020, the plant was sold for ₹ 12,000. Assuming the accounts are closed at the end of the financial year, prepare the Plant & Machine Account.

Question 23.
From the following details, prepare the Receipts and Payments Account of the NGO’s club for the year ending 31st March 2021.
Opening Balance of Cash – ₹ 3,000
Opening Bank Balance – ₹ 9,000
Subscriptions collected – ₹ 16,000
Entertainment show receipts – ₹ 8,000
Entrance fee received – ₹ 4,000
Computer purchased – ₹ 6,000
Tournament expenses – ₹ 6,000
Entertainment show – ₹ 3,600
Paid for Magazines – ₹ 2,400
Salaries paid – ₹ 2,400
Rent paid – ₹ 8,000
Cash in hand at close – ₹ 3,600

TS Inter 2nd Year Commerce Question Paper March 2023 with Solutions

Question 24.
Write any five limitations of Computerized Accounting.

Section – G
(5 × 2 = 10)

Answer any five of the following questions.

Question 25.
Define the term Depreciation.

Question 26.
What is depletion?

Question 27.
What is account sales?

Question 28.
What do you mean by Entrance fee?

Question 29.
What is the Ratio of Gaining?

Question 30.
What is a Password?

TS Inter 2nd Year Commerce Question Paper March 2023 with Solutions

Question 31.
What is a sheet?

Question 32.
X and Y are partners sharing profit and loss in the ratio of 3 : 2. They decided to admit Z for 1/5th share in profit. Calculate the new profit-sharing ratio of X, Y, and Z.

TS Inter 2nd Year Commerce Question Paper May 2022 with Solutions

Thoroughly analyzing TS Inter 2nd Year Commerce Model Papers and TS Inter 2nd Year Commerce Question Paper May 2022 helps students identify their strengths and weaknesses.

TS Inter 2nd Year Commerce Question Paper May 2022 with Solutions

Time: 3 Hours
Max. Marks: 100

Part – I (50 Marks)
Section – A
(2 × 10 = 20)

  • Answer any TWO of the following questions in not exceeding 40 lines.
  • Each question carries 10 marks.

Question 1.
What are the differences between the Money Market and the Capital Market?

Question 2.
What is a Stock Exchange? Explain its functions.

Question 3.
Discuss the Secondary functions of Banks.

TS Inter 2nd Year Commerce Question Paper May 2022 with Solutions

Question 4.
Define entrepreneur and explain the characteristics.

Question 5.
What are the Principles of Management?

Section – B
(4 × 5 = 20)

  • Answer any FOUR of the following questions in not exceeding 20 lines.
  • Each question carries 5 marks.

Question 6.
What are the differences between the Primary Market and the Secondary Market?

Question 7.
What are the various types of Retail Loans? Explain.

Question 8.
Explain the powers and functions of IRDA.

Question 9.
What are the special provisions enacted by the Telangana state for the MSMEs?

Question 10.
Discuss the benefits of International Trade.

Question 11.
Define Trade. Explain its features.

Question 12.
Explain any five characteristics of Management.

TS Inter 2nd Year Commerce Question Paper May 2022 with Solutions

Question 13.
Explain different types of life policies.

Question 14.
Write about POSDCORB.

Section – C
(5 × 2 = 10)

  • Answer any FIVE of the following questions in not exceeding 5 lines.
  • Each question carries 2 marks.

Question 15.
What is a Capital Market?

Question 16.
What is a Mutual Fund?

Question 17.
What do you mean by bull speculator?

Question 18.
What is Savings Deposits Account?

Question 19.
Write about the Endowment Policy.

Question 20.
What is Time Policy?

Question 21.
What do you mean by Innovation?

Question 22.
Write about Drone Entrepreneurs.

Question 23.
What do you mean by Multiple Shops?

TS Inter 2nd Year Commerce Question Paper May 2022 with Solutions

Question 24.
Write any two differences between Internal Trade and International Trade.

Question 25.
What is Planning?

Part – II (50 Marks)
Section – D
(1 × 20 = 20)

Answer the following question.

Question 26.
Ravi and Kiran are partners sharing profits and losses in the ratio of 3 : 2 respectively. Their Balance Sheet as of 31st March, 2020 was as follows:
TS Inter 2nd Year Commerce Question Paper May 2022 with Solutions Q26
On 1st April 2020, they decided to admit Mr. Bharath for a 1/5th share in profits. The terms of admission are:
(a) He has to bring ₹ 20,000 towards capital and ₹ 10,000 towards goodwill in cash.
(b) Furniture is to be depreciated by ₹ 1,000.
(c) Create a provision of ₹ 1,500 for Bad debts and debtors.
(d) Appreciate the value of Buildings by ₹ 5,000.
Prepare necessary ledger accounts and open the Balance Sheet of the new firm.

Section – E
(1 × 10 = 10)

Answer any ONE of the following questions.

Question 27.
Arjun of Kamareddy and Vittal of Karimnagar are in the consignment business. Vittal sent goods to Arjun ₹ 20,000. Vittal paid freight ₹ 800 and insurance ₹ 700. Arjun met sales expenses ₹ 750. Arjun sold the entire stock for ₹ 30,000 and he is entitled to a commission of 5% on sales. Prepare necessary ledger accounts in the books of Vittal.

Question 28.
Distinguish between consignment and sale.

Question 29.
From the following Receipts and payments account of ‘Sri Kala Nilayam’ for the year ended 31-03-2019, prepare the Income and Expenditure account.
TS Inter 2nd Year Commerce Question Paper May 2022 with Solutions Q29
Additional Information:
(a) Outstanding salaries ₹ 500
(b) Subscriptions outstanding for 2018-19 ₹ 1,000
(c) Depreciate furniture by 10%.

Section – F
(2 × 5 = 10)

Answer any TWO of the following questions.

Question 30.
Raghu & Co. purchased furniture on 1st April 2016 for ₹ 40,000. Depreciation is provided at the rate of 10% under the straight-line method. On 31st March 2020, the scrap of the furniture was sold for ₹ 18,000. Prepare Furniture A/c.

Question 31.
Explain the causes of depreciation.

TS Inter 2nd Year Commerce Question Paper May 2022 with Solutions

Question 32.
From the following details, prepare the Receipts and Payments account:
Opening Balance of cash – ₹ 1,500
Opening Bank Balance – ₹ 4,500
Subscriptions collected – ₹ 8,000
Entertainment show receipts – ₹ 4,000
Entrance fees received – ₹ 2,000
Computer purchased – ₹ 3,000
Tournament expenses – ₹ 3,000
Entertainment show expenses – ₹ 1,800
Paid for Periodicals – ₹ 1,200
Salaries Paid – ₹ 1,200
Rent Paid – ₹ 4,000
Cash in hand at close – ₹ 1,800

Question 33.
Write the differences between the Receipts and Payments account and the Income and Expenditure account.

Question 34.
Explain the features of Computerised Accounting.

Question 35.
Explain the differences between Manual and Computerised Accounting.

Section – G
(5 × 2 = 10)

Answer any FIVE of the following questions.

Question 36.
Define the term Depreciation.

Question 37.
What is the Straight Line Method?

Question 38.
What is Account Sales?

Question 39.
What is the Del Credre Commission?

Question 40.
What is Deferred Revenue Expenditure?

Question 41.
Explain the term Donation.

Question 42.
What do you mean by Entrance fees?

TS Inter 2nd Year Commerce Question Paper May 2022 with Solutions

Question 43.
What is Goodwill?

Question 44.
Radha and Rani shared profits and losses in the ratio of 4 : 3. Manjatha admitted into business for 1/8th share in the future profits. Calculate the new profit-sharing ratio.

Question 45.
What is the Ratio of Gaining?

Question 46.
What is Computerized Accounting?

Question 47.
Write any two Advantages of Computerized Accounting.